You are on page 1of 132

D RU ŠTVO MATEMATI ČARA S RBIJE

M ATERIJALI ZA MLADE MATEMATI ČARE , SV. 47

Igor Dolinka

ELEMENTARNA TEORIJA BROJEVA:


MOJI OMILJENI ZADACI

B EOGRAD , 2007.
DR I GOR D OLINKA, vanredni profesor PMF-a u Novom Sadu
dockie@im.ns.ac.yu
ELEMENTARNA TEORIJA BROJEVA: MOJI OMILJENI ZADACI
Materijali za mlade matematičare, sveska 47
Izdavač: D RU ŠTVO MATEMATI ČARA S RBIJE
Beograd, Kneza Mihaila 35/IV
dms@matf.bg.ac.yu
Za izdavača: DR B RANISLAV P OPOVI Ć
Recenzenti: Z ORAN K ADELBURG, redovni profesor Matematičkog fakulteta
DR
u Beogradu
DR S INI ŠA C RVENKOVI Ć , redovni profesor PMF-a u Novom Sadu
Urednik: DR Z ORAN K ADELBURG
Slog: AUTOR

°
c Društvo matematičara Srbije, 2007.

CIP – Katalogizacija u publikaciji


Narodna biblioteka Srbije, Beograd

511.1(076)

DOLINKA, Igor
Elementarna teorija brojeva: moji omiljeni zadaci / Igor
Dolinka. — Beograd : Društvo matematičara Srbije, 2007
(Kragujevac : Skver). — 132 str. ; 24 cm. — (Materijali za
mlade matematičare / Društvo matematičara Srbije ; sv. 47)

Tiraž 400.

ISBN: 86-81453-62-9

a) Teorija brojeva – Zadaci

COBISS.SR-ID 136647692

ISBN 86-81453-62-9
Tiraž: 400 primeraka
Štampa: ”S KVER”, Kragujevac
Sadržaj

Predgovor 5

Zadaci 9

Nekoliko poznatih teorema 19

Rešenja 21

Tabelarni pregled porekla zadataka 132

3
Predgovor

Na dan kada pišem ovaj predgovor, navršava se tačno 10 godina od smrti Pala
Erdeša (Erdős Pál, Budapest, 26.3.1913. – Warszawa, 20.9.1996.), velikog majsto-
ra teorije brojeva, kombinatorike, i još mnogih drugih oblasti matematike. Kao
što to i naslovna korica sugeriše, ova knjižica posvećena je uspomeni na ovog
velikana. Iz istog razloga, ona sadrži baš 1996 − 1913 = 83 problema. Svojim
višedecenijskim radom u službi kraljice nauka, Erdeš je ostavio nezaobilazan trag,
i predstavlja jednu od najneobičnijih, a ujedno najznačajnijih figura matematike
XX veka.
Zbirka je, kao što naslov kazuje, sastavljena isključivo od elementarnih za-
dataka. To znači da za njihovo rešavanje nisu potrebne metode matematičke ana-
lize, i da se kao pretpostavljeno predznanje uzimaju samo osnovni, klasični rezul-
tati aritmetike i veštine obuhvaćene srednjoškolskim gradivom iz algebre. (Kratak
-
pregled najznačajnijih tvrdenja iz teorije brojeva na koja ćemo se u rešenjima eks-
plicitno pozivati dat je nakon liste zadataka, a pre rešenja.) Uostalom, svi ovde
analizirani zadaci i vode poreklo sa matematičkih takmičenja srednjoškolaca širom
sveta u periodu od 1969. do 1995. godine. Izbor je zaista internacionalan (kao što
-
je i Erdeš bio istinski svetski putnik, gradanin planete), a bliži podaci o poreklu za-
dataka dati su uz njihova rešenja. Pri tome, akronim MMO označava Medunarodnu -
matematičku olimpijadu, smotru mladih matematičara koja se organizuje svake go-
dine počev od 1959. Jedini izuzetak je bila 1980. godina, kada je umesto MMO or-
-
ganizovano nekoliko manjih medunarodnih matematičkih turnira (MMT). Najzad,
skraćenica BMO označava Balkanske matematičke olimpijade, čija je organizacija
počela 1984.
Po kom kriterijumu sam birao zadatke? Teško je odgovoriti na to pitanje, pošto
bi odgovor na njega bio prilično blizak odredenju- (inače potpuno fluidnog) pojma
matematičke lepote. Negde je to lucidnost i elegancija rešenja, lukava igra detekcije
koja vodi željenom cilju. U drugom slučaju, to je estetika jednostavnosti formu-
lacije, prirodnost problema koja privlači pažnju. U trećem, diskretno nagoveštena
mogućnost generalizacije ili suptilnih veza sa ”višom” matematikom (šta god taj
termin označavao), pre svega sa apstraknom algebrom, analitičkom teorijom bro-

5
jeva, kombinatorikom. Uglavnom, kriterijum je bio dakako subjektivan, i osla-
njao se više na osećaj i iskustvo (mnoge od narednih zadataka sam i sam rešavao
pripremajući se svojevremeno za matematička takmičenja), nego na neke racional-
ne argumente.
Ključna primedba koju treba da pomenem u ovom predgovoru jeste da ovo nije
-
knjiga posvećena ispisivanju rešenja odredenog skupa zadataka na najkraći mogući
način. Naprotiv. Ono što mi se čini kao suština ove zbirke jeste pokušaj da se u naj-
većem broju slučajeva da metodološka analiza postupka rešavanja datih zadataka.
Drugim rečima, pokušao sam (kad god je to bilo moguće) da što verodostojnije
prikažem analitički, deduktivni način razmišljanja u procesu otkrivanja rešenja, da
pružim kakav-takav uvid u realne misaone tokove i motivišem i opravdam uvodenje -
novih ideja u tom procesu. Naravno da su gotova, ”sintetička” rešenja mnogo kon-
ciznija, ponekad i efektnija. Medutim,- moj prvi cilj je bio da knjiga bude ”user-
friendly”, a zakonitosti metodike matematike nalažu sasvim druge prioritete od
konciznosti i efektnosti.
Naime, želeo sam da pokažem da se do rešenja matematičkog problema ne
dolazi gledanjem u kristalnu kuglu (u kojoj iznenada primećujemo egzotične i
nebulozne matematičke relacije), već da ono nastaje kao rezultat razmišljanja i
logičkog rezonovanja, asocijativnih veza koje se bude i uspostavljaju posmatra-
njem sastavnih elemenata problema. Matematika je posmatračka disciplina, rekao
je Gaus (a možemo dodati i: detektivska disciplina), pa je suština rešavanja prob-
lema u njihovom ”seciranju”, razlaganju na osnovne motive i ideje. Rešenje za-
datka mora da, pre svega, zvuči prirodno, tako da deluje potpuno razumno i oprav-
dano da nam u odredenom- trenutku padne na pamet odredena - nova ideja. Sjajno
je kada čitalac, proučivši rešenje, zaključi: ”I ja sam mogao da se setim ovoga”.
To ne samo da podstiče samopouzdanje, neophodno svakom matematičaru, već i
otkriva pravu prirodu matematičkog stila mišljenja. Neki od najboljih primera koji
ilustruju ovakav pristup jesu rešenja zadataka br. 5, 8, 14, 17, 51 i 77.
Neka od rešenja snabdevena su i dodatnim komentarima. Osim onih tehničkog
karaktera, ti komentari su uglavnom vezani za moguće generalizacije i produbljenja
datih problema, ili pak bliže objašnjavaju odredene- ideje i detalje.
-
Ova knjižica namenjena je kako dacima srednjoškolskog uzrasta koji se pripre-
maju za matematička takmičenja, tako i svima onima koji vole matematiku i uživaju
u njoj, svim njenim ljubiteljima. Takode, - ona može biti korisna i svim studentima
koji izučavaju teoriju brojeva u okviru studija matematike. Na fakultetu na kome
predajem (PMF u Novom Sadu), to su studenti teorijske i primenjene matema-
tike koji počev od II godine mogu slušati predmet Teorija brojeva (na teorijskoj
matematici je on čak i obavezan). Takode, - to su i studenti master programa smera
profesor matematike koji izučavaju teoriju brojeva u okviru predmeta Elementarna
matematika I. S obzirom na tako heterogen sastav potencijalnih čitalaca, nije na

6
odmet napomenuti da ćemo u ovoj zbirci — za razliku od prakse u matematičkoj
logici — pod skupom prirodnih brojeva podrazumevati N = {1, 2, 3, . . .}.
Na kraju, želim da izrazim ogromnu zahvalnost koju dugujem svom velikom
prijatelju, Žoltu Gajdošu (Gajdos Zsolt), čiji se doprinos u oblikovanju ove zbirke
praktično može meriti sa koautorstvom. Medutim,- on je moju ponudu da se i nje-
-
govo ime nade na koricama džentlmenski odbio, sa lakonskim i savršeno logičnim
obrazloženjem: ”Ipak su to tvoji omiljeni zadaci.” Kako god bilo, veliko mu hvala,
sa bitnom napomenom da sve kritike na račun knjige pripadaju samo meni. Takode, -
zahvaljujem se i recenzentima: prof. dr Zoranu Kadelburgu i prof. dr Siniši Cr-
venkoviću, na izuzetno korisnim sugestijama i pruženoj podršci.

N OVI S AD , 20.9.2006.

7
Zadaci

1. Za koje prirodne brojeve n je broj

32n+1 − 22n+1 − 6n

složen?

2. (a) Pokazati da postoji n ∈ N tako da

21990 | (1989n − 1).

Naći najmanje takvo n.


(b) Neka je m > 3 neparan prirodan broj. Odrediti najmanje n za koje

21989 | (mn − 1).

3. Naći sve trojke prirodnih brojeva (a, b, c) takve da proizvod svaka dva broja daje
ostatak 1 pri deljenju sa trećim.

4. Naći sve prirodne brojeve a, b, c, 1 < a < b < c, takve da

(a − 1)(b − 1)(c − 1) | (abc − 1).

5. Neka su a, b, c prirodni brojevi, (a, b) = (b, c) = (c, a) = 1. Naći najveći ceo


broj koji se ne može predstaviti u obliku xbc + yca + zab, gde su x, y, z > 0 celi
brojevi.

6. Neka ogrlica A ima 14 bisera, a ogrlica B 19 bisera. Dokazati da za svaki


neparan prirodan broj n postoji način da numerišemo sve bisere brojevima iz skupa

{n, n + 1, . . . , n + 32},

tako da je svaki broj korišćen tačno jednom, i da su brojevi koji odgovaraju sused-
nim biserima uzajamno prosti.

9
7. Dokazati: postoje celi brojevi a1 , . . . , am i b1 , . . . , bk takvi da brojevi ai bj , 1 6
i 6 m, 1 6 j 6 k, čine potpun sistem ostataka po modulu mk ako i samo ako važi
(m, k) = 1.

8. Ekscentrični matematičar se kreće po lestvama sa n prečki, tako što prelazi


odjednom a prečki kada se kreće nagore, a b prečki kada se spušta nadole. Nizom
koraka naviše i naniže, on se sa zemlje penje na vrh lestvi i zatim silazi ponovo na
zemlju. Naći najmanje n za koje je ovo moguće.

9. Izračunati (f1960 , f1988 ), gde je fn niz Fibonačijevih brojeva (definisan sa f1 =


f2 = 1 i fn+2 = fn+1 + fn za n > 1).

10. Za dato c ∈ N, neka je n0 najmanji prirodan broj takav da je 2n0 > c. Dokazati
da tada za sve n > n0 važi
an | cbn ⇒ a | b,
gde su a, b ∈ N.

11. Dokazati da proizvod pet uzastopnih prirodnih brojeva ne može biti potpun
kvadrat.

12. Postoji li 23-cifren prirodan broj takav da zamenom proizvoljne cifre nikada ne
dobijamo broj deljiv sa 11?

13. Naći sve prirodne brojeve n za koje su svi brojevi koji imaju u dekadnom
prikazu n − 1 cifru 1 i jednu cifru 7 prosti.

14. Prirodan broj je dupli ako se njegov dekadni zapis sastoji od dva identična bloka
cifara. Dokazati da medu - duplim brojevima postoji beskonačno mnogo potpunih
kvadrata.

15. Neka je σ(n) zbir svih pozitivnih delitelja prirodnog broja n. Prirodan broj m
zovemo jakim ako za sve 1 6 k < m važi

σ(k) σ(m)
< .
k m
Dokazati da postoji beskonačno mnogo jakih brojeva.

16. Broj n je dobar ako se može predstaviti kao zbir (ne obavezno različitih) prirod-
nih brojeva čiji je zbir recipročnih vrednosti jednak 1. Ako je poznato da su brojevi
33, . . . , 73 dobri, dokazati da su svi brojevi > 33 dobri.

10
17. Označimo sa S(n) zbir cifara prirodnog broja n u dekadnom zapisu. Broj m
zovemo lošim ako se ne može predstaviti u obliku

m = n + S(n).

Koliko ima loših brojeva — konačno ili beskonačno mnogo?

18. Neka je S(n) zbir cifara u dekadnom zapisu prirodnog broja n. Naći sve
prirodne brojeve M takve da važi

S(kM ) = S(M )

za svako prirodno k 6 M .

19. Neka je f1 (x) kvadrat zbira cifara prirodnog broja x, a

fn (x) = f1 (fn−1 (x)).

Izračunati f1991 (21990 ).

20. Neka su a, b, c, d, a < b < c < d, neparni prirodni brojevi za koje važi:

(1) ad = bc,

(2) a + d = 2k , b + c = 2m , za neke prirodne brojeve k, m.

Dokazati da je a = 1.

21. Broj 9 se može predstaviti kao zbir dva uzastopna prirodna broja 9 = 4 + 5;
štaviše, on se može zapisati kao suma bar dva uzastopna prirodna broja na tačno
dva načina:
9 = 4 + 5 = 2 + 3 + 4.
Da li postoji prirodan broj koji se može predstaviti kao zbir 1990 uzastopnih prirod-
nih brojeva i koji se može zapisati kao zbir bar dva uzastopna prirodna broja na
tačno 1990 načina?

22. (a) Za koje prirodne brojeve n > 3 postoji skup od n uzastopnih prirodnih bro-
jeva takav da je najveći element tog skupa delitelj najmanjeg zajedničkog sadržaoca
preostalih brojeva?
(b) Za koje prirodne brojeve n > 3 postoji jedinstven skup sa gornjim svojstvom?

11
23. Neka su a1 , . . . , ak , b1 , . . . , bk prirodni brojevi takvi da je (ai , bi ) = 1 za sve
i, 1 6 i 6 k. Neka je m najmanji zajednički sadržalac brojeva b1 , . . . , bk , a
ci = ai m
bi , 1 6 i 6 k. Dokazati:

(a1 , . . . , ak ) = (c1 , . . . , ck ).

24. Neka je p prost broj, a n prirodan broj. Dokazati da postoji najviše jedan
pozitivan broj d takav da važi d | pn2 i da je n2 + d potpun kvadrat.

25. Neka je k dati prirodan broj. Dokazati da postoji beskonačno mnogo potpunih
kvadrata oblika 2k n − 7.

26. Dokazati da postoji prirodan broj k takav da je broj 2n k+1 složen za sve n ∈ N.

27. Neka su m i n prirodni brojevi sa osobinom da za sve prirodne brojeve k važi


(11k − 1, m) = (11k − 1, n). Dokazati da tada za neki ceo broj s važi m s
n = 11 .

28. Naći sve aritmetičke progresije u kojima su za sve n ∈ N zbirovi prvih n


članova potpuni kvadrati.

29. Neka je n > 6 i neka su a1 < a2 < . . . < ak svi prirodni brojevi manji od n i
uzajamno prosti sa n. Dokazati: ako je niz ai aritmetička progresija, tada je n prost
broj ili stepen dvojke.

30. Neka je n ∈ N. Dokazati da postoje različiti brojevi a, b, c ∈ N tako da je



n2 < a, b, c < n2 + n + 3 n
i a | bc.

31. Za prirodne brojeve a, b, c, d važi ad = bc i a < b < c < d. Dokazati da postoji


prirodan broj n tako da je a < n2 < d.

32. Naći sve prirodne brojeve n < 1978 sa osobinom: ako je prirodan broj m,
1 < m < n, uzajamno prost sa n, tada je on prost.

33. Predstaviti broj 51985 − 1 kao proizvod tri prirodna broja od kojih je svaki veći
od 5100 .

34. Neka je p(x) polinom sa celim koeficijentima takav da je p(0) = p(1) = 1.


Neka je a1 proizvoljan ceo broj. Definišemo niz a1 , a2 , . . . , an , . . . tako da za sve
n > 1 važi
an+1 = p(an ).
Dokazati da je svaki par različitih elemenata ovog niza uzajamno prost.

12
35. Neka je n > 1. Dokazati da sledeći polinom nema racionalne nule:
n
X xk
pn (x) = .
k!
k=0

x2 +k x + p, gde je p ∈ N. Dokazati: ako je f (x) prost broj za


36. Neka je f (x) =jq
p
sve x ∈ {0, 1, . . . , 3 }, tada je f (x) prost broj za sve x ∈ {0, 1, . . . , p − 2}.

37. Dat je polinom

f (x) = x8 + 4x6 + 2x4 + 28x2 + 1.

Neka je p > 3 prost broj takav da postoji z ∈ N za koji p | f (z). Dokazati da


postoje celi brojevi z1 , . . . , z8 takvi da za g(x) = (x − z1 ) . . . (x − z8 ) važi da su
svi koeficijenti polinoma f (x) − g(x) deljivi sa p.

38. Neka je k > 2 proizvoljan prirodan broj. Dokazati da postoji iracionalan broj
rk takav da za svaki prirodan broj n važi:

brkn c ≡ −1(mod k).

39. Dokazati: ako


¥ prirodan
√ ¦ A nije potpun kvadrat, tada postoji prirodan broj n
broj
1
tako da je A = n + n + 2 .

40. Dokazati da niz brojeva bn 2c, n ∈ N, sadrži beskonačno mnogo potpunih
kvadrata.

41. Dokazati da niz brojeva bn 2c, n ∈ N, sadrži beskonačno mnogo stepena
dvojke.

42. Neka je n ceo broj. Dokazati: ako je broj


p
2 + 2 28n2 + 1

prirodan, onda je on potpun kvadrat.

43. Za koje prirodne brojeve n postoji prirodan broj m tako da ni jedan od brojeva
m + 1, m + 2, . . . , m + n nije stepen prostog broja?

44. Naći najveći prirodan broj k za koji


¯³ ´
¯ 1992 1990
1991k ¯ 19901991 + 19921991 .

13
¯¡ ¢
45. (a) Dokazati da (n + 1) ¯ 2n
n za sve n ∈ N.
(b) Za sve k ∈ N, naći najmanji prirodan broj Ck takav da
¯ µ ¶
¯ 2n
¯
(n + k + 1) ¯ Ck
n+k
za sve n > k.

46. Neka je p > 3 prost broj. Pokazati da je broj


µ ¶ µ ¶ µ ¶
p p p
+ + ... + j k
1 2 2p
3

deljiv sa p2 .
-
47. Neka je p prost broj, a n prirodan broj. Dokazati da su sledeća dva tvrdenja
ekvivalentna:
¡ ¢
(i) Nijedan od binomnih koeficijenata nk , 1 6 k < n, nije deljiv sa p.

(ii) n = ps q − 1 za neke cele brojeve s, q takve da je s > 0 i 0 < q < p.

48. Naći sve prirodne brojeve m tako da važi:


µ ¶
m(m + 1)
1! 3! 5! . . . (2m − 1)! = !.
2

49. Neka f (m) označava najveći ceo broj k za koji 2k | m!. Dokazati da za sve
prirodne brojeve n postoji beskonačno mnogo prirodnih brojeva m tako da je

m − f (m) = n.

50. Neka su a i b celi, a n prirodan broj. Dokazati:


¯Ã !
¯ n−1
¯ n−1 Y
n! ¯ b (a + kb) .
¯
k=0

51. Neka je an poslednja nenula cifra u dekadnom prikazu broja n!. Da li niz
a1 , a2 , . . . može biti, počev od nekog člana, periodičan?

52. Posmatrajmo sve parove prirodnih brojeva (m, n), m < n, sa osobinom da se
poslednje tri cifre u dekadnom zapisu brojeva 1978m i 1978n poklapaju. Naći sve
takve parove (m, n) za koje je m + n minimalno.

14
53. Dokazati da za sve prirodne brojeve m postoji prirodan broj n > m takav da se
dekadni zapis broja 5n dobija dopisivanjem izvesnog broja cifara sleva dekadnom
zapisu broja 5m .

54. Dokazati da postoji beskonačno mnogo prirodnih brojeva n takvih da se u


dekadnom zapisu broja 5n pojavljuje 1976 uzastopnih nula.

55. Dokazati da ¯ ³ nn ´
¯ n
1989 ¯ nn − nn
za sve prirodne brojeve n > 3.

56. (a) Dokazati da postoji beskonačno mnogo parova prironih brojeva (m, n)
takvih da je 4mn − m − n + 1 potpun kvadrat.
(b) Dokazati da ne postoji nijedan par prirodnih brojeva (m, n) takav da je 4mn −
m − n potpun kvadrat.

57. (a) Dokazati da za svaki prirodan broj a > 3 postoji beskonačno mnogo prirod-
nih brojeva n za koje n | (an − 1).
(b) Naći sve prirodne brojeve n za koje n | (2n − 1).
(c) Neka je k > 2 i neka su n1 , n2 , . . . , nk prirodni brojevi takvi da

ni+1 |(2ni − 1)

za sve 1 6 i 6 k − 1, kao i n1 |(2nk − 1). Dokazati da je n1 = . . . = nk = 1.

58. Dokazati da je za svaki prirodan broj n ∈ N, niz


2 22
2, 22 , 22 , 22 , . . . (mod n)

konstantan počev od nekog člana.

59. Naći sve prirodne brojeve n za koje n2 | (2n + 1).

60. Naći sve prirodne brojeve n za koje je

d21 + d22 + d23 + d24 = n,

gde su 1 = d1 < d2 < . . . < dk = n (k > 4) svi pozitivni delioci broja n.

61. Rešiti u skupu celih brojeva:

x5 − x3 − x2 + 1 = y 2 .

15
62. Da li jednačina
x2 + xy + y 2 = 2
ima racionalna rešenja?

63. Naći sva celobrojna rešenja jednačine

a2 + b2 + c2 = a2 b2 .

64. Naći sva celobrojna rešenja jednačine

x3 − y 3 = 2xy + 8.

65. Naći sva celobrojna rešenja jednačine

x3 + x2 y + xy 2 + y 3 = 8(x2 + xy + y 2 + 1).

66. Dokazati da jednačina


x2 + 5 = y 3
nema rešenja u skupu celih brojeva.

67. Neka je n > 2 prirodan broj. Dokazati da jednačina

xn + 1 = y n+1

nema rešenje x, y ∈ N za koje važi (x, n + 1) 6= 1.

68. Naći sve prirodne brojeve x, y za koje važi:

7x − 3 · 2y = 1.

69. Naći sve prirodne brojeve x, y, z za koje važi:

3x + 4y = 5z .

70. (a) Prirodni brojevi x, y su takvi da je broj

x2 + y 2
x+y
ceo i deli 1978. Dokazati da je x = y.
(b) Dokazati da na kružnici opisanoj oko kvadrata sa temenima (0, 0), (1978, 0),
(1978, 1978), (0, 1978), nema celobrojnih tačaka, sem navedenih.

16
71. (a) Neka za prirodne brojeve x, y, z važi xy − z 2 = 1. Dokazati da postoje
nenegativni celi brojevi a, b, c, d tako da je

x = a2 + b2 , y = c2 + d2 , z = ac + bd.

(b) Dokazati: ako je p prost broj i p ≡ 1(mod 4), tada se p može predstaviti kao
zbir dva kvadrata prirodnih brojeva.

72. Neka su a, b prirodni brojevi, i neka se pri deljenju a2 + b2 sa a + b dobija


količnik q i ostatak r. Naći sve parove (a, b) za koje je q 2 + r = 1977.

73. Dokazati: ako su a i b prirodni brojevi i a2 + b2 − a je deljivo sa 2ab, tada je a


potpun kvadrat.

74. Naći sva rešenja jednačine

x2 − 5xy + y 2 + 5 = 0

u skupu prirodnih brojeva.

75. Dokazati: ako je za neke prirodne brojeve a, b broj

a2 + b2
ab + 1
ceo, tada je on potpun kvadrat.

76. (a) Neka je n ∈ N. Ako jednačina

x3 − 3xy 2 + y 3 = n

ima rešenje u skupu prirodnih brojeva, tada ona ima bar tri različita rešenja u skupu
prirodnih brojeva. Dokazati.
(b) Dokazati da gornja jednačina za n = 2891 nema rešenja u skupu prirodnih
brojeva.

77. Neka su a, b, c celi brojevi različiti od 0. Poznato je da jednačina

ax2 + by 2 + cz 2 = 0

ima celobrojno rešenje različito od x = y = z = 0. Dokazati da jednačina

ax2 + by 2 + cz 2 = 1

ima racionalno rešenje.

17
78. Neka su a, b celi brojevi koji nisu potpuni kvadrati. Dokazati: ako jednačina

x2 − ay 2 − bz 2 + abw2 = 0

ima netrivijalno celobrojno rešenje, tada to važi i za jednačinu

x2 − ay 2 − bz 2 = 0.

79. Naći celobrojno rešenje jednačine

x21 + x22 + . . . + x229 = 29x1 x2 . . . x29

tako da za bar jedno 1 6 k 6 29 važi xk > 19882 .

80. Rešiti jednačinu u skupu prirodnih brojeva:

x2n+1 − y 2n+1 = xyz + 22n+1 ,

pri čemu važe ograničenja n > 2 i z 6 5 · 22n .

81. Naći sve prirodne brojeve x, y za koje važi

x + y 2 + z 3 = xyz,

gde je z = (x, y).

82. Neka je n prirodan broj, A skup koji se sastoji od tačno n + 1 prirodnih brojeva,
a P skup svih prostih faktora elemenata skupa A. Ako je |P | 6 n, dokazati da
postoji B ⊆ A, B 6= ∅, tako da je proizvod elemenata skupa B potpun kvadrat.

83. Konstruisati funkciju f : Q+ → Q+ (gde Q+ označava skup pozitivnih racio-


nalnih brojeva) tako da važi

f (x)
f (xf (y)) =
y

za sve x, y ∈ Q+ .

18
Nekoliko poznatih teorema

Euklidov stav
Neka je x1 , . . . , xn , y ∈ Z i n > 2. Postoje celi brojevi α1 , . . . , αn takvi da je

α1 x1 + . . . + αn xn = y

ako i samo ako (x1 , . . . , xn ) | y. Specijalno, brojevi x1 , . . . , xn su uzajamno prosti


(tj. (x1 , . . . , xn ) = 1) ako i samo ako važi

α1 x1 + . . . + αn xn = 1

za neke α1 , . . . , αn ∈ Z.

Kineska teorema o ostacima


Neka su m1 , . . . , mn prirodni brojevi takvi da je (mi , mj ) = 1 za sve 1 6 i 6=
j 6 n, i neka su r1 , . . . , rn proizvoljni celi brojevi. Tada sistem kongruencija

x ≡ r1 (mod m1 ),
..
.
x ≡ rn (mod mn ),

ima rešenje. Štaviše, to rešenje je jedinstveno u skupu

{0, 1, . . . , M − 1},

gde je M = m1 . . . mn . Ako sa x0 označimo to rešenje, tada su sva preostala


rešenja oblika x = x0 + kM , k ∈ Z.

Osnovna teorema aritmetike


Svaki prirodan broj n > 2 se može zapisati u obliku

n = pα1 1 . . . pαk k ,

19
gde su α1 , . . . , αk > 1 prirodni brojevi, a p1 , . . . , pk različiti prosti brojevi. Pri
tome je gornji prikaz jedinstven do na permutaciju prostih faktora.

Bertranov stav
Za svaki prirodan broj n > 2 postoji prost broj p takav da je n < p < 2n.
Stoga, ako su pi i pi+1 dva uzastopna prosta broja, važi

pi+1 < 2pi .

Ležandrova teorema
Najviši stepen kojim prost broj p deli n! je jednak
¹ º ¹ º ¹ º
n n n
+ 2 + 3 + ...
p p p

Ojlerova i mala Fermaova teorema


Neka je n > 2 prirodan broj i neka ϕ(n) označava broj prirodnih brojeva 6 n i
uzajamno prostih sa n (Ojlerova funkcija). Tada za sve a ∈ N takve da (a, n) = 1
važi
aϕ(n) ≡ 1(mod n).
Specijalno, ako je n prost broj, tada je ϕ(n) = n − 1, pa važi: ako n - a, tada je

an−1 ≡ 1(mod n).

Vilsonova teorema
Neka je p prost broj. Tada je

(p − 1)! ≡ −1(mod p).

20
Rešenja

1. S.S.S.R., 1990.

Za koje prirodne brojeve n je broj

32n+1 − 22n+1 − 6n

složen?

Rešenje. Dati izraz možemo zapisati u obliku

3 · (3n )2 − 2 · (2n )2 − 3n · 2n .

Nakon smene x = 3n , y = 2n , dobijamo 3x2 − 2y 2 − xy, što se može rastaviti na


sledeći način:

3x2 − 2y 2 − xy = 3x2 − 3xy + 2xy − 2y 2 = (x − y)(3x + 2y).

Uvrštavajući vrednosti za x, y, imamo

32n+1 − 22n+1 − 6n = (3n − 2n )(3n+1 + 2n+1 ).

Kako za n > 2 važi 3n − 2n > 1, to je dati broj složen za sve prirodne brojeve
n 6= 1. Za n = 1, on je jednak prostom broju 13.

Komentar. Potpuno analognim postupkom, moguće je pokazati da je za a > 2 i


n > 2 broj (a + 1)2n+1 − a2n+1 − (a(a + 1))n složen.

21
2. -
(b): Predlog za MMO, 1989. (Rumunija); (a): Madarska, 1990.

(a) Pokazati da postoji n ∈ N tako da

21990 | (1989n − 1).

Naći najmanje takvo n.


(b) Neka je m > 3 neparan prirodan broj. Odrediti najmanje n za koje

21989 | (mn − 1).

Rešenje. Rešićemo zadatak koji je opštiji od (b): odredićemo minimalno n za koje


2k | (mn − 1), gde je k dati prirodan broj.
Napišimo n = 2t q, gde je q neparan broj. Imamo faktorizaciju
t t
h t t
i
mn − 1 = (m2 )q − 1 = (m2 − 1) (m2 )q−1 + . . . + m2 + 1 .

Broj u uglastoj zagradi je neparan (pošto je reč o zbiru q neparnih brojeva), pa


t
2k | (mn − 1) ako i samo ako 2k | (m2 − 1). Otuda sledi da je q = 1 za traženo
minimalno n.
S druge strane, važi
t t−1
m2 − 1 = (m2 − 1)(m2 + 1) . . . (m2 + 1).

Budući da je ovde m neparan broj, m2 daje ostatak 1 pri deljenju sa 4, a isto važi
r
i za broj oblika m2 , r > 1. Zbog toga su u gornjem proizvodu sa desne strane
svi činioci sem prvog deljivi sa 2, ali ne i sa 4, što znači da je najviši stepen kojim
t−1
dvojka deli (m2 + 1) . . . (m2 + 1) jednak t − 1. Prema tome, prostaje da se
razmotri stepen dvojke u faktoru m2 − 1 = (m − 1)(m + 1).
Kako m može davati ostatak 1 ili 3 (mod 4), posebno razmatramo ova dva
slučaja. Ako je m ≡ 1(mod 4), uočimo najveći broj s > 2 sa osobinom da
2s | (m − 1). Tada je m + 1 deljiv sa 2, ali ne i sa 4, pa je najviši stepen kojim
2 deli m2 − 1 jednak s + 1. S druge strane, ako je m ≡ 3(mod 4), posmatramo
najveći broj s > 2 za koji važi 2s | (m + 1). Slično kao i malopre, sledi da je broj
m2 − 1 deljiv sa 2s+1 , ali ne i sa 2s+2 .
Dakle, ako je broj s odreden - kao što je to opisano u prethodnom pasusu (a
t
on zavisi isključivo od m), tada je najviši stepen kojim 2 deli m2 − 1 jednak
(t − 1) + (s + 1) = t + s. Stoga je traženo minimalno rešenje n = 2k−s u slučaju
s 6 k, u suprotnom je u pitanju n = 1.
U zadatku (a) je m = 1989 = 4 · 494 + 3, pa je u tom slučaju s = 2, dok
je k = 1990, što znači da je rešenje zadatka n = 21988 . Za zadatak pod (b) treba
primeniti gornje rešenje za k = 1989.

22
3. SR Nemačka, 1990.

Naći sve trojke prirodnih brojeva (a, b, c) takve da proizvod svaka dva broja daje
ostatak 1 pri deljenju sa trećim.

Rešenje. Uslov zadatka možemo formulisati na sledeći način: postoje celi brojevi
α, β, γ tako da je

ab − 1 = γc, bc − 1 = αa, ca − 1 = βb,

za posmatrane prirodne brojeve a, b, c. Primetimo da pri tome a, b, c moraju biti


različiti od 1. Množenjem gornjih jednakosti, dobijamo:

αβγabc = (abc)2 − abc(a + b + c) + ab + bc + ca − 1.

Prebacujući na jednu stranu sve članove koji sadrže abc, zaključujemo da postoji
prirodan broj µ tako da je

µabc = ab + bc + ca − 1. (1)

Dalje, razlikujemo dva slučaja: ako su neka dva broja jednaka, npr. a = b, tada važi
αa = ac − 1. Sledi a = 1, što je nemoguće. Znači, moguć je samo drugi slučaj,
kada su brojevi a, b, c različiti. Neka je, na primer, a < b < c. Tada imamo, na
osnovu gornje jednakosti i upravo usvojenog poretka:

abc < ab + bc + ca < 3bc,

pa je a < 3, tj. a = 2. Uvrštavajući u (1), imamo:

(2µ − 1)bc = 2(b + c) − 1,

pa zaključujemo:
bc < 2(b + c) < 4c,
tj. b < 4. Kako je b > a = 2, mora biti b = 3. Otuda sledi

γc = 5,

pa je c = 5. Sada se sva rešenja zadatka dobijaju kao permutacije trojke (2, 3, 5).

23
4. MMO, 1992. (Novi Zeland)

Naći sve prirodne brojeve a, b, c, 1 < a < b < c, takve da

(a − 1)(b − 1)(c − 1) | (abc − 1).

Rešenje. Najpre, uočimo:

(a − 1)(b − 1)(c − 1) = abc − (ab + bc + ca) + a + b + c − 1 < abc − 1,

pa sledi da mora biti

abc − 1 > 2(a − 1)(b − 1)(c − 1). (2)


-
Iz gornje nejednakosti se nakon množenja i sredivanja dobija:

abc 6 2(ab + bc + ca) − 2(a + b + c) + 1 < 6bc, (3)

pa je a 6 5. Razmotrićemo posebno svaki od slučajeva a ∈ {2, 3, 4, 5}.


Dalju analizu će na ovom mestu znatno ubrzati primedba da a, b, c moraju biti
iste parnosti. Naime, ako je bar jedan od tih brojeva paran, tada je abc − 1 neparan
broj, pa on nema parnih delilaca; zato u tom slučaju sva tri broja a, b, c moraju biti
parna.
Ako je a = 2, tada iz (2) sledi

2bc − 1 > 2(b − 1)(c − 1)

(stroga nejednakost važi zbog različite parnosti leve i desne strane), pa imamo

2bc − 1 > 3(b − 1)(c − 1),


-
odakle je bc 6 3b + 3c − 4 < 6c, b 6 5. Po malopredašnjoj primedbi, sledi da je
b = 4, pa se ispitivani uslov svodi na

3(c − 1) | (8c − 1) = 8(c − 1) + 7.

Znači, c − 1 = 7, tj. c = 8. Sada neposredno proveravamo da trojka (2, 4, 8) jeste


jedno od rešenja zadatka.
-
Pre nego što predemo na preostale slučajeve, transformišimo (3) u sledeći oblik:

(a − 2)bc 6 (2a − 2)b + (2a − 2)c − (2a − 1).

Pošto je b < c, imamo

(2a − 2)b + (2a − 2)c − (2a − 1) < (4a − 4)c − (2a − 1) < 4(a − 1)c,

24
odakle je (a − 2)bc < 4(a − 1)c. Odavde, pod pretpostavkom a 6= 2, sledi
a−1
b<4 . (4)
a−2
Ovu nejednakost ćemo koristiti u analizi sva tri preostala slučaja.
Dakle, ako je a = 3, tada po (4) imamo b < 8, tj. b ∈ {5, 7}. Za b = 5, iz

8(c − 1) | (15c − 1) = 15(c − 1) + 14

zaključujemo da je c = 15 (podsetimo se, c mora biti neparno), što takode - zado-


voljava uslove zadatka, pa smo dobili rešenje (3, 5, 15). U drugom slučaju, imamo
b=7i
12(c − 1) | (21c − 1).
-
Medutim, ova relacija je nemoguća, jer −1 nije deljivo sa 3.
U slučaju a = 4 nejednakost (4) povlači 4 < b < 6, i odmah imamo kontradik-
ciju, budući da b mora biti parno.
Najzad, u slučaju a = 5 iz (4) sledi b 6 5, što je nemoguće, pošto je po
uslovima zadatka b > a = 5.
Prema tome, skup rešenja je

{(2, 4, 8), (3, 5, 15)}.

25
5. MMO, 1983. (SR Nemačka)

Neka su a, b, c prirodni brojevi, (a, b) = (b, c) = (c, a) = 1. Naći najveći ceo broj
koji se ne može predstaviti u obliku xbc + yca + zab, gde su x, y, z > 0 celi brojevi.

Rešenje. Označimo traženi broj sa M . Rešenje započinjemo razmatranjima koja će


nam omogućiti da otkrijemo njegovu vrednost.
Kao što je to dato uslovima zadatka, mi želimo da postignemo sledeće: za sve
prirodne brojeve n > M , jednačina

bcx + cay + abz = n

ima nenegativno rešenje (x, y, z), dok za n = M to nije slučaj.


Jasno, kako je (a, b) = (b, c) = (c, a) = 1, sledi da važi (ab, bc, ca) = 1, pa po
Euklidovom stavu za svaki prirodan broj n postoje celi brojevi x0 , y0 , z0 tako da je

bcx0 + cay0 + abz0 = n.

Prema tome, naš problem se svodi na to da se ustanovi kada postoji nenegativno


celo rešenje jednačine

bc(x − x0 ) + ca(y − y0 ) + ab(z − z0 ) = 0. (5)

Iz ove jednačine sledi da a | (x−x0 ) i b | (y−y0 ). Drugim rečima, važi x = x0 +as


i y = y0 + bt za neke s, t ∈ Z. S druge strane, za proizvoljne s, t ∈ Z i x, y koji
su navedenog oblika, nakon uvrštavanja u (5) dobijamo da je z = z0 − c(s + t).
Prema tome,
{(x0 + as, y0 + bt, z0 − c(s + t)) : s, t ∈ Z} (6)
predstavlja skup svih celobrojnih rešenja jednačine (5). Sada je jasno da je pitanje
predstavljivosti broja n u traženom obliku ekvivalentno postojanju trojke iz gornjeg
skupa koja se sastoji od nenegativnih brojeva.
Iz (6) se vidi da vrednost z, treće komponente rešenja, raste ako s, t opadaju.
Pošto pokušavamo da u posmatranom skupu nademo - trojku nenegativnih brojeva,
uočićemo s0 , t0 ∈ Z koji su minimalni sa osobinom da je x0 +as0 > 0 i y0 +bt0 >
0. Označimo, pri tome, x1 = x0 + as0 , y1 = y0 + bt0 i z1 = z0 − c(s0 + t0 ). Kako
tada za proizvoljno nenegativno rešenje (x, y, z) jednačine (5) mora biti x > x1 i
y > y1 , sledi da je abz = n − bcx − cay 6 n − bcx1 − cay1 = abz1 , tj. 0 6 z 6 z1 ,
zaključujemo da je egzistencija nenegativnog rešenja u skupu (6) ekvivalnetna sa
z1 > 0.
Pošto je očito 0 6 x1 6 a − 1 i 0 6 y1 6 b − 1, dobijamo da važi
abz1 = n − bcx1 − cay1 > n − bc(a − 1) − ca(b − 1) =

26
= n − (2abc − bc − ca).
Kako je željena nejednakost z1 > 0 ekvivalentna sa z1 > −1, tj. sa abz1 > −ab,
zaključujemo da se svaki broj n koji zadovoljava n − (2abc − bc − ca) > −ab,
odnosno n > 2abc − ab − bc − ca, može predstaviti na traženi način.
Zbog toga, zadatak će biti rešen ukoliko pokažemo da 2abc − ab − bc − ca nije
moguće prikazati u obliku bcx + cay + abz za neke x, y, z > 0. U suprotnom,
-
nakon sredivanja bismo imali

bc(x + 1) + ca(y + 1) + ab(z + 1) = 2abc,

pa bi zbog a | (x + 1) i x > 0 sledilo x + 1 > a, a slično i y + 1 > b i z + 1 > c.


Odatle bismo dobili

2abc > cba + cab + abc = 3abc,

što je kontradikcija. Znači, traženi broj je M = 2abc − ab − bc − ca.

Komentar 1. Izvorna formulacija zadatka je bila da se dokaže da je 2abc − ab −


bc − ca najveći ceo broj koji se ne može prikazati u obliku bcx + cay + abz za
neke cele x, y, z > 0. Izmenom formulacije se dobija teži, ali i mnogo zanimljiviji
zadatak.

Komentar 2. Može se pokazati da važi uopštenje tvrdenja - zadatka: naime, ako su


a1 , a2 , . . . , an prirodni brojevi takvi da je (ai , aj ) = 1 za sve 1 6 i 6= j 6 n, tada
je à !
X n
1
a1 a2 . . . an n − 1 −
ai
i=1

najveći ceo broj koji se ne može prikazati u obliku

x1 a2 . . . an + a1 x2 . . . an + . . . + a1 . . . an−1 xn ,

gde su x1 , x2 , . . . , xn nenegativni celi brojevi.

27
6. S.A.D., 1990.

Neka ogrlica A ima 14 bisera, a ogrlica B 19 bisera. Dokazati da za svaki neparan


prirodan broj n postoji način da numerišemo sve bisere brojevima iz skupa

{n, n + 1, . . . , n + 32},

tako da je svaki broj korišćen tačno jednom, i da su brojevi koji odgovaraju sused-
nim biserima uzajamno prosti.

Rešenje. Osnovna ideja rešenja je da što je moguće više koristimo uzastopne


prirodne brojeve za označavanje susednih bisera, pošto su oni uzajamno prosti.
Pokušaćemo da iz datog skupa na pogodan način izdvojimo 14 uzastopnih brojeva
kojima ćemo označiti bisere ogrlice A, naime,

n + m, n + m + 1, . . . , n + m + 13
-
(gde će broj m, 1 6 m 6 18 biti naknadno odreden), dok će preostali brojevi činiti
dva niza uzastopnih brojeva dužine m, odnosno 19 − m. U pitanju su

n, n + 1, . . . , n + m − 1, n + m + 14, . . . , n + 32,

i njima ćemo, ne menjujući im redosled, označiti bisere ogrlice B. Primetimo


najpre da je na ogrlici B ispunjen uslov (n, n + 32) = (n, 32) = 1, jer je n neparan
-
broj. Prema tome, uslovi iz kojih odredujemo m su

(n + m, n + m + 13) = 1, (n + m − 1, n + m + 14) = 1.

Važe sledeće jednakosti:

(n + m, n + m + 13) = (n + m, 13),

(n + m − 1, n + m + 14) = (n + m − 1, 15),
pa m tražimo iz sledeća tri uslova:
(1) m 6≡ −n(mod 13),

(2) m 6≡ 1 − n(mod 3),

(3) m 6≡ 1 − n(mod 5).


-
Medutim, od brojeva 1, 2, . . . , 18 najviše dva ne zadovoljavaju prvi uslov, tačno
šest ne zadovoljavaju drugi i najviše četiri treći uslov. Prema tome, medu - njima
postoji broj m0 koji ispunjava sva tri uslova. Ako sada stavimo m = m0 , onda
numeracija obe ogrlice ima tražene osobine.

28
7. -
Predlog za MMO, 1987. (Madarska)

Dokazati: postoje celi brojevi a1 , . . . , am i b1 , . . . , bk takvi da brojevi ai bj , 1 6


i 6 m, 1 6 j 6 k, čine potpun sistem ostataka po modulu mk ako i samo ako važi
(m, k) = 1.

Rešenje. (⇒) Neka brojevi ai bj , 1 6 i 6 m, 1 6 j 6 k, čine potpun sistem


ostataka po modulu mk. Tada je tačno jedan od njih deljiv sa mk; na primer,
neka mk | a1 b1 . Sledi da postoje brojevi a0 | a1 i b0 | b1 tako da je mk = a0 b0 .
Sada ne može biti a0 | (ai − as ) za neke indekse i 6= s, jer bi u suprotnom važilo
mk = a0 b0 | (ai b1 −as b1 ), što je u suprotnosti sa početnom pretpostavkom. Odavde
zaključujemo da mora biti a0 > m; u suprotnom bi od m brojeva ai , bar dva davala
isti ostatak po modulu a0 . Slično, b0 > k, pa zbog mk = a0 b0 sledi a0 = m,
b0 = k, što znači da brojevi ai , bj redom čine potpun sistem ostataka po modulu m,
odnosno k.
Pretpostavimo sada da je (m, k) > 1 i neka je p prost broj takav da p | (m, k).
Tada medu - brojevima ai ima m − m onih koji nisu deljivi sa p. Analogno, medu -
p
brojevima bj , k − kp nisu deljivi sa p. Znači, (m − m k
p )(k − p ) brojeva ai bj nije
-
deljivo sa p. Medutim, kako ti brojevi čine potpun sistem ostataka po modulu mk,
to medu njima ima mk − mk
-
p onih koji nisu deljivi sa p. Najzad,
µ ¶µ ¶ µ ¶
m k mk
m− k− 6= mk −
p p p
daje kontradikciju. Dakle, mora biti (m, k) = 1.
(⇐) Neka je (m, k) = 1. Naš cilj je da navedemo dva niza celih brojeva
a1 , . . . , am i b1 , . . . , bk tako da proizvodi oblika ai bj čine potpun sistem ostataka
po modulu mk.
Kao što smo to već pokazali, tada ai , 1 6 i 6 m, mora činiti potpun sistem
ostataka po modulu m, dok brojevi bj , 1 6 j 6 k, moraju davati sve različite
ostatke po modulu k. Želimo da ustanovimo u kakvom su odnosu brojevi ai i broj
k. Polazeći od pretpostavke da je k > 1, neka je q prost faktor od k. Neka medu -
brojevima ai ima tačno x onih koji nisu deljivi sa q. Analogno kao i malopre,
medu - brojevima bj ima k − k onih koji nisu deljivi sa q. S druge strane, medu -
q
proizvodima ai bj ima mk − mk q onih koji nisu deljivi sa q. Otuda je
µ ¶
k mk
x k− = mk − ,
q q
tj. x = m. Zbog toga, nijedan od brojeva ai ne može biti deljiv sa q. Pošto je prost
faktor q od k bio proizvoljno odabran, sledi da je (ai , k) = 1 za sve 1 6 i 6 m.
Analogno, (bj , m) = 1 za sve 1 6 j 6 k.

29
Prethodna razmatranja sugerišu da uočimo sledeće brojeve:
ai = ki + 1, 1 6 i 6 m,
bj = mj + 1, 1 6 j 6 k.
Očigledno, ar − as = k(r − s), što zbog (m, k) = 1 i |r − s| < m povlači da
su svi ai različiti (mod m). Slično, brojevi bj su svi različiti (mod k). Štaviše,
(ai , k) = (bj , m) = 1 za sve i, j. Ukoliko bi brojevi ai bj i ar bs davali isti ostatak
pri deljenju sa mk, tada bi bilo:

mk | (ai bj − ar bs ) = (ki + 1)(mj + 1) − (kr + 1)(ms + 1) =

= km(ij − rs) + m(j − s) + k(i − r).


Zbog (m, k) = 1, moralo bi biti k | (j − s) i m | (i − r), što znači i = r i j = s.
Prema tome, cilj je postignut, tj. navedeni brojevi imaju traženu osobinu.

30
8. Predlog za MMO, 1990. (Irska)

Ekscentrični matematičar se kreće po lestvama sa n prečki, tako što prelazi odjed-


nom a prečki kada se kreće nagore, a b prečki kada se spušta nadole. Nizom koraka
naviše i naniže, on se sa zemlje penje na vrh lestvi i zatim silazi ponovo na zemlju.
Naći najmanje n za koje je ovo moguće.

Rešenje. Bez ograničenja opštosti, možemo pretpostaviti da je a > b: u suprotnom,


zamenimo uloge brojeva a i b, pa tada svakom ”uspešnom” nizu koraka u prvobitnoj
postavci odgovara, kada taj niz koraka posmatramo unazad, uspešan niz koraka u
novoj postavci.
- opšti slučaj se može svesti na slučaj kada je (a, b) = 1. Naime, ako je
Takode,
(a, b) = d > 1, tada je jasno da matematičar može dospeti samo do onih prečki čiji
su redni brojevi deljivi sa d (gde zemlju posmatramo kao nultu prečku). Stoga je
svaki niz koraka koji matematičar može preduzeti u očiglednoj bijektivnoj kores-
pondenciji sa odgovarajućim nizom koraka u situaciji kada su umesto parametara
zadatka a, b dati redom ad i db . Zbog toga, ako sa n(a, b) označimo traženi broj, tada
je µ ¶
a b
n(a, b) = n , d.
d d
Prema tome, u daljem možemo pretpostaviti da je (a, b) = 1.
U načelu, možemo razlikovati dva tipa nizova koraka koje matematičar može da
preduzme. Prva mogućnost je da on u jednom trenutku dvaput uzastopno prelazi po
-
a prečki naviše. Medutim, da bi to bilo moguće, potrebno je da imamo na raspo-
laganju > 2a prečki. Mi ćemo pokazati da matematičar može da obavi traženi
zadatak i za n koje je manje od 2a, a to je moguće samo ukoliko posmatramo drugi
tip nizova koraka, kod kojih se ne pojavljuju dva uzastona koraka naviše. Drugim
rečima, nakon svakog koraka od a prečki naviše sledi nekoliko (bar jedan) koraka
od po b prečki nadole. Kako bismo minimizovali n, broj prečki na lestvama, intu-
itivno je opravdano da posmatramo (u izvesnom smislu ”ekstreman”) niz koraka,
definisan sledećim algoritmom.

1. Na početku, matematičar je na nivou 0.

2. Ukoliko se matematičar nalazi (posle i-te iteracije ovog algoritma) na nivou


ri < b (kada ne može da se kreće nadole), tada preduzima jedan korak naviše,
do prečke a + ri .

3. Sa prečke a+ri , matematičar silazi za po b prečki naniže onoliko puta koliko


- do prečke ri+1 < b.
je to moguće, dok ne dode

31
4. Ako je ri+1 > 0, vratimo se na korak 2. U suprotnom, algoritam je završen.

Posmatrajući korak br. 2, jasno je da je ovaj algoritam moguće sprovesti do kraja


ukoliko je n > a + b − 1.
Pokažimo sada da za n = a + b − 1 opisani algoritam upravo proizvodi niz
koraka koji zadovoljava zahteve zadatka. Najpre, induktivnim argumentom se lako
-
potvrduje da važi ri ≡ ia (mod b). Zaista, sa ri -te prečke, matematičar najpre
prelazi na prečku a + ri , a zatim nekoliko (recimo, qi ) puta silazi po b prečki
nadole, sve dok ne dode - na prečku ri+1 = a + ri − qi b < b. Odavde se odmah
vidi da ri ≡ ia (mod b) povlači ri+1 ≡ (i + 1)a (mod b). S jedne strane, dobijena
kongruencija znači da ćemo nakon b iteracija imati rb ≡ ba ≡ 0 (mod b) (tj. da će
se matematičar vratiti na zemlju). S druge strane, pošto je (a, b) = 1, postoji j < b
tako da je rj ≡ ja ≡ b−1 (mod b). Stoga u narednom koraku matematičar dospeva
na vrh lestvi, pa zaključujemo da posmatrani niz koraka ima željene osobine.
Preostaje da pokažemo da za n = a + b − c, gde je c > 2, matematičar ne može
da realizuje postavljeni zadatak. Zapravo, jedini način na koji on može da se kreće
na lestvama je upravo onaj opisan u gornjem algoritmu. Naime, na svakom nivou
on ima na raspolaganju najviše jedan korak: na nivoima 6 b − c on može samo
da ide a prečki nagore, na nivoima > b može samo da ide b prečki nadole, a na
prečkama b − c + 1, . . . , b − 1 nema na raspolaganju nijedan potez (tj. ”zaglavio”
-
se). Medutim, kako ia ≡ 0 (mod b) povlači b | i, on ne može ispuniti zadatak pre
nego što ne obavi bar b iteracija. Ali, kako je ja ≡ b − 1 (mod b) za neko j < b,
on će se sigurno zaglaviti pre toga: ili se zaglavio pre j-te iteracije, ili ako nije,
dospeva na prečku b − 1 i zaglavljuje se upravo u j-toj iteraciji.
Dakle, ako je (a, b) = 1, tada je n(a, b) = a + b − 1. To znači da je rešenje u
opštem slučaju n(a, b) = a + b − (a, b).

32
9. Predlog za MMO, 1988. (Južna Koreja)

Izračunati (f1960 , f1988 ), gde je fn niz Fibonačijevih brojeva (definisan sa f1 =


f2 = 1 i fn+2 = fn+1 + fn za n > 1).

Rešenje. Sama rekurentna relacija Fibonačijevog niza izražava zavisnost člana


fn+2 od fn i fn+1 . Ako pokušamo da izrazimo fn+i za i > 2 preko fn i fn+1 ,
dobijamo redom:

fn+3 = fn+2 + fn+1 = (fn+1 + fn ) + fn+1 = 2fn+1 + fn ,


fn+4 = fn+3 + fn+2 = (2fn+1 + fn ) + (fn+1 + fn ) = 3fn+1 + 2fn ,
fn+5 = fn+4 + fn+3 = (3fn+1 + 2fn ) + (2fn+1 + 1fn ) = 5fn+1 + 3fn ,
...

Sada se na rutinski način induktivno pokazuje da važi

fn+i = fi fn+1 + fi−1 fn (7)

za sve i > 2. Uvrštavajući i = (k − 1)n za k > 2, sledi

fkn = f(k−1)n fn+1 + f(k−1)n−1 fn ,

odakle se lako (ponovo indukcijom) dokazuje da važi

fn | fkn (8)

za sve n, k ∈ N. Najzad, neposredno se dobija da je

(fn+1 , fn ) = (fn , fn−1 ) = . . . = (f2 , f1 ) = 1 (9)

za sve n ∈ N.
Iz (7) dobijamo:
f1988 = f1961 f28 + f1960 f27 .
Znači, ako je r = (f1960 , f1988 ), tada r | f28 , pošto je na osnovu (9)

(f1961 , f1960 ) = 1.

Ali, kako je 1988 = 71 · 28 i 1960 = 70 · 28, to f28 | r (zbog (8)), pa je

r = f28 = 317811.

Komentar. Uopšte, može se pokazati da važi: (fs , ft ) = f(s,t) .

33
10. Rumunija, 1991.

Za dato c ∈ N, neka je n0 najmanji prirodan broj takav da je 2n0 > c. Dokazati


da tada za sve n > n0 važi
an | cbn ⇒ a | b,
gde su a, b ∈ N.

Rešenje. Neka je p prost broj i neka su α, β, γ redom najviši stepeni kojim p deli
brojeve a, b, c, za koje smo pretpostavili da an | cbn (n > n0 ). Dakle, imamo

nα 6 nβ + γ.

S druge strane, kako je 2n0 > c, to je γ < n0 6 n. Otuda je

nα < nβ + n = n(β + 1),

pa dobijamo α < β + 1, tj. α 6 β. Pošto je prost broj p u ovom razmatranju bio


proizvoljan, sledi a | b.

34
11. Predlog za MMO, 1984. (Velika Britanija)

Dokazati da proizvod pet uzastopnih prirodnih brojeva ne može biti potpun kvadrat.

Rešenje. Pretpostavimo suprotno: postoji pet uzastopnih prirodnih brojeva x, x+1,


x+2, x+3, x+4 čiji je proizvod potpun kvadrat. Neka je p > 5 prost broj. Najpre,
primetimo da najviše jedan od navedenih brojeva može biti deljiv sa p. Ukoliko pak
p | (x + i), 0 6 i 6 4, tada zbog naše pretpostavke najviši stepen kojim p deli x + i
mora biti paran.
-
Ovo poslednje tvrdenje povlači da svaki od pet posmatranih brojeva mora biti
oblika
2α 3β t20
za neko t0 ∈ N. Razmatrajući parnost brojeva α i β, zaključujemo da svaki od
uočenih brojeva može biti prikazan u jednom od oblika t2 , 2t2 , 3t2 , 6t2 za odgo-
varajuće t ∈ N. Po Dirihleovom principu, od tih pet brojeva postoje dva koja
se redom mogu prikazati kao au2 i av 2 za neke različite u, v ∈ N i neko fiksno
a ∈ {1, 2, 3, 6}. Ako je npr. u > v, imamo

au2 − av 2 = a(u2 − v 2 ) 6 (x + 4) − x = 4,

što znači da je u2 − v 2 6 a4 . To je moguće samo ako je a = 1, u = 2 i v = 1,


pa su 1 i 4 medu - posmatranim brojevima, tj. u pitanju su baš 1, 2, 3, 4, 5. Medutim,
-
1 · 2 · 3 · 4 · 5 = 120, što nije potpun kvadrat. Dobijena kontradikcija pokazuje da
-
je tvrdenje zadatka tačno.

35
12. Čehoslovačka, 1988.

Postoji li 23-cifren prirodan broj takav da zamenom proizvoljne cifre nikada ne


dobijamo broj deljiv sa 11?

Rešenje. Pretpostavimo da takav broj postoji; neka je to


22
X
x= ak 10k .
k=0

Budući da je 102k ≡ 1(mod 11) i 102k−1 ≡ −1(mod 11), ako je r ostatak pri
deljenju x sa 11, imamo:
11
X 11
X
r≡ a2k − a2k−1 (mod 11).
k=0 k=1

Ako je a2k > r za neko k, tada a2k možemo zameniti sa a02k = a2k − r (što je
cifra, jer je a02k 6 a2k ), a u slučaju a2k 6 r − 2 sa a02k = 11 + a2k − r (pri čemu
a02k 6 9 sledi iz a2k 6 r − 2), pa će tako dobijeni broj biti deljiv sa 11. Dakle, sve
cifre na parnim mestima broja x su jednake r − 1 (r 6= 0).
Analogno, ako je a2k−1 6 9 − r, tada zamenom cifre a2k−1 cifrom a02k−1 =
a2k−1 + r (6 (9 − r) + r = 9), odnosno za a2k−1 > 11 − r cifrom a02k−1 =
a2k−1 + r − 11 (6 a2k−1 ) dobijamo broj deljiv sa 11. Stoga su sve cifre broja x na
neparnim mestima jednake 10 − r.
-
Tada je, medutim,

r ≡ 12(r − 1) − 11(10 − r) ≡ r − 1(mod 11),

što je nemoguće. Dakle, takav broj ne postoji.

36
13. Predlog za MMO, 1990. (S.S.S.R.)

Naći sve prirodne brojeve n za koje su svi brojevi koji imaju u dekadnom prikazu
n − 1 cifru 1 i jednu cifru 7 prosti.

Rešenje. Broj N koji se sastoji od n − 1 cifara 1 i jedne cifre 7 može se zapisati u


obliku
N = An + 6 · 10k ,
gde je An broj koji je napisan sa n jedinica, a 0 6 k < n.
Ako 3 | n, tada je zbir cifara broja N deljiv sa 3, pa stoga 3 | N , zbog čega N
nije prost.
Sada posmatrajmo slučaj n > 6. Kako brojevi As redom za s = 1, 2, 3, 4, 5, 6
daju ostatke 1, 4, 6, 5, 2, 0 pri deljenju sa 7, i kako je

Am+6 = Am 106 + A6 ≡ Am + A6 ≡ Am (mod 7),

sledi da 7 | A` ako i samo ako 6 | `. Dalje, za k = 0, 1, 2, 3, 4, 5 broj 10k


daje redom ostatke 1, 3, 2, 6, 4, 5, pa 6 · 10k za iste vrednosti k redom daje ostatke
6, 4, 5, 1, 3, 2. Odavde sledi zaključak: ako za neko n > 6 važi An ≡ r(mod 7) i
3 - n, tada možemo naći takvo k 6 5 da je 6 · 10k ≡ −r(mod 7), pa onda

7 | (An + 6 · 10k ),
| {z }
N

tj. N nije prost.


Znači, dovoljno je da proverimo vrednosti n = 2, 4, 5. Za n = 5 imamo
11711 = 7 · 1673, a za n = 4 je 1711 = 29 · 59. S druge strane, za n = 2, brojevi
17 i 71 su prosti. Prema tome, rešenje zadatka je n ∈ {1, 2}.

37
14. Predlog za MMO, 1988. (Velika Britanija)

Prirodan broj je dupli ako se njegov dekadni zapis sastoji od dva identična bloka
cifara. Dokazati da medu- duplim brojevima postoji beskonačno mnogo potpunih
kvadrata.

-
Rešenje. Neka je 2k broj cifara duplog broja koji se sastoji od dva bloka a. Tvrde-
nje zadatka je sada ekvivalentno tome da jednačina

a(10k + 1) = b2 (10)

ima beskonačno mnogo rešenja (a, b, k) takvih da a ima tačno k cifara, tj.

10k−1 6 a < 10k .

Plan rešenja je sledeći: najpre ćemo za pogodne vrednosti k (kojih će biti
beskonačno mnogo) naći po jedno rešenje (a1 , b1 , k) jednačine (10) tako da a1
ima ne više od k cifara, a1 < 10k . Nakon toga, na osnovu primedbe da je tada
rešenje i (n2 a1 , nb1 , k) za sve n ∈ N, odabraćemo n0 ∈ N tako da za a = n20 a1
važe tražena ograničenja 10k−1 6 a < 10k .
Naravno, ako posmatramo gornju jednačinu (10) bez ikakvih ograničenja, a =
10k + 1 jeste jedno očigledno rešenje. Kako bismo postigli prvi od postavljenih
-
ciljeva (rešenje u kome je a < 10k ), dovoljno je da pronademo beskonačno mnogo
vrednosti k za koje je broj 10 + 1 deljiv potpunim kvadratom d2 > 1. Tada će,
k

naime,
10k + 1 10k + 1
a1 = , b1 = ,
d2 d
predstavljati rešenje razmatrane jednačine (10).
Najpre, uočavamo da je

103 + 1 = (10 + 1)(102 − 10 + 1) = 11 · 91,

tj. važi 103 = −1 + 11c (za c = 91). Sada je ideja da se na osnovu ove relacije
dobije stepen od 10 koji daje ostatak −1 pri deljenju sa 112 . To ćemo postići ako
stepenujemo poslednju jednakost sa 11, jer tada iz binomnog obrasca imamo
µ ¶ µ ¶
33 11 11 11
10 = (−1 + 11c) = −1 + 11c − 112 c2 + . . . = −1 + 112 c0
1 2

za neko c0 ∈ N. Otuda je

1033(2m−1) ≡ −1 (mod 112 )

38
za sve m ∈ N, tj. 112 | (1066m−33 + 1). To znači da je k = 66m − 33 i d =
11 pogodan izbor, pa za svaku vrednost m imamo po jedno rešenje (10) kao u
prethodnom pasusu.
- n0 ∈ N tako da važi
Najzad, preostaje da se pronade

10k + 1
10k−1 6 n20 < 10k ,
112
gde je k oblika 66m − 33. Očigledno, gornje nejednakosti su ekvivalentne sa

10k−1 ³ n ´2 10k
0
6 < .
10k + 1 11 10k + 1
Ove nejednakosti će biti ispunjene za sve k ∈ N ukoliko je
1 ³ n ´2 10
0
6 < ,
10 11 11
što važi za sve n0 ∈ {4, 5, 6, 7, 8, 9, 10}.
Rezimirajući, pokazali smo da je za sve m ∈ N dupli broj koji se sastoji iz dva
bloka
16
a= (1066m−33 + 1)
121
potpun kvadrat (ovde smo uzeli n0 = 4), pa je zadatak rešen.

39
15. Predlog za MMO, 1983. (Belgija)

Neka je σ(n) zbir svih pozitivnih delitelja prirodnog broja n. Prirodan broj m
zovemo jakim ako za sve 1 6 k < m važi

σ(k) σ(m)
< .
k m
Dokazati da postoji beskonačno mnogo jakih brojeva.

Rešenje. Obeležimo am = σ(m) m . Jasno, broj m je jak ako i samo ako je ak < am
za sve k < m. Sada je dovoljno dokazati da niz an , n ∈ N, nema najveći element,
pošto je tada lako izvesti da postoji beskonačno mnogo jakih brojeva. Naime, ako
-
je broj m jak, nadimo najmanje m0 > m tako da je am < am0 . Tada je i broj m0
očigledno jak.
Dakle, neka je n proizvoljan prirodan broj. Za svaki njegov delilac d važi da
2d | 2n. Kako, osim toga, trivijalno važi 1 | 2n, dobijamo nejednakost

σ(2n) > 2σ(n) + 1.

Odavde deljenjem sa 2n odmah sledi a2n > an , pa dobijamo traženi rezultat.

40
16. S.A.D., 1978.

Broj n je dobar ako se može predstaviti kao zbir (ne obavezno različitih) prirodnih
brojeva čiji je zbir recipročnih vrednosti jednak 1. Ako je poznato da su brojevi
33, . . . , 73 dobri, dokazati da su svi brojevi > 33 dobri.

Rešenje. Neka je n dobar broj, pri čemu je za neke prirodne brojeve ai ispunjeno:
1 1
a1 + . . . + ak = n, + ... + = 1.
a1 ak
Tada je
1 1 1
+ ... + = ,
2a1 2ak 2
1 1 1 1 1
pa zbog 2 = 4 + 4 = 3 + 6 nizovi

(4, 4, 2a1 , . . . , 2ak ) i (3, 6, 2a1 , . . . , 2ak )

imaju zbir recipročnih vrednosti jednak 1. Otuda dobijamo da su brojevi 2n + 8 i


- dobri. Medutim,
2n + 9 takode - 2 · 33 + 8 = 74 i 2 · 33 + 9 = 75, pa jednostavnom
primenom indukcije dobijamo, na osnovu date hipoteze, da su svi brojevi > 33
dobri.

41
17. Čehoslovačka, 1988.

Označimo sa S(n) zbir cifara prirodnog broja n u dekadnom zapisu. Broj m


zovemo lošim ako se ne može predstaviti u obliku

m = n + S(n).

Koliko ima loših brojeva — konačno ili beskonačno mnogo?

Rešenje. Dokazaćemo da postoji beskonačno mnogo loših brojeva. Ideja rešenja


se sastoji u tome da se konstruiše niz prirodnih brojeva an , n > 1, povezanih
pogodnom rekurentnom relacijom koja će obezbediti da svi članovi niza budu loši.
S obzirom na formulaciju zadatka, biće veoma pogodno da svi članovi niza imaju
različit broj cifara u dekadnom zapisu: tačnije, an će imati n + 1 cifru.
Najpre, neposredno se proverava da je broj 20 loš, pa definišemo a1 = 20.
-
Podimo -
od pretpostavke da smo uspeli da nademo loše brojeve a1 , . . . , an−1 u
skladu sa gornjim uslovima. Naš cilj je da konstruišemo (n + 1)-cifreni loš broj
an . Da bismo to postigli, moramo da sprečimo da jednakost an = k + S(k) bude
ispunjena za bilo koje k ∈ N. Razmotrićemo posebno slučajeve kada k ima ne više
od n cifara, odnosno kada k ima tačno n + 1 cifru.
U prvom slučaju je k 6 10n −1. Tada je S(k) 6 9n, pa sledi k +S(k) 6 10n +
9n − 1. Prema tome, ukoliko (odgovarajućom rekurentnom relacijom) obezbedimo
da važi
an > 10n + 9n − 1, (11)
tada će razmatrani slučaj biti nemoguć.
Drugi slučaj nastupa kada je k > 10n . Tada možemo pisati

k = c · 10n + α,

gde je c prva cifra od k i α 6 10n − 1. Pri tome je S(k) = S(α) + c, tj.

k + S(k) = c · 10n + α + S(α) + c.

Gornja jednakost sugeriše da se (za neko fiksno c ∈ {1, 2, . . . , 9}) za traženu reku-
rentnu vezu uzme
an = an−1 + c(10n + 1),
jer tada iz pretpostavke da je an = k + S(k) sledi da je an−1 = α + S(α), što
omogućava da se zaokruži induktivni dokaz.
Pokazaćemo da su za c = 1 ispunjeni svi traženi uslovi. Dakle, razmatramo
rekurentnu relaciju
an = an−1 + 10n + 1, (12)

42
pri čemu je a1 = 20. Indukcijom dokazujemo da su svi članovi ovako definisanog
niza loši.
Prvo uočavamo da važi nejednakost (11). Naime, očigledno imamo

ar = ar−1 + 10r + 1 > 10r ,

zbog čega je (za n > 3):

an = an−1 + 10n + 1 > 10n + 10n−1 > 10n + 9n − 1.

Za n = 1, 2 se (11) proverava direktno. Dakle, ako bi bilo an = k + S(k), tada


mora biti k > 10n (štaviše, za slučaj k = 10n se lako vidi da je nemoguć).
S druge strane, važi an < 2 · 10n , što dobijamo neposrednom primenom induk-
cije na (12). Kako iz jednakosti an = k + S(k) sledi k < an , dobijamo da je prva
cifra broja k jednaka 1. Sada iz gornjih razmatranja imamo da je

k + S(k) = α + S(α) + 10n + 1,

gde je k = 10n +α, pa pretpostavka da je an = k+S(k) povlači an−1 = α+S(α).


Pošto je poslednja jednakost po induktivnoj pretpostavci nemoguća, sledi da se ni
an ne može predstaviti u obliku k + S(k), tj. an je loš broj.

43
18. -
Madarska, 1989. (konkurs ”Kürschák József”)

Neka je S(n) zbir cifara u dekadnom zapisu prirodnog broja n. Naći sve prirodne
brojeve M takve da važi
S(kM ) = S(M )
za svako prirodno k 6 M .

Rešenje. Najpre, primetimo da je M = 1 trivijalno rešenje zadatka.


-
Podimo najpre od pretpostavke da je M broj sa n cifara u dekadnom zapisu koji
zadovoljava uslov zadatka. Neka je d njegova prva cifra; u tom slučaju, možemo
pisati
M = 10n−1 d + m,
gde je m < 10n−1 . Tada je S(M ) = d + S(m). Jasno je da (osim u slučaju n = 1)
ne može biti M = 10n−1 , pa možemo uzeti k = 10n−1 + 1, čime dobijamo:
kM = 102n−2 d + 10n−1 (m + d) + m,
odakle je
S(kM ) = S(m) + S((10n−1 d + m + d)10n−1 ) = S(m) + S(M + d).
Znači, mora biti
d = S(M + d).
Podsetimo se da je d prva cifra broja M , pa je zato M + d > 10n−1 d. Prema tome,
ako bi bilo d < 9, tada bi prva cifra broja M + d bila ili d, ili d + 1. U oba slučaja
bismo imali S(M + d) > d. Zbog toga, d = 9. Štaviše, zbir cifara broja M + 9
treba da je 9, što je moguće samo ako je M + 9 > 10n . Lako se uočava da zapravo
sve cifre broja M moraju biti jednake 9, tj. M = 10n − 1.
S druge strane, uverićemo se da svi brojevi oblika 10n −1 zadovoljavaju traženi
uslov. Zaista, ako je M = 10n − 1, tada je S(M ) = 9n i za k 6 M važi:
S(kM ) = S(k(10n − 1)) = S((k − 1)10n + (10n − k)).
Sada je (k − 1)10n > 10n > 10n − k > 1 (ukoliko je k > 1), pa je zato
S(kM ) = S(k − 1) + S(10n − k).
Možemo pisati 10n − k kao (10n − 1) − (k − 1), pa dobijamo:
S(10n − k) = 9n − S(k − 1).
Tako, kao konačan rezultat imamo:
S(kM ) = 9n,
što je i trebalo pokazati.
Dakle, rešenja zadatka su M = 1 i M = 10n − 1, n > 1.

44
19. -
Predlog za MMO, 1990. (Madarska)

Neka je f1 (x) kvadrat zbira cifara prirodnog broja x, a

fn (x) = f1 (fn−1 (x)).

Izračunati f1991 (21990 ).

Rešenje. Najpre imamo sledeće grube ocene:

21990 < 8700 < 10700 ,

pa je zato
f1 (21990 ) < (9 · 700)2 < 5 · 107 .
Otuda je, dalje,
f2 (21990 ) 6 (4 + 7 · 9)2 < 5000
i
f3 (21990 ) < (4 + 3 · 9)2 = 312 = 961.
S druge strane, zbog 26 ≡ 1(mod 9) sledi

21990 ≡ 24 ≡ 7(mod 9).

Pošto svaki broj daje isti ostatak po modulu 9 kao i njegov zbir cifara, imamo da je
f1 (x) ≡ x2 (mod 9) za sve x ∈ N, pa je zato

f1 (21990 ) ≡ 72 ≡ 4(mod 9)

i
f2 (21990 ) ≡ 42 ≡ 7(mod 9).
Najzad, imamo f3 (21990 ) = n2 , pri čemu je n < 31 i

n ≡ f2 (21990 ) ≡ 7(mod 9).

Prema tome,
f3 (21990 ) ∈ {49, 256, 625}.
-
U sva tri slučaja je, medutim, f4 (21990 ) = 169, pa imamo za n > 2:

f2n (21990 ) = 169, f2n+1 (21990 ) = 256.

Tako je konačan rezultat f1991 (21990 ) = 256.

45
20. MMO, 1984. (Poljska)

Neka su a, b, c, d, a < b < c < d, neparni prirodni brojevi za koje važi:

(1) ad = bc,

(2) a + d = 2k , b + c = 2m , za neke prirodne brojeve k, m.

Dokazati da je a = 1.

Rešenje. Neposredno iz uslova zadatka sledi:

0 < (d − b)(d − c) = d2 − bd + bc − cd = d2 − bd + ad − cd = d(a + d − b − c),

zbog čega je a + d > b + c, odnosno k > m.


Dalje, iz d = 2k − a i c = 2m − b imamo a(2k − a) = b(2m − b), tj.

(b − a)(b + a) = 2m (b − 2k−m a). (13)

Zbog k > m, b − 2k−m a je neparan broj, pa je (b − a)(b + a) deljivo sa 2m , ali ne


i sa 2m+1 . Otuda je:

b − a = 2m1 p,
b + a = 2m2 q,

pri čemu je m1 + m2 = m, m1 , m2 > 1 i p, q su neparni brojevi. Sabiranjem ove


dve jednačine sledi
2b = 2m1 p + 2m2 q.
Pošto je b neparan broj, mora biti m1 = 1 ili m2 = 1. S druge strane, zbog
b + c = 2m i b < c važi

2m−1 > b > b − a = 2m1 p > 2m1 ,

tj. m1 < m − 1 i m2 = m − m1 > 1. Tako je m1 = 1 i m2 = m − 1, pa prethodni


sistem postaje

b − a = 2p,
b + a = 2m−1 q.

Na osnovu toga je 2m = b + c > b + a = 2m−1 q, pa je q < 2, tj. q = 1.


Uvrštavajući dobijene rezultate u (13), dobijamo

p = b − 2k−m a.

46
Iz prethodnog sistema je b = p + 2m−2 , odakle je p = p + 2m−2 − 2k−m a, odnosno

2k−m a = 2m−2 .

Kako je a neparan, ovo je moguće samo ukoliko je k = 2m − 2 i a = 1, što je i


trebalo pokazati.

Komentar. Iz gornjeg rešenja se lako vidi da su sve četvorke brojeva (a, b, c, d) koje
zadovoljavaju uslove zadatka date sa

(1, 2m−1 − 1, 2m−1 + 1, 22m−2 − 1),

gde je m ∈ N.

47
21. Predlog za MMO, 1990. (Australija)

Broj 9 se može predstaviti kao zbir dva uzastopna prirodna broja 9 = 4 + 5;


štaviše, on se može zapisati kao suma bar dva uzastopna prirodna broja na tačno
dva načina:
9 = 4 + 5 = 2 + 3 + 4.
Da li postoji prirodan broj koji se može predstaviti kao zbir 1990 uzastopnih prirod-
nih brojeva i koji se može zapisati kao zbir bar dva uzastopna prirodna broja na
tačno 1990 načina?

Rešenje. Pretpostavimo da broj N ima tražena svojstva. Prvi od dva uslova zadatka
se može zapisati kao

N = m + (m + 1) + . . . + (m + 1989) = 995(2m + 1989),

za neko m, stoga je N neparan broj, deljiv sa 5 i 199. Drugi uslov je da postoji


tačno 1990 parova prirodnih brojeva (n, k) za koje je
(k + 1)(2n + k)
N = n + (n + 1) + . . . + (n + k) = .
2
Prema tome, postoji tačno 1990 načina da prikažemo 2N u obliku:

2N = (k + 1)(2n + k),

pri čemu je k > 1. Pošto je N neparan broj, to je jedan od ova dva faktora neparan,
dok je drugi deljiv sa 2, ali ne i sa 4. Kako je očito k + 1 < 2n + k, sledi da svaka
opisana faktorizacija broja 2N jednoznačno daje traženi par (n, k). Ako zapišemo:

2N = 2 · 5s1 199s2 ps33 . . . psrr ,

gde su pi prosti brojevi različiti od 2, 5 i 199, dobijamo da je broj delilaca broja 2N


jednak
(1 + 1)(s1 + 1)(s2 + 1) . . . (sr + 1).
Tako, faktorizacija 2N = uv, u < v, ima (s1 + 1)(s2 + 1) . . . (sr + 1), a pošto
trivijalna faktorizacija 2N = 1 · 2N daje k = 0, opisanih parova (n, k) ima

(s1 + 1)(s2 + 1) . . . (sr + 1) − 1.

Sada je (s1 + 1)(s2 + 1) . . . (sr + 1) = 1991 = 11 · 181. Budući da zbog 5 · 199 | N


imamo s1 , s2 > 0, zaključujemo da je s1 = 10, s2 = 180 ili s1 = 180, s2 = 10,
kao i s3 = . . . = sr = 0. To znači da je N = 510 · 199180 ili N = 5180 · 19910 , pa
smo tako dobili jedina dva broja sa traženim osobinama.

48
22. MMO, 1981. (Belgija)

(a) Za koje prirodne brojeve n > 3 postoji skup od n uzastopnih prirodnih brojeva
takav da je najveći element tog skupa delitelj najmanjeg zajedničkog sadržaoca
preostalih brojeva?
(b) Za koje prirodne brojeve n > 3 postoji jedinstven skup sa gornjim svojstvom?

Rešenje. (a) Označimo najveći od traženih brojeva sa m; tada tražimo prirodan broj
n tako da važi
m | [m − n + 1, . . . , m − 2, m − 1].
Očito, mora biti m > n. Neka je k najveći ceo broj sa osobinom pk | m, pri čemu
-
je p proizvoljan prost broj. Medutim, sada pk deli najmanji zajednički sadržalac
brojeva m − 1, m − 2, . . . , m − n + 1 ako i samo ako deli bar jedan od njih. To
znači da pk mora deliti bar jedan od brojeva 1, 2, . . . , n − 1, odnosno njihov naj-
manji zajednički sadržalac, koji ćemo označiti sa a(n). Zaključak je da je uslov,
postavljen u zadatku, ekvivalentan sa postojanjem prirodnog broja m > n takvog
da m | a(n).
Egzistencija bar jednog takvog broja m je ekvivalentna nejednakosti a(n) > n,
tj. a(n) > n − 1. Po definiciji broja a(n), (n − 1) | a(n). Pošto je n > 3, imamo
da (n − 2) | a(n). Zbog (n − 2, n − 1) = 1 sledi (n − 2)(n − 1) | a(n). Dakle,
a(n) > (n−2)(n−1), što je > (n−1) kad god je n > 4. S druge strane, a(3) = 2,
što znači da odgovor na pitanje (a) glasi: n > 4.
(b) Uslov ovog dela zadatka ekvivalentan je postojanju jedinstvenog prirodnog
broja m > n tavkog da m | a(n). Pri tome, zbog prethodnog dela zadatka možemo
pretpostaviti da je n > 4. Pošto tada m = a(n) zadovoljava tražene uslove, naš cilj
-
se svodi na odredivanje svih brojeva n > 4 za koje su svi pravi delitelji od a(n)
manji od n.
Kao što smo to malopre videli, važi a(n) = (n − 1)(n − 2)b za neko b > 1.
Kako za n > 4 imamo (n − 1)(n − 2) > n, sledi da mora biti b = 1, tj. a(n) =
-
(n − 1)(n − 2). Medutim, pošto (n − 3) | a(n) i (n − 3, n − 2) = 1, zaključujemo
da (n − 3) | (n − 1). Drugim rečima, (n − 3) | 2, pa je n ∈ {4, 5}. Pošto je
6 | 12 = a(5) i a(4) = 6, dobijamo da je odgovor na pitanje (b): n = 4.

49
23. Predlog za MMO, 1974. (Poljska)

Neka su a1 , . . . , ak , b1 , . . . , bk prirodni brojevi takvi da je (ai , bi ) = 1 za sve i, 1 6


i 6 k. Neka je m najmanji zajednički sadržalac brojeva b1 , . . . , bk , a ci = ai m bi ,
1 6 i 6 k. Dokazati:
(a1 , . . . , ak ) = (c1 , . . . , ck ).

Rešenje. Neka je p prost broj koji deli bar jedan od datih brojeva a1 , . . . , ak ,
b1 , . . . , bk . Neka su αi , βi redom najviši stepeni kojima p deli ai , odnosno bi . Tada
je najviši stepen kojim p deli m jednak µ = maxj βj , dok je najviši stepen kojim
p deli ci jednak γi = αi + µ − βi . Imajući u vidu proizvoljnost prostog faktora p,
-
tvrdenje zadatka je ekvivalentno jednakosti

min αi = min(αi + µ − βi ) = µ + min(αi − βi ). (14)


i i i

Pri tome zbog (ai , bi ) = 1 važi αi 6= 0 ⇒ βi = 0.


Radi kraćeg zapisa, definišimo δi = αi − βi . S obzirom na uočeni odnos
eksponenata αi i βi , važi
(
−βi ako αi = 0,
δi =
αi ako αi 6= 0.

Razlikujemo dva slučaja. Ako postoji indeks i0 tako da je αi0 = 0, tada je


mini αi = 0. S druge strane, mini δi = − maxi βi = −µ, pa tada i desna strana u
(14) ima vrednost 0. U suprotnom slučaju, αi > 0 za sve 1 6 i 6 k. Ali, tada je
βi = 0 za sve 1 6 i 6 k, pa je µ = 0 i δi = αi za sve 1 6 i 6 k. To znači da je
jednakost (14) tačna i u ovom slučaju, pa je njen dokaz kompletan.

50
24. -
Madarska, 1990. (konkurs ”Kürschák József”)

Neka je p prost broj, a n prirodan broj. Dokazati da postoji najviše jedan pozitivan
broj d takav da važi d | pn2 i da je n2 + d potpun kvadrat.

Rešenje. Neka je
n2 + d = m2 , (15)
i neka je z = (n, m). Tada imamo n = xz i m = yz, gde je (x, y) = 1, pa iz
(15) sledi da je d = az 2 za neko a. Pri tome, az 2 = d | pn2 = px2 z 2 , pa a | px2 .
Nakon uvrštavanja u (15) i skraćivanja sa z 2 , dobijamo

x2 + a = y 2 .

Množenjem ove jednakosti sa p, sledi

px2 + pa = py 2 .

Kako a | px2 , leva strana ove jednakosti je deljiva sa a, pa a | py 2 . Pošto je


(x, y) = 1 (⇒ (x2 , y 2 ) = 1), postoje celi brojevi α, β tako da je αx2 + βy 2 = 1.
Ali, tada
a | (αpx2 + βpy 2 ) = p(αx2 + βy 2 ) = p.
Prema tome, a = 1 ili a = p.
Prvi od dva navedena slučaja, a = 1, rezultuje jednačinom x2 + 1 = y 2 , odakle
je x = 0 i n = xz = 0, što je nemoguće. Dakle, moguć je samo drugi slučaj,
a = p. Tada važi x2 + p = y 2 , odnosno p = y 2 − x2 = (y − x)(y + x), što povlači
y − x = 1 i y + x = p, tj.
p+1 p−1
y= , x= .
2 2
Otuda je n = xz = 21 (p − 1)z.
Znači, egzistencija broja d sa traženim osobinama povlači da 21 (p − 1) | n.
S druge strane, ukoliko ova deljivost važi, tada imamo jedinstveno d za koje su
ispunjeni traženi uslovi, naime
³ n ´2 µ ¶2
2n
d = pz 2 = p =p .
x p−1
-
Time je traženo tvrdenje dokazano.

51
25. Predlog za MMO, 1995. (Rumunija)

Neka je k dati prirodan broj. Dokazati da postoji beskonačno mnogo potpunih


kvadrata oblika 2k n − 7.

Rešenje. Dokažimo najpre da za svaki prirodan broj k postoji prirodan broj ak sa


osobinom
a2k ≡ −7(mod 2k ).
Primetimo da izbor ak = 1 zadovoljava traženi uslov za k 6 3. Za k > 4,
-
podimo od pretpostavke a2k ≡ −7(mod 2k ). Sada očito imamo dve mogućnosti:

a2k ≡ −7(mod 2k+1 ),

ili
a2k ≡ 2k − 7(mod 2k+1 ).
U prvom slučaju definišimo ak+1 = ak , a u drugom ak+1 = ak + 2k−1 . Pošto je
ak neparno, u potonjem slučaju sledi:

a2k+1 = a2k + 2k ak + 22k−2 ≡ a2k + 2k ak ≡ a2k + 2k ≡ −7(mod 2k+1 ),

koristeći induktivnu pretpostavku.


Najzad, primetimo da niz ak nije ograničen, pošto mora biti a2k > 2k − 7,
što znači da posmatrani niz ima beskonačno mnogo različitih vrednosti. Otuda
dobijamo traženi rezultat, pošto za m > k imamo a2m ≡ −7(mod 2k ) i možemo
definisati
a2 + 7
n= mk .
2

52
26. S.A.D., 1982.

Dokazati da postoji prirodan broj k takav da je broj 2n k + 1 složen za sve n ∈ N.

Rešenje. Neka je n = 2r p, gde je p neparan broj. Ako je broj M takav da je


k ≡ 1(mod M ), tada se k može ”izbrisati” iz izraza 2n k + 1 (mod M ), tj. imamo
rp r r
2n k + 1 ≡ 2n + 1 = 22 + 1 = (22 )p + 1 = (22 + 1)A (mod M )

za neki prirodan broj A. Odavde očigledno sledi da ako za M odaberemo baš


r
22 + 1 (ili bilo koji njegov delitelj), tada iz datih pretpostavki sledi da je 2n k + 1
r
deljivo sa 22 + 1.
r
Poznato je da su za 0 6 r 6 4, brojevi Fr = 22 + 1 prosti. (Inače, brojevi
oblika Fr su Fermaovi, a u slučaju da je takav broj prost, on se zove Fermaov prost
broj. Malopre navedeni brojevi su ujedno i jedini do sada poznati Fermaovi prosti
-
brojevi.) Medutim, kao što je to još 1732. godine pokazao L.Ojler, F5 je složen:
najmanji prost faktor mu je 641.
Ideja rešenja se sastoji u tome da se k odredi uz pomoć kineske teoreme, tako
da zadovoljava pogodan sistem linearnih kongruencija. Taj sistem kongruencija će
biti ”podešen” tako da je za sve n broj 2n k + 1 deljiv ili nekim od F0 , . . . , F4 , ili sa
641, ili sa G = F5 /641. Razlikovanje slučajeva se vrši po najvišem stepenu dvojke
koji deli n. Naime, već smo videli da za 0 6 r 6 4 imamo da ako važi n = 2r p za
neki neparan broj p i k ≡ 1(mod Fr ), tada Fr | (2n k + 1). Takode, - ako je n = 25 p
za neki neparan broj p i k ≡ 1(mod 641), tada 641 | (2n k + 1).
Prema tome, preostaje da se razmotri slučaj kada je n = 26 q za neko q ∈ N.
Tada imamo

2n k + 1 = k(2n − 1) + (k + 1) = k((264 )q − 1) + (k + 1) =

= k(264 − 1)B + (k + 1) = kF5 (F5 − 2)B + (k + 1)


za neki prirodan broj B, pa ukoliko je k ≡ −1(mod G), dobijamo da važi G |
(2n k + 1) (pošto G | F5 ).
Rezimirajući prethodna razmatranja, tražimo prirodan broj k koji zadovoljava

k ≡ 1(mod Fr ), 0 6 r 6 4, k ≡ 1(mod 641), k ≡ −1(mod G).

Pošto su brojevi F0 , . . . , F4 , 641 prosti, a F5 nije deljivo sa 6412 (što se nepos-


redno proverava), moguća je primena kineske teoreme o ostacima. Štaviše, gornji
sistem ima proizvoljno veliko rešenje k, pa tako i rešenje koje zadovoljava k >
max(F0 , F1 , F2 , F3 , F4 , 641, G). Zbog toga je

2n k + 1 > max(F0 , F1 , F2 , F3 , F4 , 641, G)

53
za sve n ∈ N. S druge strane, pokazali smo da je broj oblika 2n k + 1 (za na opisani
način odabrano k) uvek deljiv bar jednim od F0 , F1 , F2 , F3 , F4 , 641, G. Zato je
2n k + 1 složen broj za sve n ∈ N, što je i trebalo dokazati.

54
27. Rumunija, 1978.

Neka su m i n prirodni brojevi sa osobinom da za sve prirodne brojeve k važi


(11k − 1, m) = (11k − 1, n). Dokazati da tada za neki ceo broj s važi m s
n = 11 .

Rešenje. Neka je m = 11a p, n = 11b q, pri čemu je a, b > 0 i brojevi p, q nisu


-
deljivi sa 11. Dokazaćemo da je p = q, odakle sledi tvrdenje zadatka.
Kako je (p, 11) = 1, po kineskoj teoremi o ostacima postoji prirodan broj x
koji zadovoljava:
x ≡ 0(mod p), x ≡ −1(mod 11).
Ali, tada je x = 11k − 1 za neki prirodan broj k, pa je:

p = (x, 11a p) = (11k − 1, m) = (11k − 1, n) = (x, 11b q) = (x, q) 6 q.

Potpuno analogno možemo pokazati da je q 6 p, stoga sledi zaključak p = q.

55
28. Rumunija, 1991.

Naći sve aritmetičke progresije u kojima su za sve n ∈ N zbirovi prvih n članova


potpuni kvadrati.

Rešenje. Ako je a prvi član aritmetičke progresije, a d njena razlika, tada imamo
formulu za sumu prvih n članova progresije:
1
Sn = n(2a + (n − 1)d).
2
Specijalno, imamo S1 = a i S4 = 2(2a + 3d), odakle je a = b2 i d = 2d1 za neke
prirodne brojeve b i d1 , pa sledi:

Sn = n(b2 + (n − 1)d1 ) = n(nd1 + (b2 − d1 )).

Za proizvoljan prost broj p iz p | Sp sledi p2 | Sp , što na osnovu gornje jednakosti


povlači p | (b2 − d1 ). To je moguće ako i samo ako je b2 − d1 = 0, tj. d = 2b2 .
S druge strane, ako su članovi aritmetičke progresije oblika (2n − 1)b2 , gde je b
proizvoljan prirodan broj, tada su odgovarajuće sume Sn = n2 b2 , što znači da smo
našli sve tražene progresije.

56
29. MMO, 1991. (Rumunija)

Neka je n > 6 i neka su a1 < a2 < . . . < ak svi prirodni brojevi manji od n
i uzajamno prosti sa n. Dokazati: ako je niz ai aritmetička progresija, tada je n
prost broj ili stepen dvojke.

Rešenje. Na početku uočavamo da je a1 = 1 i a2 = p, gde je p najmanji prost


broj koji ne deli n. Zato je razlika posmatrane aritmetičke progresije jednaka p − 1.
- ak = n − 1.
Takode,
Ako je n neparan broj, tada je a2 = 2, pa je uočena progresija zapravo 1, 2, . . . ,
n − 1, odakle sledi da je broj n prost.
Ukoliko je n paran, tada je p > 3, pa razmatramo dva slučaja. Ako je p = 3,
tada je reč o progresiji 1, 3, . . . , n − 1, što znači da je n uzajamno prost sa svakim
neparnim brojem manjim od n, što je moguće samo ako je n = 2m za neki prirodan
broj m. Ako je p > 3, sledi da 3 | n. Tada zbog

n − 2 = (n − 1) − 1 = ak − a1 = (p − 1)(k − 1)
-
važi (p − 1) | (n − 2), pa ako je q prost broj i q | (p − 1), tada q | (n − 2). Medutim,
q < p, pa q | n. Otuda je q | 2, tj. q = 2, kao i p − 1 = 2s za s > 2, odnosno
t
p = 2s + 1. Pošto je p prost broj, to je s = 2t , p = 22 + 1 za t > 1. Ali, sada
imamo:
t
a3 = 1 + 2(p − 1) = 2p − 1 = 22 +1 + 1,
pa 3 | a3 . Zato 3 | (a3 , n) = 1. Kontradikcija.

57
30. -
Madarska, 1991.

Neka je n ∈ N. Dokazati da postoje različiti brojevi a, b, c ∈ N tako da je



n2 < a, b, c < n2 + n + 3 n
i a | bc.

Rešenje. Pošto za tražene brojeve mora da važi a | bc, postoje x, y tako da je


a = xy, pri čemu x | b i y | c. Budući da nejednakosti date u zadatku sugerišu da
brojevi a, b, c treba da budu relativno ”bliski”, potražićemo ih u obliku:
a = xy, b = x(y + 1), c = (x + 1)y,
pri čemu je x < n < y (odakle je a < b < c). Zbog toga, pišimo x = n − k,
y = n + ` za neke prirodne brojeve k, `.
Kako bismo pronašli brojeve x, y sa željenim osobinama, potrebno je i dovoljno

da bude xy = (n−k)(n+`) > n2 i (x+1)y = (n−k +1)(n+`) < n2 +n+3 n.
-
Nakon kraćeg sredivanja, ovi uslovi postaju

(` − k)n − k` > 0, (` − k)n − k` + ` < 3 n.
Naravno, ovde će izbor k, ` zavisiti od n. Iz gornjih nejednakosti vidimo da broj
(`−k)n−k` treba da bude pozitivan, ali, s druge strane, ne previše velik. Zbog toga
je naš cilj da razlika ` − k bude što je manja moguća. Kako iz prve nejednakosti
očigledno sledi da mora biti ` − k > 0, intuitivno je opravdano da pokušamo da
postignemo da bude ` − k = 1, tj. ` = k + 1. Ako ovo uvrstimo u gornji sistem

nejednakosti, dobijamo uslove k(k+1) < n i n−k 2 +1 < 3 n, što je ekvivalentno

sa k 2 > n − 3 n + 1.
Zbog ovih uslova, pogodno će biti da za k uzmemo najveću vrednost za koju je
ispunjena nejednakost k(k + 1) < n. Drugim rečima, treba da bude
k(k + 1) < n 6 (k + 1)(k + 2).
Kao što smo već videli, nejednakost k(k + 1) < n je ekvivalentna sa a > n2 , pa

preostajemo da pokažemo da za opisani izbor k i ` imamo c < n2 + n + 3 n, tj.

k 2 > n − 3 n + 1. Zaista, ako pretpostavimo da ova nejednakost nije ispunjena,
tada je √ √
k 2 6 n − 3 n + 1 < ( n − 1)2 ,

odakle je k < n − 1. Stoga, zaključujemo:
√ √
(k + 1)(k + 2) = k 2 + 3k + 2 < (n − 3 n + 1) + 3( n − 1) + 2 = n,
što je kontradikcija sa izborom k.

58
31. -
Madarska, 1991.

Za prirodne brojeve a, b, c, d važi ad = bc i a < b < c < d. Dokazati da postoji


prirodan broj n tako da je a < n2 < d.

-
Rešenje. Pretpostavimo suprotno tvrdenju zadatka; tada postoji prirodan broj n
2 2
tako da je n 6 a i d 6 (n + 1) . Otuda mora biti d − a 6 2n + 1. S druge strane,
iz (c − b)2 > 0 i uslova zadatka sledi

(c + b)2 − (c − b)2 = 4bc = 4ad = (d + a)2 − (d − a)2 .

Zbog d − a > c − b > 0 važi (d − a)2 > (c − b)2 , odakle iz gornjih veza imamo
(d + a)2 > (c + b)2 , tj. d + a > c + b, kao i:

(d − a)2 = (d + a)2 − (c + b)2 + (c − b)2 > (d + a)2 − (c + b)2 =


= (d − c − b + a)(d + c + b + a). (16)
-
Po prethodnoj primedbi, d − c − b + a > 0, odnosno d − c − b + a > 1. Takode,
važi d + c + b + a > 4n2 , pa je

d − a > 2n.

Zaključujemo da je jedina mogućnost d − a = 2n + 1, a = n2 , d = (n + 1)2 ,


bc = n2 (n + 1)2 .
Ako je n = 1, tada je a = 1, d = 4, odakle je b = 2, c = 3 i bc 6= 4, pa je taj
slučaj isključen. S druge strane, za n > 2 važi nejednakost

(2n + 1)2 = 4n2 + 4n + 1 < 8n2 = 2 · 4n2 ,

pa uočavamo da (zbog (16)) mora biti d − c − b + a = 1, tj.


√ √
c + b = (n + 1)2 + n2 − 1 = 2n(n + 1) = 2 ad = 2 bc.
√ √
Transformacijom gornje jednakosti sledi ( c − b)2 = 0, odnosno b = c. Kon-
tradikcija.

59
32. Predlog za MMO, 1978. (Čehoslovačka)

Naći sve prirodne brojeve n < 1978 sa osobinom: ako je prirodan broj m, 1 <
m < n, uzajamno prost sa n, tada je on prost.

Rešenje. Za svaki prost broj p, p2 je složen broj (i to su, u izvesnom smislu, ”najjed-
nostavniji” složeni brojevi). Zbog toga, ako za n ∈ N sa osobinom iz formulacije
zadatka važi p2 < n, tada n i p2 ne smeju biti uzajamno prosti, tj. p | n. Drugim
rečima, ako p - n, tada je n < p2 .
Imajući u vidu ovaj zaključak, neka je q = pk+1 najmanji prost broj koji ne
deli prirodan broj n sa traženom osobinom (ovde je p1 = 2, p2 , p3 , . . . niz prostih
brojeva). Tada je n < q 2 , dok s druge strane svi brojevi p1 , . . . , pk dele n, odakle
p1 . . . pk | n i n > p1 . . . pk . Prema tome, važi nejednakost

p1 . . . pk < p2k+1 .
-
Medutim, po Bertranovom stavu važi pk+1 < 2pk < 4pk−1 (pri tome pretpostav-
ljamo da je k > 2, dok će slučajevi k ∈ {0, 1} biti razmotreni kasnije). Otuda je
p2k+1 < 8pk−1 pk , pa dobijamo

p1 . . . pk−2 < 8,

tj. k 6 4 (naime, 2 · 3 = 6 < 8, dok je 2 · 3 · 5 = 30 > 8).


Dakle, preostaje da se razmotre slučajevi k ∈ {0, 1, 2, 3, 4}.
Za k = 0 imamo q = p1 = 2, pa je n < 4 i 2 - n, odnosno n ∈ {1, 3}. Jasno,
odmah se vidi da su ovo rešenja zadatka.
Za k = 1 je q = 3, pa n zadovoljava n < 9 i 2 | n, 3 - n. Ponovo se lako
proverava da sva tri ”kandidata” 2, 4, 8 predstavljaju rešenja zadatka.
Za k = 2 važi q = 5, odakle je n < 25, 6 | n i 5 - n, tj. n ∈ {6, 12, 18, 24}.
Direktnom proverom dobijamo da sva četiri navedena broja zadovoljavaju uslove
zadatka.
Za k = 3 sledi q = 7, pa mora biti n < 49, 30 | n i 7 - n. Jedini broj
sa navedenim osobinama je n = 30, za koji se direktno proverava da zadovoljava
uslove zadatka.
Najzad, za k = 4 je q = 11, što povlači da je n < 121 i 2 · 3 · 5 · 7 = 210 | n.
Jasno, takav prirodan broj ne postoji.
Prema tome, skup rešenja zadatka je

{1, 2, 3, 4, 6, 8, 12, 18, 24, 30}.

Komentar. Primetimo da je uslov n < 1978 suvišan u zadatku. Zvanično rešenje


zadatka prezentirano žiriju MMO 1978. je nešto duže i koristi ovaj uslov.

60
33. Predlog za MMO, 1985. (S.S.S.R.)

Predstaviti broj 51985 − 1 kao proizvod tri prirodna broja od kojih je svaki veći od
5100 .

Rešenje. Najpre, primetimo da je 1985 = 5 · 397. Zbog toga, označimo x = 5397 .


Sada je razmatrani broj x5 − 1. Očigledno, imamo faktorizaciju

x5 − 1 = (x − 1)(x4 + x3 + x2 + x + 1).

Naš cilj je da izraz u drugoj zagradi predstavimo kao razliku dva kvadrata, pri
čemu treba imati na umu da je x neparan stepen od 5, zbog čega je 5x potpun
kvadrat (5x = 5398 = (5199 )2 ). Imajući to u vidu, tražićemo cele brojeve a, b, c
tako da važi

x4 + x3 + x2 + x + 1 = (x2 + ax + 1)2 − 5x(bx + c)2 . (17)

Razvijanjem izraza sa desne strane, dobijamo

x4 + (2a − 5b2 )x3 + (a2 + 2 − 10bc)x2 + (2a − 5c2 )x + 1,

odakle nije teško videti da će (17) važiti za a = 3, b = c = 1. Prema tome, sledi
£ ¤
x5 − 1 = (x − 1) (x2 + 3x + 1)2 − 5x(x + 1)2 .

Kao što je već rečeno, izraz u uglastoj zagradi je razlika kvadrata, pa se on može
faktorisati. Neposredno se proverava da su sva tri ovako dobijena faktora od x5 − 1
veća od 5100 .

61
34. Predlog za MMT, Mersch, Luksemburg, 1980. (Belgija)

Neka je p(x) polinom sa celim koeficijentima takav da je p(0) = p(1) = 1. Neka


je a1 proizvoljan ceo broj. Definišemo niz a1 , a2 , . . . , an , . . . tako da za sve n > 1
važi
an+1 = p(an ).
Dokazati da je svaki par različitih elemenata ovog niza uzajamno prost.

Rešenje. Posmatrajmo elemente am i ak datog niza, pri čemu je m > k. Ako je


d = m − k, tada imamo da je

am = ak+d = pd (ak ) = p(p(. . . (p(ak )) . . .)).


| {z }
d

- polinom sa celim koeficijentima.


Pri tome, pd (x) je takode
Kako je p(0) = 1, sledi da je slobodni član polinoma p(x) jednak 1. Medu- -
tim, p2 (0) = p(p(0)) = p(1) = 1, pa to isto važi i za polinom p2 (x). Ali, sada
se indukcijom lako dokazuje da je slobodni član polinoma pr (x) jednak 1 za sve
r > 1, budući da iz pretpostavke da je pr (0) = 1 sledi

pr+1 (0) = p(pr (0)) = p(1) = 1.

Za svaki polinom f (x) sa celim koeficijentima postoji polinom q(x) ∈ Z[x]


tako da je f (x) = xq(x) + f (0). Specijalno, postoji qd (x) ∈ Z[x] tako da je

pd (x) = xqd (x) + pd (0) = xqd (x) + 1.

Otuda dobijamo da važi

am = pd (ak ) = ak qd (ak ) + 1 = ak Q + 1,
-
gde je Q = qd (ak ) ceo broj. Iz gornje jednakosti sada neposredno sledi tvrdenje
zadatka.

62
35. Predlog za MMO, 1989. (Bugarska)

Neka je n > 1. Dokazati da sledeći polinom nema racionalne nule:


n
X xk
pn (x) = .
k!
k=0

Rešenje. Pretpostavimo da je α = qr racionalna nula datog polinoma, (q, r) = 1.


Tada važi
n! ³ q ´k
X n
= n!pn (α) = 0,
k! r
k=0

odakle množenjem sa rn sledi


n−1
X n! k n−1−k
qn + r q r = 0.
k!
k=0

Znači, r | q n , pa mora biti r = 1 (tj. α je ceo broj). Zbog toga, gornju jednakost
možemo pisati u obliku
n−1
X n!
n
q + q k = 0. (18)
k!
k=0

Kako je za 0 6 k 6 n − 1 broj n! n
k! deljiv sa n, zaključujemo da n | q . To
n
znači da za proizvoljan prost faktor p broja n imamo p | q , odakle p | q. Ideja u
preostalom delu rešenja je da dobijemo kontradikciju posmatrajući stepene kojim
p deli pojedine sabirke na levoj strani jednakosti (18).
Imajući to u vidu, označimo sa m najviši stepen kojim p deli n!. Pošto pk | q k ,
dobijamo da pm+k | n!q k za sve 1 6 k 6 n. S druge strane, stepen kojim p deli k!
je po Ležandrovoj formuli jednak:
¹ º ¹ º ¹ º Xs s
X
k k k k k
+ 2 + ... + s 6 j
6 < k.
p p p p 2j
j=1 j=1

Stoga pk - k!, tj. stepen kojim p deli k! nije veći od k − 1. Zbog toga,
¯µ ¶
¯
m+1 ¯ n! k
p ¯ k! q ,

gde je 1 6 k 6 n. Posmatranjem člana u relaciji (18) koji odgovara k = 0,


-
zaključujemo da pm+1 | n!. Medutim, to je kontradikcija sa izborom broja m.

63
36. MMO, 1987. (S.S.S.R.)

x2 +k x + p, gde je p ∈ N. Dokazati: ako je f (x) prost broj za sve


Neka je f (x) =jq
p
x ∈ {0, 1, . . . , 3 }, tada je f (x) prost broj za sve x ∈ {0, 1, . . . , p − 2}.

-
Rešenje. Zadatak će biti rešen ako dokažemo sledeće tvrdenje: ako postoji x ∈
{0, . . . , p − 2} takav da je f (x) složen
q i ako je y najmanji broj iz skupa {0, . . . ,
p − 2} sa tom osobinom, tada je y < p3 .
Posmatrajmo najpre razliku f (y) − f (x), pošto je to, s jedne strane, način da
”eliminišemo” slobodni član p, a s druge strane, ovu razliku možemo faktorisati:

f (y) − f (x) = (y + x + 1)(y − x).

Ako x uzima vrednosti redom od 0 do y − 1, tada prvi od faktora u zagradi uzima


vrednosti y + 1, y + 2, . . . , 2y, a drugi faktor vrednosti y, y − 1, . . . , 1. Zbog toga,
za svaki prost broj q 6 2y postoji x, 0 6 x 6 y − 1, tako da q | (f (y) − f (x)).
Stoga, ako odaberemo da q bude baš najmanji prost faktor složenog broja f (y) i
pretpostavimo da je q 6 2y, imamo da q | f (x0 ) za neko x0 ∈ {0, 1, . . . , y − 1}.
-
Medutim, pa definiciji y, broj f (x0 ) je prost, pa dobijamo da je f (x0 ) = q. Ali,
tada važe nejednakosti

y − x0 6 p − 2 < p + x0 + x20 = f (x0 ) = q

i
y + x0 + 1 6 p + x0 − 1 < p + x0 + x20 = f (x0 ) = q,
pa je relacija q | (y − x0 )(y + x0 + 1) = f (y) − f (x0 ) nemoguća. Kontradikcija,
pa mora biti q > 2y + 1. p
S druge strane, po definiciji broja q važi q 6 f (y), odnosno f (y) > q 2 .
Odatle je

f (y) = y 2 + y + p > q 2 > (2y + 1)2 = 4y 2 + 4y + 1 > 4y 2 + y,


q
tj. p > 3y 2 , pa je y < p3 , što je i trebalo dokazati.

64
37. Predlog za MMO, 1992. (Irska)

Dat je polinom
f (x) = x8 + 4x6 + 2x4 + 28x2 + 1.
Neka je p > 3 prost broj takav da postoji z ∈ N za koji p | f (z). Dokazati da
postoje celi brojevi z1 , . . . , z8 takvi da za g(x) = (x − z1 ) . . . (x − z8 ) važi da su
svi koeficijenti polinoma f (x) − g(x) deljivi sa p.

Rešenje. Za početak, nameće se očigledna smena X = x2 . Na taj način, imamo


polinom
f1 (X) = X 4 + 4X 3 + 2X 2 + 28X + 1.
Naredni korak u ”pojednostavljivanju” ovog polinoma je da eliminišemo (baš kao u
klasičnoj algebri, tj. teoriji algebarskih jednačina) kubni član. To postižemo putem
smene y = X + 1, pa dobijamo

h(y) = y 4 − 4y 2 + 32y − 28.

Ideja rešenja je da (po modulu p) faktorišemo ovaj polinom. Naš prvi cilj će biti da
pokažemo da dati uslovi omogućavaju da se h(y) predstavi kao razlika kvadrata.
Naime, primetimo da važi

h(y) = (y 2 + 2)2 − 2(2y − 4)2 .

Dokazaćemo da pod datim pretpostavkama postoji kvadrat celog broja koji daje
ostatak 2 pri deljenju sa p (tj. da je 2 kvadratni ostatak po modulu p).
Kako p | f (z) = h(t) (gde smo označili t = z 2 + 1), sledi

(t2 + 2)2 ≡ 2(2t − 4)2 (mod p).

Sada 2t − 4 nije deljivo sa p, jer bi u suprotnom bilo ili p = 2, ili t ≡ 2(mod p).
U drugom slučaju bismo imali (iz gornje kongruencije) da p | 62 , što je suprotno
pretpostavci p > 3. Dakle, postoji u tako da je u(2t − 4) ≡ 1(mod p), odakle je

[u(t2 + 2)]2 ≡ 2(mod p).

Označimo a = u(t2 + 2). Sada imamo:

h(y) ≡ (y 2 + 2)2 − (2a(y − 2))2 (mod p).

Nakon razvijanja gornje razlike kvadrata, jedan od faktora je

y 2 + 2 − 2a(y − 2) = y 2 − 2ay + a2 − a2 + 4a + 2 = (y − a)2 + (2 + 4a − a2 ),

65
što je po modulu p kongruentno sa (y − a)2 + 4a. Na potpuno analogan način,
drugi faktor je (mod p) jednak (y + a)2 − 4a, pa je tako

h(y) ≡ [(y − a)2 + 4a][(y + a)2 − 4a] (mod p).

Sada za gornje faktore primenjujemo prethodnu ideju, pokazujući da su oba


broja a, −a kvadratni ostaci po modulu p. Kako je h(t) deljivo sa p, jedan od
brojeva
(t − a)2 + 4a, (t + a)2 − 4a
je deljiv sa p. Pošto p 6= 2, bar jedan od brojeva a, −a je kongruentan nekom
kvadratu (mod p), tj. postoji b tako da je

b2 ≡ ±a(mod p).

U oba slučaja, zaključujemo da za neki ceo broj b važi:

h(y) ≡ [(y − b2 )2 + 4b2 ][(y + b2 )2 − 4b2 ] (mod p), (19)

i p deli broj

[(t − b2 )2 + 4b2 ][(t + b2 )2 − 4b2 ] = [(t − b2 )2 + 4b2 ](t + b2 − 2b)(t + b2 + 2b) =

= [(t − b2 )2 + (2b)2 ][z 2 + (b − 1)2 ][z 2 + (b + 1)2 ].


(Podsetimo se da je t = z 2 + 1.) Sledi da p deli zbir dva kvadrata; štaviše, u pitanju
je zbir dva kvadrata od kojih nijedan nije deljiv sa p (dovoljno je uočiti da p - z po
uslovima zadatka, kao i da p - 2b jer je p > 3 i p | b bi povlačilo p | a, što nije
moguće po izboru a). Prema tome, za neke cele brojeve v, w takve da p - v, p - w
imamo v 2 ≡ −w2 (mod p). Ako je c ceo broj takav da je cw ≡ 1(mod p), tada je

(cv)2 ≡ −(cw)2 ≡ −1(mod p), (20)

tj. −1 kvadratni ostatak (mod p). Uvrštavajući ovo u (19), h(y) se (mod p) razlaže
na linearne faktore:

h(y) ≡ (y − b2 − 2bcv)(y − b2 + 2bcv)(y + b2 − 2b)(y + b2 + 2b).

Vraćajući smenu y = x2 + 1 i vodeći računa o (20), sledi da je

[x2 − (b + cv)2 ][x2 − (b − cv)2 ][x2 − ((b − 1)cv)2 ][x2 − ((b + 1)cv)2 ]

razlaganje polinoma f (x) po modulu p, pa osam brojeva ±(b + cv), ±(b − cv),
±(b − 1)cv, ±(b + 1)cv zadovoljava tražene uslove.

66
38. Predlog za MMO, 1987. (Jugoslavija)

Neka je k > 2 proizvoljan prirodan broj. Dokazati da postoji iracionalan broj rk


takav da za svaki prirodan broj n važi:

brkn c ≡ −1(mod k).

Rešenje. Budući da su najjednostavnije iracionalnosti kvadratne, osnovna ideja je


da se broj rk konstruiše kao rešenje kvadratne jednačine. Naime, posmatrajmo
jednačinu
t2 − at + b = 0,
gde je a, b ∈ N, pri čemu su njena rešenja r, s realna. Tada je r + s = a i rs = b,
pa se indukcijom lako dokazuje da je za sve n ∈ N broj rn + sn ceo — primenjuje
se jednakost rn+1 + sn+1 = (r + s)(rn + sn ) − rs(rn−1 + sn−1 ), tj. ako označimo
Xn = rn + sn , tada je Xn+1 = aXn − bXn−1 . Ukoliko su r, s pozitivni, i pri tom
je još s < 1, tada je sn < 1 za sve n ∈ N, pa sledi da je

rn + sn = brn c + 1.

Drugim rečima, brn c = Xn − 1, pa se naš cilj svodi na to da obezbedimo da svi


brojevi Xn budu deljivi sa k. Očigledno, to će biti tačno ako su oba koeficijenta
a, b deljiva sa k.
Prema tome, posmatraćemo kvadratnu jednačinu oblika

x2 − kpx + kq = 0, (21)

a parametre p, q ∈ N ćemo odrediti tako da njena rešenja budu pozitivna, ira-


cionalna, i da jedno njeno rešenje s bude < 1. Kao što smo iz prethodnog videli,
tada se njeno drugo rešenje može uzeti za rk .
Da bi rešenja od (21) uopšte bila realna, potrebno je da bude

(kp)2 − 4kq > 0,


- uslovom
dok se pozitivnost rešenja i s < 1 obezbeduje
p
kp − (kp)2 − 4kq
0< < 1.
2
Ove dve relacije zajedno su ekvivalentne sa uslovima
4q
p2 > , p > q.
k
67
Očito, ove uslove je moguće postići. Iracionalnost rešenja uočenog kvadratnog
trinoma dobijamo ako uzmemo npr. q = k. Naime, tada je diskriminanta jednaka
k 2 (p2 − 4), što ne može biti potpun kvadrat, jer bi u suprotnom bilo

4 = p2 − t2 = (p − t)(p + t)

za neki prirodan broj t, što je nemoguće. Sada je dovoljno uzeti proizvoljno p >
k + 1.

68
39. Predlog za MMO, 1979. (Belgija)

Dokazati: ako prirodan


¥ broj ¦A nije potpun kvadrat, tada postoji prirodan broj n

tako da je A = n + n + 12 .

Rešenje. Pretpostavimo da se broj A0 ne može prikazati u traženom obliku. To



znači da interval [A0 , A0 + 1) ne sadrži nijedan član niza n + n + 12 , pa postoji
takvo n0 ∈ N da važi
√ 1
n0 + n0 + < A0 ,
2
kao i
√ 1
A0 + 1 6 n0 + 1 + n0 + 1 + .
2
Objedinjujući ove dve nejednakosti, dobijamo:
√ 1 √
n0 < A0 − n0 − 6 n0 + 1,
2
pa kvadriranjem imamo:
1
n0 < (A0 − n0 )2 − (A0 − n0 ) + 6 n0 + 1,
4
odnosno, dodavanjem (A0 − n0 ):
1
A0 < (A0 − n0 )2 + 6 A0 + 1.
4
Budući da su brojevi A0 i n0 prirodni, zaključak koji sledi iz ovih nejednakosti je

A0 = (A0 − n0 )2 .
-
Dakle, A0 je kvadrat prirodnog broja, pa je time tvrdenje zadatka dokazano.

69
40. Predlog za MMO 1989. (Rumunija)


Dokazati da niz brojeva bn 2c, n ∈ N, sadrži beskonačno mnogo potpunih kvad-
rata.
√ √
Rešenje. Uslov bn 2c = k 2 ekvivalentan je sa k 2 6 n 2 < k 2 + 1, odnosno

k 4 6 2n2 < (k 2 + 1)2 . (22)


-
Naš zadatak je da pronademo beskonačno mnogo parova prirodnih brojeva (n, k)
koji zadovoljavaju ovaj uslov. Zahtev da dva broja, od kojih je jedan kvadrat (k 4 ),
a drugi dvostruki kvadrat (2n2 ), budu ”bliski” asocira na Pelovu jednačinu

2x2 − y 2 = 1.

Ako ovu jednačinu zapišemo u obliku 2x2 = y 2 +1 i pomnožimo je sa y 2 dobićemo


-
2(xy)2 = y 4 + y 2 . Medutim, za svaki ceo broj y važi

y 4 6 y 4 + y 2 < y 4 + 2y 2 + 1 = (y 2 + 1)2 ,

što znači da za svako rešenje (a, b) gornje jednačine, brojevi n = ab, k = b zadovo-
ljavaju (22). Zato preostaje da pokažemo da posmatrana jednačina ima beskonačno
mnogo rešenja. √ √
Uočimo da je 1 = 1m = ( 2 + 1)m ( 2 − 1)m . Lako se pokazuje (indukcijom,
ili primenom binomne formule) da tada postoje prirodni brojevi xm , ym tako da je
√ √
( 2 + 1)m = xm 2 + ym .
-
Medutim, ako je m neparno, tada je
√ √
( 2 − 1)m = xm 2 − ym ,

Množenjem ove dve jednakosti dobijamo 2x2m = ym 2 + 1, pa sledi da je par

(xm , ym ) za neparne vrednosti m jedno od željenih rešenja. Očigledno, svi ovi


parovi su različiti, pa je zadatak rešen, pošto je
√ 2
bxm ym 2c = ym .

70
41. Predlog za MMO 1985. (Rumunija)


Dokazati da niz brojeva bn 2c, n ∈ N, sadrži beskonačno mnogo stepena dvojke.

-
Rešenje. Traženo tvrdenje se može iskazati i na sledeći način: postoji beskonačno
mnogo prirodnih brojeva n za koje postoji prirodan broj k tako da je

2k 6 n 2 < 2k + 1,

odnosno,
√ √ 1
2k−1 2 6 n < 2k−1 2 + √ .
2
h √ √ ´
-
Medutim, interval 2k−1 2, 2k−1 2 + √12 sadrži ceo broj ako i samo ako je

√ 1
{2k−1 2} > 1 − √ ,
2
gde {x} označava razlomljeni deo realnog broja x. Pretpostavimo sada, suprotno
-
traženom tvrdenju, da navedeni intervali sadrže samo konačno mnogo celih bro-
jeva. Tada postoji k0 ∈ N tako da za sve k > k0 važi
√ 1 1
{2k 2} 6 1 − √ ≈ 0, 292 . . . < .
2 2

Definišimo, za i > 0, niz realnih brojeva:



ri = {2i+k0 2}.

Očito, važi: (
2ri ri < 12 ,
ri+1 = {2ri } =
2ri − 1 ri > 12 .
1
Po pretpostavci, imamo ri < 2 za sve i > 0, pa je ri+1 = 2ri , odakle je ri = r0 2i
(i > 0). Sada imamo
1
r0 2i <
2

-
za sve i > 0, što je moguće samo ako je r0 = 0. Medutim, r0 = {2k0 2} je
iracionalan broj. Kontradikcija.

71
42. -
Madarska, 1969. (konkurs ”Kürschák József”)

Neka je n ceo broj. Dokazati: ako je broj


p
2 + 2 28n2 + 1

prirodan, onda je on potpun kvadrat.

Rešenje. Pretpostavimo da je
p ³ p ´
2 + 2 28n2 + 1 = 2 1 + 28n2 + 1 = m

za neki prirodan broj m. Tada je 28n2 + 1 neparan prirodan broj, odakle sledi
da m mora biti deljiv√sa 4, tj. m = 4k za neko k ∈ N. Uvrštavajući ovo u gornju
jednakost, dobijamo 28n2 + 1 = 2k − 1, odakle kvadriranjem sledi 28n2 + 1 =
4k 2 − 4k + 1 = 4k(k − 1) + 1, odnosno

7n2 = k(k − 1).

Budući da je (k, k − 1) = 1, postoje celi brojevi q, r tako da važi jedna od dve


mogućnosti:

(1) k = q 2 , k − 1 = 7r2 ,

(2) k = 7q 2 , k − 1 = r2 .

U slučaju (1) traženi zaključak neposredno sledi, pošto je tada

m = 4k = 4q 2 = (2q)2 .

S druge strane, slučaj (2) je nemoguć, pošto bismo tada imali r2 = 7q 2 − 1, što
-
bi značilo da važi r2 ≡ −1(mod 7). Medutim, lako se proverava da kvadrati celih
brojeva daju ostatke 0, 1, 2, 4 pri deljenju sa 7.

Komentar.
√ Može se pokazati da postoji beskonačno mnogo celih brojeva n za koje
je 28n2 + 1 prirodan broj. Kao što se vidi iz gornjeg rešenja, ako je (q, r) rešenje
Pelove jednačine
x2 − 7y 2 = 1,
tada n = qr ima traženu osobinu. Postojanje beskonačno mnogo rešenja ove
jednačine pokazuje se potpuno analogno
√ kao i u rešenju zadatka br. 40, posmat-
ranjem binomnih razvoja (8 ± 3 7)m (pri čemu uzimamo baš ove koeficijente jer
je 82 − 7 · 32 = 64 − 63 = 1).

72
43. MMO 1989. (Švedska)

Za koje prirodne brojeve n postoji prirodan broj m tako da ni jedan od brojeva


m + 1, m + 2, . . . , m + n nije stepen prostog broja?

Rešenje. Broj x nije stepen prostog broja ako i samo ako ima bar dva prosta faktora,
što je ekvivalentno rastavljanju x = ab na dva uzajamno prosta činioca a, b > 1.
Prema tome, ako želimo da obezbedimo da broj m + k ne bude stepen prostog
broja, možemo pokušati da posmatramo broj m deljiv sa k, jer tada iz m = km0
sledi m + k = k(m0 + 1). Da bi k i m0 + 1 bili uzajamno prosti, dovoljno je da
k | m0 .
Pri svemu tome, mora biti k > 2. Kako bismo obezbedili da ovaj uslov bude
ispunjen, uvedimo smenu m = m0 + 1 i posmatrajmo niz brojeva

m0 + 2, m0 + 3, . . . , m0 + n + 1.

Kao što smo videli, uslov k | m0 omogućava faktorizaciju m0 + k = k(m00 + 1) za


pogodno m00 ∈ N. Zbog toga, neka m0 ima oblik m0 = (n + 1)!m1 . Tada za sve
2 6 k 6 n + 1 važi
µ ¶
(n + 1)!
m0 + k = k m1 + 1 .
k

U skladu sa gornjim razmatranjima, uslov k | m1 povlači da


µ ¶
(n + 1)!
k, m1 + 1 = 1,
k

pa tada m0 + k nije stepen prostog broja. Dakle, ako odaberemo

m1 = (n + 1)!,

imaćemo da nijedan od brojeva m0 + 2, . . . , m0 + n + 1 nije stepen prostog broja.


Stoga traženi broj m postoji za sve n ∈ N, pošto smo upravo pokazali da je do-
voljno uzeti
m = [(n + 1)!]2 + 1.

73
44. Predlog za MMO 1991. (Bugarska)

Naći najveći prirodan broj k za koji


¯³ ´
¯ 1992 1990
1991k ¯ 19901991 + 19921991 .

-
Rešenje. Radi bolje preglednosti, označimo a = 1991. Naš cilj je sada odredivanje
najvećeg prirodnog broja k za koji
¯³ ´
¯ a+1 a−1
ak ¯ (a − 1)a + (a + 1)a .

Pošto je a neparan broj, očito je da prvi sabirak u zagradi daje ostatak −1 pri
deljenju sa a, dok drugi daje ostatak 1. Zbog toga ćemo posebno razmotriti najviše
a−1 a+1
stepene kojim a deli (a + 1)a − 1, odnosno (a − 1)a + 1 (ideja je sledeća: ako
se ispostavi da su ova dva najviša stepena različita, onda je manji od njih upravo
traženo k).
n
Razvijanjem izraza ¡(a +¢ 1)a po binomnom obrascu uočavamo da je poslednji
n
sabirak 1, pretposlednji a1 a = an+1 , dok su ostali deljivi sa an+2 . Ovo zapažanje
-
sada potvrdujemo indukcijom: dokazujemo da za sve prirodne brojeve n postoji sn
takav da a - sn i
n
(1 + a)a = 1 + sn an+1 .
Zaista, za n = 1 imamo:
µ ¶ µ ¶ µ ¶
a a a 2 a a
(1 + a) = 1 + a+ a + ... + a =
1 2 a
µ µ ¶ µ ¶ ¶
2 a a
=1+a 1+ + a + . . . = 1 + s1 a2 .
2 3
¡ ¢
Pošto je a neparno, to a | a2 , pa zato a ne deli
µ ¶ µ ¶
a a
s1 = 1 + + a + ...
2 3
-
Iz pretpostavke da navedeno tvrdenje važi za n, dobijamo:
µ ¶ µ ¶
an+1
¡ ¢
n+1 a a n+1 a 2 2n+2
(1 + a) = 1 + sn a =1+ sn a + s a + ... =
1 2 n
µ µ ¶ ¶
n+2 a 2 n
=1+a sn + s a + . . . = 1 + sn+1 an+2
2 n
i pošto a - sn , to a - sn+1 .

74
Na potpuno analogan način se pokazuje da za svaki prirodan broj n postoji
prirodan broj tn takav da a - tn i da važi
n
(a − 1)a = −1 + tn an+1 .
-
Prema tome, iz gornja dva tvrdenja imamo da je aa najveći stepen od a koji
a−1
-
deli (a + 1)a − 1 (prvo od gornja dva tvrdenja za n = a − 1), kao i da je najveći
a+1
stepen kojim a deli (a − 1) a -
+ 1 jednak aa+2 (drugo tvrdenje za n = a + 1).
Otuda je traženi broj kmax = a = 1991.

75
45. Predlog za MMO 1982. (Australija)

¯¡ ¢
(a) Dokazati da (n + 1) ¯ 2n
n za sve n ∈ N.
(b) Za sve k ∈ N, naći najmanji prirodan broj Ck takav da
¯ µ ¶
¯ 2n
¯
(n + k + 1) ¯ Ck
n+k

za sve n > k.

Rešenje. (a) Važi:


µ ¶ µ ¶
1 2n (2n)! 1 2n
= = ,
n+1 n (n + 1)!n! n n−1

odakle dobijamo µ ¶ µ ¶
2n 2n
n = (n + 1) .
n n−1
¡ ¢
Kako je (n, n + 1) = 1, to sledi (n + 1) | 2n n .
(b) Iz prethodne tačke
¡ 2n ¢sledi da je C 0 = 1, pa zato pretpostavimo da je k > 0.
Ako je n = k, tada je n+k = 1, što znači da mora biti Ck > 2k + 1. Pokušaćemo
da pokažemo da je baš Ck = 2k + 1, tj. da važi
¯ µ ¶
¯ 2n
(n + k + 1) ¯¯ (2k + 1)
n+k

za sve n > k. Kako je


µ ¶ µ ¶
2n 2n
(2k + 1) = [(n + k + 1) − (n − k)] ,
n+k n+k

dovoljno je pokazati da
¯ µ ¶
¯ 2n
(n + k + 1) ¯¯ (n − k) .
n+k
-
Medutim, na analogan način kao i u tački (a), dobijamo:
µ ¶ µ ¶
2n (2n)! 2n
(n − k) = = (n + k + 1) ,
n+k (n + k)!(n − k − 1)! n−k−1

odakle sledi željeni zaključak.

76
46. Predlog za BMO, 1994. (Rumunija); Bugarska, 1994.

Neka je p > 3 prost broj. Pokazati da je broj


µ ¶ µ ¶ µ ¶
p p p
+ + ... + j k
1 2 2p
3

deljiv sa p2 .
j k
2p
Rešenje. Označimo sa S datu sumu binomnih koeficijenata i q = 3 . Za 1 6
k 6 q imamo: µ ¶
p p(p − 1)(p − 2) . . . (p − k + 1)
= =
k k!
p · [(p − 1)(p − 2) . . . (p − k + 1)] · [(k + 1) . . . q]
= .
q!
Naravno, pošto je q < p, sledi da p - q!. Takode, - svaki od binomnih koeficijenata
-
koji čine S je očito deljiv sa p, pa je tvrdenje koje dokazujemo, p2 | S, ekvivalentno
q!
sa p | p S. Pri tome je iz gornje jednakosti
µ ¶
q! p
= [(p − 1)(p − 2) . . . (p − k + 1)] · [(k + 1) . . . q].
p k

Za izraz u prvoj od dve uglaste zagrade važi

(p − 1)(p − 2) . . . (p − k + 1) ≡ (−1)k−1 (k − 1)! (mod p),

odakle je
µ ¶
q! p q!
≡ (−1)k−1 (k − 1)!(k + 1) . . . q = (−1)k−1 (mod p).
p k k

Prema tome,
q
X (−1)k−1
q!
S ≡ q! (mod p).
p k
k=1

Zbog toga, zadatak će biti rešen ako pokažemo da je brojilac sledećeg racionalnog
broja (nakon skraćivanja) deljiv sa p:
q
X (−1)k
.
k
k=1

77
Pošto je p > 3, prost broj p mora biti oblika 6m ± 1. Ako je p = 6m + 1, tada
imamo q = 4m i
1 1 1
−1 + − + . . . + =
2 3 4m
· µ ¶¸
1 1 1 1 1
− 1 + + ... + −2 + + ... + =
2 4m 2 4 4m
µ ¶
1 1 1 1
= − 1 + + ... + − 1 − − ... − =
2 4m 2 2m
µ ¶
1 1 1
=− + + ... + =
2m + 1 2m + 2 4m
µ ¶
1 1 1 1
=− + + + + ... =
2m + 1 4m 2m + 2 4m − 1
µ ¶
6m + 1 6m + 1 6m + 1
=− + + ... + =
4m(2m + 1) (4m − 1)(2m + 2) 3m(3m + 1)
µ ¶
1 1 1
= −p + + ... + =
4m(2m + 1) (4m − 1)(2m + 1) 3m(3m + 1)
pA pA(2m − 1)!
=− =−
2m(2m + 1) . . . 4m q!
za neki prirodan broj A. Pošto p - q!, posmatrani razlomak će nakon skraćivanja
imati brojilac deljiv sa p.
Slučaj p = 6m − 1 se razmatra analogno.

78
47. MMT, Mersch, Luksemburg, 1980. (Jugoslavija)

-
Neka je p prost broj, a n prirodan broj. Dokazati da su sledeća dva tvrdenja ekvi-
valentna:
¡ ¢
(i) Nijedan od binomnih koeficijenata nk , 1 6 k < n, nije deljiv sa p.

(ii) n = ps q − 1 za neke cele brojeve s, q takve da je s > 0 i 0 < q < p.

Rešenje. (i) ⇒ (ii) Pošto polazimo od pretpostavke da p ne deli


µ ¶
n n(n − 1) . . . (n − k + 1)
=
k 1 · 2 · ... · k

za sve 1 6 k 6 n − 1, imamo da ta pretpostavka važi specijalno za sve k < p (sem


ako je n < p − 1, no tada je trivijalno n = ps q − 1 za s = 0 i q = n + 1, pa je
tražena implikacija dokazana), pa i za k = p − 1. Pošto p - (p − 1)!, sledi da broj

n(n − 1) . . . (n − p + 2)

nije deljiv sa p, što znači da n ne može davati nijedan od ostataka 0, 1, . . . , p − 2


pri deljenju sa p. Drugim rečima, n ≡ −1(mod p), tj. p | (n + 1). Stoga za neke
s, q > 0 važi n = ps q − 1, pri čemu p - q. Preostaje da se pokaže da mora biti
q < p.
Medu- ”uzastopnim” binomnim koeficijentima imamo sledeću vezu:
µ ¶ µ ¶
n n . . . (n − k + 2)(n − k + 1) n+1−k n
= = . (23)
k k! k k−1

S obzirom na to, brojilac i imenilac razlomka n+1−kk moraju za sve 1 6 k 6 n


sadržati prost faktor p sa istim stepenom,
¡ ¢ jer bismo u suprotnom za najmanje k
koje nema tu osobinu imali da je nk deljivo sa p. Medutim,
-

n+1−k ps q − k
= ,
k k
pa odmah imamo da važi jednakost najviših stepeni kojima p deli brojilac i imenilac
posmatranog razlomka kad god ps - k. Prema tome, dovoljno je posmatrati brojeve
oblika k = ps m, m 6 q. Tada je

n+1−k ps q − ps m q−m
= = .
k ps m m

79
Ukoliko bi bilo q > p (slučaj q = p je očito nemoguć, jer p - q), tada bismo
za m = q − p imali kontradikciju sa (i): brojilac posmatranog razlomka bi bio
q − (q − p) = p, a imenilac q − p, pri čemu p - (q − p). Stoga zaključujemo da
mora biti q < p, što okončava dokaz tražene implikacije.
(ii) ⇒ (i) Ova implikacija se¡ neposredno
¢ dokazuje indukcijom, koristeći jed-
n -
nakost (23). Naime, za¡ k =¢ 1, 1 = n očito nije deljivo sa p. Ukoliko podemo
n
od pretpostavke da p - k−1 , tada je na osnovu (23) za kompletiranje induktivnog
koraka dovoljno dokazati da su najviši stepeni kojima p deli n + 1 − k i k jednaki.
Neka je k = pr m, gde p - m i r > 0. Ako je r < s, tada je

n + 1 − k = ps q − pr m = pr (ps−r q − m)

deljivo sa pr , ali ne i sa pr+1 (baš kao i k). S druge strane, ako je r = s (slučaj
r > s je jasno nemoguć, jer bismo tada imali k > ps+1 > ps q > n), tada je

n + 1 − k = ps (q − m),

i pri tome p - (q − m), jer je m < q < p. Zato su i n + 1 − k i k deljivi sa ps , ali ne


i sa ps+1 . Time je induktivni korak dokazan, pa odmah sledi tražena implikacija.

80
48. Čehoslovačka, 1988.

Naći sve prirodne brojeve m tako da važi:


µ ¶
m(m + 1)
1! 3! 5! . . . (2m − 1)! = !.
2

Rešenje. Ideja rešenja je da se posmatraju prosti faktori leve i desne strane date
jednačine. Pošto za proizvoljan prost broj p i prirodan broj n važi da p - n! ako i
samo ako n < p, zaključujemo da egzistencija prostog broja p takvog da

m(m + 1)
2m − 1 < p 6
2
povlači da m ne može biti rešenje (jer je tada desna strana deljiva sa p, dok leva
nije). Po Bertranovom stavu, takav prost broj će postojati kad god je

m(m + 1)
> 2(2m − 1),
2
-
što je ekvivalentno sa m2 − 7m + 4 > 0. Medutim, ova nejednakost je očito tačna
za m > 7, pa preostaju da se razmotre brojevi m 6 6.
Za m = 6 je 2m − 1 = 11 < 13 < 21 = 6·7 2 , dok za m = 5 imamo
2m − 1 = 9 < 11 < 15 = 5·6 2 , pa ove vrednosti m nisu rešenja posmatrane
jednačine.
S druge strane, m = 1, 2 očigledno jesu rešenja. Rešenje je i m = 3, jer je
- i m = 4, pošto je 1! 3! 5! 7! = 6! 7! = 720 · 7! =
1! 3! 5! = 6 · 5! = 6!, a takode
10 · 9 · 8 · 7! = 10!.

81
49. Predlog za MMO, 1989. (Kolumbija)

Neka f (m) označava najveći ceo broj k za koji 2k | m!. Dokazati da za sve
prirodne brojeve n postoji beskonačno mnogo prirodnih brojeva m tako da je

m − f (m) = n.

Rešenje. Po Ležandrovoj formuli,


Xjmk
f (m) = .
2k
k>1

Ako je ar ar−1 . . . a1 a0 binarni prikaz broja m, tj.

m = ar 2r + . . . + a1 21 + a0 ,

tada za 1 6 k 6 r važi
jmk
= ar 2r−k + . . . + ak+1 21 + ak = ar . . . ak+1 ak
2k
(za k > r posmatrani ceo deo je 0). Dakle,

X r
r X r
X i
X r
X
f (m) = ai 2i−k = ai 2i−k = ai (2i − 1).
k=1 i=k i=1 k=1 i=1

Zato je
m − f (m) = ar + . . . + a1 + a0 .
Prema tome, m − f (m) = n ako i samo ako m u binarnom zapisu ima tačno n
jedinica. Takvih brojeva ima očito beskonačno mnogo.

82
50. Predlog za MMO, 1985. (Bugarska)

Neka su a i b celi, a n prirodan broj. Dokazati:


¯Ã !
¯ n−1
¯ n−1 Y
n! ¯ b (a + kb) .
¯
k=0

Rešenje. Ako je p prost broj i s > 0 najveći ceo broj za koji ps | n!, tada je po
Ležandrovoj teoremi:

X¹ n º X∞ j
n k
s= 6 < n, (24)
pm 2m
m>1 m=1

pa je s 6 n − 1 (isto pomoćno tvrdenje - smo imali u zadatku br. 35). Ako p | b,


tada p n−1 | b n−1 s
, a time i p | b n−1 . U suprotnom, p - b, pa p deli bar jedan
od a, a + b, . . . , a + (p − 1)b, pošto ovaj niz od p brojeva tada čini potpun sistem
ostataka po modulu p (u suprotnom bi dva takva broja davala isti ostatak (mod p),
pa bi p | (i − j)b za neke i, j ∈ {0, 1, . . . , p − 1}, što je nemoguće).
¥ ¦ Zato p deli
2 2
bar bn/pc faktora proizvoda, datog u zadatku, p deli bar n/p tih faktora, itd.
Zaključak je da je stepen prostog broja p u tom proizvodu bar s (vidi (24)), odakle
-
sledi tvrdenje zadatka.

83
51. Predlog za MMO, 1983. i 1991. (S.S.S.R.)

Neka je an poslednja nenula cifra u dekadnom prikazu broja n!. Da li niz a1 , a2 , . . .


može biti, počev od nekog člana, periodičan?

Rešenje. Ideja rešenja je da posmatramo jednake uzastopne članove datog niza


cifara, pa da na osnovu dobijenih informacija pokušamo da dokažemo negativan
odgovor na postavljeno pitanje. Naime, ako definišemo skup

A = {n : an−1 = an },

tada iz pretpostavke da je dati niz periodičan za n > n0 sa periodom T dobijamo


da iz n > n0 i n ∈ A (tj. an−1 = an ) sledi

an−1+kT = an+kT ,

odnosno n + kT ∈ A, za sve k > 0. Koristeći osobine skupa A, dokazaćemo da


ovo nije moguće.
Pokušaćemo najpre da iz uslova an−1 = an saznamo nešto o samom broju n.
Prikažimo (n−1)! = 10q (10u+an−1 ) i n = 10r (10v +x), pri čemu je 1 6 x 6 9.
Tada je
n! = (n − 1)! · n = 10q+r (10s + an−1 x),
gde je s = 10uv + ux + an−1 v.
Ukoliko 5 - an−1 x, tada je an , poslednja nenula cifra od n!, jednaka cifri je-
dinica proizvoda an−1 x. Ta cifra treba da bude jednaka an−1 , tj. važi

10 | (an−1 x − an−1 ) = an−1 (x − 1).

Pošto ispitujemo slučaj kada 5 - an−1 x, sledi da 5 | (x − 1), odnosno x ∈ {1, 6}.
Drugi slučaj je kada 5 | an−1 x, tj. an−1 = 5 ili x = 5. Ispitaćemo da li moguća
jednakost at = 5 za neko t. U tu svrhu, prikažimo t! = 2α 5β γ, gde γ nije deljivo
ni sa 2, ni sa 5. Po Ležandrovoj formuli je
¹ º ¹ º ¹ º ¹ º
t t t t
α = + + + ... + i + ...,
2 4 8 2
¹ º ¹ º ¹ º ¹ º
t t t t
β = + 2 + 3 + ... + i + ...,
5 5 5 5

pa se lako pokazuje da važi α > β kad god je t > 2. Zbog toga je t! = 10β (2α−β γ),
što znači da se at poklapa sa cifrom jedinica broja 2α−β γ. Za t > 2, ta cifra je

84
očigledno parna, dok je a1 = 1. Prema tome, at 6= 5 za sve t ∈ N, pa iz 5 | an−1 x
sledi x = 5.
Sumirajući prethodna razmatranja, možemo zaključiti da n ∈ A povlači da je
poslednja nenula cifra broja n jedna od 1, 5, 6.
Uočimo sada da za proizvoljno m ∈ N imamo (10m )! = 10m · (10m − 1)!,
zbog čega je 10m ∈ A. Kao što je ranije rečeno, tada za sve k > 0 i za sve m takve
da je 10m > n0 mora biti
10m + kT ∈ A.
Neka je
T = 2a 5b c,
pri čemu je c neparan broj koji nije deljiv sa 5. Ideja je da pokažemo da T ima
umnožak `T čija poslednja nenula cifra nije nijedna od 1, 5, 6, jer tada za dovoljno
veliko m (naime, za m takvo da je 10m > max(`T, n0 )) važi da je poslednja nenula
cifra broja 10m + `T ista kao i poslednja nenula cifra broja `T . Pošto ta cifra nije
u skupu {1, 5, 6}, dobićemo kontradikciju sa uslovom 10m + `T ∈ A (koji sledi iz
prepostavljene periodičnosti niza cifara an ).
-
Medutim, traženi umnožak se lako nalazi. Naime, imamo

2b 5a T = 10a+b c,

pa je poslednja nenula cifra broja 2b 5a T ista kao i cifra jedinica broja c. Ako ta cifra
nije 1 (već 3, 7 ili 9), željeni cilj je postignut. Ali, ako je ta cifra 1, posmatrajmo
dvostruko veći broj
2b+1 5a T = 10a+b · 2c.
Poslednja nenula cifra ovog broja je cifra jedinica broja 2c, a to je u posmatranom
slučaju 2. Time je dokaz neperiodičnosti niza an kompletiran.

85
52. MMO, 1978. (Kuba)

Posmatrajmo sve parove prirodnih brojeva (m, n), m < n, sa osobinom da se


poslednje tri cifre u dekadnom zapisu brojeva 1978m i 1978n poklapaju. Naći sve
takve parove (m, n) za koje je m + n minimalno.

Rešenje. Uslov zadatka možemo zapisati kao

1978m (1978n−m − 1) = 1000q = 23 53 q,

za neko q > 1, odakle dobijamo 8 | 1978m i 125 | (1978n−m − 1). Kako je


1978 = 2 · 989, to prvi uslov daje m > 3. Iz drugog uslova imamo:

1 ≡ 1978n−m ≡ (−2)n−m (mod 5),

što je moguće samo ako je n − m = 4k za neki prirodan broj k. Preostaje da


odredimo najmanji prirodan broj k za koji je 19784k −1 deljivo sa 125. Neposredno
dobijamo
19784 ≡ 6(mod 125),
pa se posmatrani uslov svodi na

6k ≡ 1(mod 125).

Iz Ojlerove teoreme sledi

6100 = 6ϕ(125) ≡ 1(mod 125).

Kako je 6100 − 1 = (650 − 1)(650 + 1) i poslednja cifra broja 650 + 1 je 7, sledi


da 125 | (650 − 1). Ponavljajući ovaj argument još jednom u odnosu na 650 − 1 =
(625 − 1)(625 + 1), dobijamo da 125 | (625 − 1). Zbog toga, najmanje k za koje
125 | (6k − 1) zadovoljava k | 25, tj. k ∈ {1, 5, 25}. Direktno proveravamo da
65 −1 nije deljivo sa 125 (kao ni 61 −1). Stoga je k = 25 najmanji broj sa traženom
osobinom, pa je suma m + n minimalna za n = 103, m = 3.

86
53. BMO, 1984. (Bugarska)

Dokazati da za sve prirodne brojeve m postoji prirodan broj n > m takav da se


dekadni zapis broja 5n dobija dopisivanjem izvesnog broja cifara sleva dekadnom
zapisu broja 5m .

Rešenje. Uslov zadatka se može zapisati kao 10r | (5n − 5m ), gde je r broj cifara
u dekadnom zapisu broja 5m , tj. r = blog10 5m c + 1. Pošto je r 6 m, posmatrana
relacija deljivosti je ekvivalentna sa 2r | (5n − 5m ) = 5m (5n−m − 1), tj. sa

2r | (5n−m − 1).

Po Ojlerovoj teoremi, važi:


r)
5ϕ(2 ≡ 1(mod 2r ).

Ali, tada je očito da se za n oblika

n = m + ϕ(2r )k = m + 2r−1 k,

k ∈ N, dobija
r
5n = 5m (5ϕ(2 ) )k ≡ 5m (mod 2r ),
što se i tražilo.

87
54. Predlog za MMO, 1976. (Vijetnam)

Dokazati da postoji beskonačno mnogo prirodnih brojeva n takvih da se u dekad-


nom zapisu broja 5n pojavljuje 1976 uzastopnih nula.

Rešenje. Zadatak će biti rešen ako dokažemo da postoji beskonačno mnogo prirod-
nih brojeva n takvih da se 5n može zapisati u obliku

5n = 10k+1976 q + r,

za neke prirodne brojeve q, r, k, pri čemu je r < 10k . Da bi to bilo ispunjeno,


mora biti 10k+1976 < 5n , odakle je n > k + 1976, pa zato 5k+1976 | r. Pišimo
-
r = 5k+1976 s, pri čemu će s biti naknadno odredeno. Zbog 5k+1976 | r < 10k ,
sledi 5 k+1976 k k
< 10 , tj. 2 > 51976 . To znači da mora biti

k > b1976 log2 5c + 1 = 4589.

Sada posmatrana jednačina glasi 5n−k−1976 = 2k+1976 q+s, odnosno, nakon smene
t = k + 1976:
5n−t = 2t q + s,
gde je q > 0 (što je ekvivalentno sa n − t > 0). Kako bismo mogli da primenimo
Ojlerovu teoremu, zgodno je da odaberemo s = 1. Pošto je
t
5ϕ(2 ) ≡ 1(mod 2t ),

svi traženi uslovi su ispunjeni za sve t > 4589 + 1976 = 6565 i sve n za koje važi
n>ti
ϕ(2t ) = 2t−1 | (n − t).
Drugim rečima, za sve m ∈ N i t > 6565, broj

n = 2t−1 m + t

ima željenu osobinu.

88
55. Predlog za BMO, 1989. (Kipar)

Dokazati da ¯ ³ nn ´
¯ n
1989 ¯ nn − nn

za sve prirodne brojeve n > 3.

Rešenje. Radi kraće notacije, označimo broj u zagradi sa a. Ovaj broj se može
faktorisati kao
nn n n nn n
a = nn − nn = nn (nn −n − 1),
što motiviše da označimo
n n −n
b = nn − nn = nn (nn − 1).

Najzad, neka je c = nn − n. Sada je


n
a = nn (nb − 1), b = nn (nc − 1), c = n(nn−1 − 1).

Kako je 1989 = 9 · 13 · 17, ideja rešenja je da dokažemo da ϕ(d) | b za sve


d ∈ {9, 13, 17}, budući da tada važi:

(n, d) = 1 ⇒ d | (nϕ(d) − 1) | (nb − 1) | a.

S druge strane, ako za d ∈ {13, 17} imamo (n, d) 6= 1, tada d | n | a, dok iz


(n, 9) 6= 1 sledi 3 | n, pa 9 | n2 | a. Prema tome, zadatak će biti rešen ako
pokažemo da [ϕ(9), ϕ(13), ϕ(17)] = [6, 12, 18] = 48 | b.
Najpre, 3 | n ⇒ 3 | b. S druge strane, ako n nije deljivo sa 3, imamo (n, 3) = 1.
No, tada 2 | c (jer to važi ako je n parno, a u suprotnom je nn−1 − 1 parno). Dakle,
ϕ(3) = 2 | c, pa po Ojlerovoj teoremi 3 | (nc − 1) | b.
Dalje, ako je n parno, tada 2n | b, znači 16 | b, pošto je n > 4. U suprotnom,
neka je n = 2k + 1. Sada imamo:

c = (2k + 1)((2k + 1)2k − 1) = (2k + 1)((4k(k + 1) + 1)k − 1) =

k(k + 1)
= (2k + 1)(4k(k + 1))N = 8(2k + 1) N.
2
Prema tome, ϕ(16) = 8 | c i zato, ponovo se koristeći Ojlerovom teoremom, sledi
16 | (nc − 1) | b. Time je rešenje kompletirano.

89
56. Predlog za MMO, 1984. (Kanada)

(a) Dokazati da postoji beskonačno mnogo parova prirodnih brojeva (m, n) takvih
da je 4mn − m − n + 1 potpun kvadrat.
(b) Dokazati da ne postoji nijedan par prirodnih brojeva (m, n) takav da je 4mn −
m − n potpun kvadrat.

Rešenje. (a) Posmatramo jednačinu

4mn − m − n + 1 = k 2 .

Nakon množenja sa 4 i oduzimanja 3 od obe strane, moguće je faktorisati levu


stranu, tako da se dobija

(4m − 1)(4n − 1) = 4k 2 − 3.

Ideja koja vodi nalaženju beskonačnog niza rešenja ove jednačine je da se za k


uvede odgovarajuća smena koja će 4k 2 − 3 transformisati u razliku kvadrata. Oči-
gledno, linearna smena oblika k = at + b (a, b ∈ Z) nije odgovarajuća, jer nakon
kvadriranja ostaje linearni član. Zbog toga ćemo pokušati sa smenom k = at2 + b.
Tada je

4k 2 − 3 = 4(at2 + b)2 − 3 = 4a2 t4 + 8abt2 + (4b2 − 3) =

= (2at2 + b)2 − (−4abt2 ) + (3b2 − 3).


Prema tome, ako izaberemo koeficijente a, b tako da je b2 = 1, ab < 0 i da −ab
bude potpun kvadrat, postići ćemo naš cilj. Očito je a = 1, b = −1 jedan od
adekvatnih izbora, pa tako za k = t2 − 1 imamo

4k 2 − 3 = (2t2 − 1)2 − 4t2 = (2t2 − 2t − 1)(2t2 + 2t − 1).

Kako su brojevi 2t2 − 2t = 2t(t − 1) i 2t2 + 2t = 2t(t + 1) deljivi sa 4, možemo


njih ”proglasiti” za 4m, odnosno 4n. Tako, imamo željeni niz rešenja: naime, ako
je t ∈ N, za
1 1
m = t(t − 1), n = t(t + 1),
2 2
važi 4mn − m − n + 1 = (t2 − 1)2 .
(b) Slično kao malopre, jednačina 4mn − m − n = k 2 je ekvivalentna sa

(4m − 1)(4n − 1) = 4k 2 + 1.

90
-
Pretpostavimo, suprotno tvrdenju zadatka, da postoje prirodni brojevi m, n, k za
koje važi ova jednakost. Neka je p prost broj koji deli 4m − 1. Tada je

(2k)2 ≡ −1(mod p).

S druge strane, po maloj Fermaovoj teoremi je

(2k)p−1 ≡ 1(mod p).

Kako je p očigleno neparan, sledi


p−1 p−1
1 ≡ (2k)p−1 = ((2k)2 ) 2 ≡ (−1) 2 (mod p),

odakle je p ≡ 1(mod 4). Kako je u ovom razmatranju prost faktor p od 4m − 1 bio


proizvoljan, zaključujemo (nakon što napišemo 4m − 1 kao proizvod svojih prostih
faktora) da je
4m − 1 ≡ 1(mod 4),
kontradikcija.

91
57. (a),(b): Rumunija, 1978.; (c): Predlog za MMO, 1985. (Rumunija)

(a) Dokazati da za svaki prirodan broj a > 3 postoji beskonačno mnogo prirodnih
brojeva n za koje n | (an − 1).
(b) Naći sve prirodne brojeve n za koje n | (2n − 1).
(c) Neka je k > 2 i neka su n1 , n2 , . . . , nk prirodni brojevi takvi da

ni+1 |(2ni − 1)

za sve 1 6 i 6 k − 1, kao i n1 |(2nk − 1). Dokazati da je n1 = . . . = nk = 1.

Rešenje. (a) Pretpostavimo da prirodan broj n deli m = an − 1. Tada je m = nk


za neko k ∈ N, pa je

am − 1 = (an )k − 1 = (an − 1)(an(k−1) + . . . + an + 1).

Zaključujemo da tada m | (am − 1). Stoga, ako definišemo niz

n1 = 1, nr+1 = anr − 1, (r > 1)

tada iz prethodnih razmatranja neposredno sledi da svaki član ovog niza ima željenu
osobinu. Preostaje da uočimo da iz pretpostavke a > 3 sledi da je ovaj niz strogo
rastući, tj. da se sastoji iz različitih brojeva.
(b) Očigledno, n = 1 zadovoljava traženi uslov. Pretpostavimo da postoji n >
2 koje zadovoljava navedeni uslov. Neka qn označava najmanji prost faktor broja
n. Dokažimo da važi: ako je n > 1 i p | (2n − 1), tada je p > qn . U tom slučaju,
imaćemo očiglednu kontradikciju, jer n | (2n − 1) povlači qn | (2n − 1).
Najpre, treba primetiti da ako za prirodne brojeve a, b važi

2a ≡ 2b ≡ 1(mod p),

tada je
2(a,b) ≡ 1(mod p).
Naime, ako je a > b i a = qb + r, tada važi

2r ≡ (2b )q 2r = 2a ≡ 1(mod p).

Nastavljajući očiglednu primenu Euklidovog algoritma u eksponentu, dobijamo


upravo željeni zaključak. Sada, pošto po maloj Fermaovoj teoremi važi 2p−1 ≡
1(mod p), za d = (n, p − 1) imamo 2d ≡ 1(mod p). Zbog toga je d > 1, pa važi
qn 6 d. S druge strane, d | (p − 1), što implicira p > d > qn .
Prema tome, n = 1 je jedino rešenje.

92
(c) Pretpostavimo da je n1 > 1. Tada je redom nk > 1,. . . , n2 > 1. Neka qn
označava, kao i u (b), najmanji prost faktor broja n. Iz uslova zadatka sledi

qn2 | 2n1 − 1,
-
pa po pomoćnom tvrdenju iz prethodnog zadatka (p | (2n − 1) ⇒ qn < p) sledi
qn1 < qn2 . Ponavljajući ovo razmatranje, dobijamo:

qn1 < qn2 < . . . < qnk < qn1 .

Kontradikcija.

93
58. S.A.D., 1991.

Dokazati da je za svaki prirodan broj n ∈ N, niz


2 22
2, 22 , 22 , 22 , . . . (mod n)

konstantan počev od nekog člana.

Rešenje. Označimo sa ai , i > 1, članove datog niza (preciznije, ovaj niz je definisan
sa a1 = 2 i ai+1 = 2ai za i > 1). Zadatak zapravo traži da se pokaže da je za sve
prirodne brojeve n razlika ai+1 − ai deljiva sa n za dovoljno veliko i. Drugim
rečima, ako označimo bi = ai+1 − ai , cilj je da se dokaže:

za sve n ∈ N postoji in ∈ N tako da i > in ⇒ n | bi . (25)

Najpre ćemo malo transformisati bi . Naime, važi:

bi = ai+1 − ai = 2ai − 2ai−1 = 2ai−1 (2ai −ai−1 − 1) = ai (2bi−1 − 1),

pri čemu smo po potrebi označili a0 = 1. Koristeći ovu vezu, dokazaćemo tvrdenje-
-
(25) indukcijom po n. Ono je očigledno za n = 1, pa podimo od pretpostavke da
je ono tačno za sve prirodne brojeve manje od neke odabrane vrednosti n.
Razložimo n = 2k q, gde je q neparan broj. Pošto za sve k 6 ai važi 2k | 2ai =
ai+1 , to iz činjenice da niz ai nije ograničen sledi da postoji jk tako da 2k | ai | bi
za sve i > jk . S druge strane, želimo da q | bi za dovoljno veliko i, što je moguće
samo ukoliko q | (2bi−1 − 1). Po Ojlerovoj teoremi je

2ϕ(q) ≡ 1(mod q),


-
pa bi iz ϕ(q) | bi−1 sledio željeni zaključak. Medutim, po induktivnoj pretpostavci
(koju možemo da primenimo budući da je ϕ(q) < q 6 n), ova deljivost je tačna za
sve i za koje je i − 1 > iϕ(q) . Dakle, za sve i > iϕ(q) + 1 važi q | (2bi−1 − 1) | bi .
Kako je (2k , q) = 1, sada za sve i > max(jk , iϕ(q) + 1) imamo n = 2k q | bi , što
okončava induktivni dokaz.

94
59. MMO, 1990. (Rumunija)

Naći sve prirodne brojeve n za koje n2 | (2n + 1).

Rešenje. Kako je 2n + 1 neparan broj, to n mora biti takode - neparan. Pišimo


k
n = 3 d, gde je d neparan broj koji nije deljiv sa 3. Početna ideja je da analiziramo
stepen trojke koji deli broj 2n + 1. Najpre, imamo faktorizaciju
k k
h k k
i
2n + 1 = (23 )d + 1 = (23 + 1) (23 )d−1 + . . . + 23 + 1 .

k
Budući da je (23 )m ≡ (−1)m (mod 3), i kako u uglastoj zagradi imamo neparno
mnogo sabiraka, broj u toj zagradi nije deljiv sa 3. Prema tome, stepen kojim 3 deli
k
2n + 1 je jednak stepenu kojim 3 deli 23 + 1.
k
Izraz 23 + 1 možemo faktorisati uzastopnom primenom formule za zbir kubo-
va:
k k−1 k−1
23 + 1 = (2 + 1)(22 − 21 + 1) . . . (22·3 − 23 + 1). (26)
Sada želimo da ustanovimo stepen trojke u broju oblika 22q − 2q + 1, gde je q
neparan broj. Lako se vidi da je on deljiv sa 3, pa ćemo posmatrati njegov ostatak
pri deljenju sa 9. Pošto je 26 ≡ 1(mod 9), pisaćemo q u obliku q = 6s + r,
r ∈ {1, 3, 5}, jer je tada

2q = (26 )s 2r ≡ 2r (mod 9).

Stoga je 22q − 2q + 1 ≡ 22r − 2r + 1(mod 9), pa se direktnom proverom zaključuje


da je uvek
22q − 2q + 1 ≡ 3(mod 9).
Dakle, svaki od brojeva u zagradama na desnoj strani razlaganja (26) je deljiv sa
k
3, ali ne i sa 9. Otuda je 23 + 1 (a time i 2n + 1) deljiv sa 3k+1 , ali ne i sa 3k+2 .
Ali, po uslovu zadatka, 2n + 1 treba da je deljivo sa n2 , pa tako i sa 32k . Zato je
2k 6 k + 1, tj. k ∈ {0, 1}.
Sada prelazimo na razmatranje broja d. Ako je d = 1, tada je n ∈ {1, 3} i lako
uočavamo da oba ova broja predstavljaju rešenje zadatka. Zbog toga, pretpostavimo
da je d > 1. Neka je p najmanji prost faktor broja d. Jasno, mora biti p > 5.
Takode,- imamo da je
2n ≡ −1(mod p), (27)
pa važi 22n ≡ 1(mod p). S druge strane, po maloj Fermaovoj teoremi je 2p−1 ≡
1(mod p), odakle na potpuno analogan način kao u zadatku br. 57 (b) dobijamo
-
2` ≡ 1(mod p), gde je ` = (2n, p − 1). Medutim, svi prosti faktori broja p − 1 su
očito manji od p, najmanjeg prostog faktora od d, pa sledi (d, p − 1) = 1. Otuda je

95
` = (2 · 3k , p − 1), gde je k ∈ {0, 1}, tj. ` ∈ {1, 2, 3, 6}. Pošto p | (2` − 1), p deli
jedan od brojeva 1, 3, 7, 63. Zbog ograničenja p > 5, jedina mogućnost je p = 7.
Ali, lako se proverava da stepen dvojke pri deljenju sa 7 može davati isključivo
ostatke 1, 2, 4, što je kontradikcija sa (27).
Prema tome, jedini brojevi sa traženom osobinom su n = 1 i n = 3.

Komentar. Primetimo da je prvi deo gornjeg rešenja gotovo analogan idejama koji
se pojavljuju u rešenju zadatka br. 2 (b), a kojeg je Rumunija predložila za MMO
godinu dana ranije. Može se sa velikom sigurnošću reći da ova dva zadatka potiču
od istog autora.

96
60. BMO, 1989. (Bugarska)

Naći sve prirodne brojeve n za koje je

d21 + d22 + d23 + d24 = n,

gde su 1 = d1 < d2 < . . . < dk = n (k > 4) svi pozitivni delioci broja n.

Rešenje. Ako bi bilo d2 > 2, tada bi n bio neparan broj, kao i svi njegovi delitelji
di , ali bi tada broj d21 + d22 + d23 + d24 bio paran, što je kontradikcija. Zaključujemo
da je d2 = 2, pa je tačno jedan od brojeva d3 , d4 paran.
Ako bi d3 bio paran, d3 = 2a, sledilo bi a | n i 1 < a < d3 , pa je to moguće
samo ako je a = 2, tj. d3 = 4. Tada je n = 4m, pa je 4m = 12 + 22 + 42 + d24 ,
odnosno d24 ≡ −1(mod 4), što je nemoguće.
Prema tome, d3 je neparan broj, a d4 je paran, d4 = 2b, pri čemu je b > 1.
Kako tada b | n, to je b = 2 ili b = d3 . Medutim, - prvi slučaj je nemoguć, jer je
tada d4 = 4, odakle je d3 = 3 i n = 1 + 2 + 32 + 42 = 30, što nije deljivo sa 4.
2 2

Dakle, d3 = b, pa važi n = 2bm za neki prirodan broj m. Dobijamo relaciju

5(b2 + 1) = 2bm,

pa mora biti b = 5. Otuda je d4 = 10, n = 130, što predstavlja jedino rešenje


zadatka.

97
61. S.S.S.R., 1990. (republička olimpijada)

Rešiti u skupu celih brojeva:

x5 − x3 − x2 + 1 = y 2 .

Rešenje. Faktorizacijom leve strane dobijamo

(x − 1)2 (x + 1)(x2 + x + 1) = y 2 .

Ako je x = 1, tada je y = 0. U slučaju x 6= 1, gornju jednačinu transformišemo u


oblik µ ¶2
2 y
(x + 1)(x + x + 1) = .
x−1
Dakle, (x − 1) | y. Obeležimo A = x + 1, B = x2 + x + 1. Budući da je
¡ ¢2
B − xA = 1, sledi (A, B) = 1. Kako je B = x + 21 + 34 > 0, A i B su
potpuni kvadrati. No, za x > 1 je x2 < x2 + x + 1 < (x + 1)2 , a za x 6 −2 je
x2 > x2 + x + 1 > (x + 1)2 , pa zato B ne može biti kvadrat celog broja. Preostaju
slučajevi x = −1 i x = 0 iz kojih redom dobijamo y = 0, odnosno y = ±1.

98
62. -
Madarska, 1990.

Da li jednačina
x2 + xy + y 2 = 2
ima racionalna rešenja?

-
Rešenje. Podimo od pretpostavke da data jednačina ima racionalno rešenje (x, y).
Tada možemo pisati x = ac i y = cb , pri čemu ove razlomke ne možemo skratiti
istim prirodnim brojem: drugim rečima, (a, b, c) = 1 (pri čemu je a, b, c ∈ Z).
Dobijamo:
a2 + ab + b2 = 2c2 .
Iz gornje jednakosti sledi da oba broja a, b moraju biti parna (u suprotnom bi leva
strana sadržala jedan ili tri neparna sabirka). Zbog toga, c je neparan broj (u suprot-
-
nom bi bilo 2 | (a, b, c)). Medutim, sada je leva strana gornje jednakosti deljiva sa
4, dok je desna strana deljiva sa 2, ali ne i sa 4. Kontradikcija. Dakle, odgovor na
postavljeno pitanje je negativan.

Komentar. Isti rezultat sledi i za jednačinu x2 + nxy + y 2 = 2, gde je n bilo koji


neparan ceo broj.

99
63. S.A.D., 1976.

Naći sva celobrojna rešenja jednačine

a2 + b2 + c2 = a2 b2 .

Rešenje. Očigledno, a = b = c = 0 jeste rešenje date jednačine. Stoga pretpos-


tavimo da je bar jedan od brojeva a, b, c različit od 0 i definišimo d = (a, b, c). Tada
je a = a1 d, b = b1 d i c = c1 d za neke a1 , b1 , c1 ∈ Z takve da je (a1 , b1 , c1 ) = 1.
Uvrštavajući ovo u polaznu jednačinu, sledi

a21 + b21 + c21 = a21 b21 d2 .

Desna strana (budući da je u pitanju potpun kvadrat) daje ostatak 0 ili 1 pri deljenju
sa 4. Pri tome, taj ostatak može biti 1 samo ako su a1 i b1 (a i d) neparni brojevi.
-
Medutim, tada je ostatak koji a21 + b21 + c21 daje pri deljenju sa 4 jednak 2 ili 3, pa
odmah dobijamo kontradikciju. S druge strane, ako je razmatrani ostatak 0, tada je

a21 + b21 + c21 ≡ 0(mod 4),

što je moguće samo ako su a1 , b1 , c1 parni. Ali, to je u suprotnosti sa pretpostavkom


(a1 , b1 , c1 ) = 1, pa je trivijalno ujedno i jedino rešenje posmatrane jednačine.

100
64. Predlog za MMO, 1982. (Belgija)

Naći sva celobrojna rešenja jednačine

x3 − y 3 = 2xy + 8.

Rešenje. Posmatrajmo najpre trivijalne slučajeve: za x = 0 očito dobijamo y =


−2, dok za y = 0 imamo x = 2. Zato sada možemo pretpostaviti da su x, y celi
brojevi različiti od 0.
Diskutujmo sada znak brojeva x, y. Ako je x > 0, a y < 0, tada imamo
x3 = y 3 + 2xy + 8 < 8, pa mora biti x = 1 i y 3 + 2y + 7 = 0, što ne zadovoljava
nijedan ceo broj y. Ako je x < 0 i y > 0, tada je, s jedne strane, y 3 − x3 =
−2xy − 8 < −2xy, a s druge strane:

y 3 − x3 = y 3 + (−x)3 > y 2 + (−x)2 > −2xy,


- nemoguće.
što je takode
Dakle, x, y su istog znaka, xy > 0. Tada je

0 < 2xy + 8 = x3 − y 3 = (x − y)[(x − y)2 + 3xy],

pa kako je faktor u uglastoj zagradi pozitivan, to mora biti x > y. Sada ćemo
razmotriti moguće vrednosti za x − y. Ukoliko bi bilo x − y > 2, tada bismo imali

2xy + 8 > 2(4 + 3xy) = 6xy + 8,

što je nemoguće za xy > 0. Jedina mogućnost je, zbog toga, x − y = 1, odakle je


2xy + 8 = 3xy + 1, tj. x(x − 1) = 7, što ne važi ni za jedno x ∈ Z.
Stoga su na početku uočena trivijalna rešenja (0, −2) i (2, 0) ujedno i jedina.

101
65. MMT, Mersch, Luksemburg, 1980. (Holandija)

Naći sva celobrojna rešenja jednačine

x3 + x2 y + xy 2 + y 3 = 8(x2 + xy + y 2 + 1).

Rešenje. Faktorizacijom leve strane i grupisanjem na desnoj strani, dobijamo:

(x + y)(x2 + y 2 ) = 4(x2 + y 2 ) + 4(x + y)2 + 8. (28)

Pošto x = y = 0 nije rešenje posmatrane jednačine, možemo pisati

(x + y)2 8
x+y =4+4 + 2 , (29)
x2 + y 2 x + y2
odakle sledi da je x + y > 4, tj. x + y > 5. S druge strane, nejednakost kvadratne
i aritmetičke sredine daje x2 + y 2 > 12 (|x| + |y|)2 > 12 (x + y)2 , pa je x2 + y 2 >
25
2 > 8, kao i
(x + y)2
06 2 6 2.
x + y2
Uzimajući u obzir ove nejednakosti, iz (29) zaključujemo da je x + y < 13, tj.
x + y 6 12. Osim toga, pošto su brojevi x + y i x2 + y 2 iste parnosti, iz (28) odmah
sledi da je x + y parno. Prema tome, x + y ∈ {6, 8, 10, 12}.
Ako sada, radi kraćeg zapisa, označimo x + y = 2a i xy = b, razmatrana
jednačina postaje
2a(4a2 − 2b) = 8(4a2 − b + 1).
Odavde lako možemo izraziti b:
2a3 − 8a2 − 2 18
b= = 2a2 − 4a − 8 − .
a−2 a−2
Stoga (a − 2) | 18, pa pošto je 3 6 a 6 6, imamo tri slučaja: a ∈ {3, 4, 5}. Ovi
slučajevi za vrednosti b redom daju −20, −1 i 16. Dakle, (x, y) se dobija kao par
rešenja jedne od sledeće tri kvadratne jednačine:

t2 − 6t − 20 = 0, t2 − 8t − 1 = 0, t2 − 10t + 16 = 0.

Diskriminante ovih jednačina su redom 116, 68 i 36, što znači da samo treća
jednačina ima celobrojna rešenja. Ta rešenja su 2 i 8, pa preostaje samo da prove-
rimo da su (2, 8) i (8, 2) zaista rešenja jednačine date u zadatku.

102
66. Bugarska, 1979.

Dokazati da jednačina
x2 + 5 = y 3
nema rešenja u skupu celih brojeva.

Rešenje. Pretpostavimo suprotno. Razmotrimo najpre parnost brojeva x, y. Ako bi


x bilo neparno, tada bismo imali x2 ≡ 1(mod 4), odakle bi sledilo y 3 ≡ 2(mod 4),
što je nemoguće (pošto je y tada paran, pa je y 3 ≡ 0(mod 4)). Dakle, x je paran
broj, pa važi y 3 ≡ 1(mod 4), tj. y ≡ 1(mod 4). Ako sada uvrstimo x = 2u i
y = 4v + 1 u datu jednačinu, sledi

4u2 + 5 = (4v + 1)3 = 64v 3 + 48v 2 + 12v + 1,

odnosno
u2 + 1 = v(16v 2 + 12v + 3).
Očito, 16v 2 + 12v + 3 = 4 · [v(4v + 3)] + 3, pa ovaj broj mora imati prost faktor
p oblika 4t + 3 (u suprotnom bi svi njegovi prosti faktori bili oblika 4t + 1, što bi
značilo da posmatrani broj daje ostatak 1 pri deljenju sa 4, a ne 3). Taj prost broj
deli u2 + 1, pa dobijamo kontradikciju na gotovo identičan način kao u zadatku br.
56 (b): naime, iz u2 ≡ −1(mod p) imamo

up−1 = u4t+2 = (u2 )2t+1 ≡ (−1)2t+1 = −1(mod p),

što je zbog p > 2 u suprotnosti sa malom Fermaovom teoremom.

Komentar. Jednačina data u zadatku je specijalan slučaj tzv. Bašeove jednačine

x2 + k = y 3 ,

gde je k ∈ Z. Ovu jednačinu je prvi posmatrao francuski matematičar Claude


Gaspard Bachet de Méziriac (1581–1638), po kome je dobila ime. Opšta Bašeova
jednačina je i danas predmet aktivnog izučavanja u teoriji brojeva.

103
67. -
MMT, Mariehamn, Finska, 1980. (Madarska)

Neka je n > 2 prirodan broj. Dokazati da jednačina

xn + 1 = y n+1

nema rešenje x, y ∈ N za koje važi (x, n + 1) 6= 1.

Rešenje. Transformišimo jednačinu:

xn = y n+1 − 1 = (y − 1)(y n + . . . + y + 1).

Pri tome je, očigledno,

y n + . . . + y + 1 ≡ n + 1(mod y − 1),

odakle sledi da d = (y − 1, y n + . . . + y + 1) deli n + 1.


Ako bi sada bilo d = 1, tada bi oba uzajamno prosta broja y−1 i y n +. . .+y+1
-
bili n-ti stepeni celih brojeva. Medutim,

y n < y n + . . . + y + 1 < (y + 1)n ,

pa dobijamo kontradikciju. Dakle, d > 1.


Ali, ako je p prost delitelj od d, tada p | xn , odakle p | x. Istovremeno,
-
podsetimo se da p | (n + 1). Otuda sledi tvrdenje zadatka.

104
68. S.S.S.R., 1990. (republička olimpijada)

Naći sve prirodne brojeve x, y za koje važi:

7x − 3 · 2y = 1.

Rešenje. Data jednačina je ekvivalentna sa:


7x − 1
2y−1 = = 7x−1 + 7x−2 + . . . + 1.
7−1
Otuda je x = 1, y = 1, jedno od njenih rešenja, pa pretpostavimo sada da je y > 2.
Tada na levoj strani gornje jednačine stoji paran broj, a na desnoj zbir x neparnih
brojeva, odakle je x parno. To znači da možemo faktorisati:

2y−1 = (7 + 1)(7x−2 + 7x−4 + . . . + 1),

odnosno
2y−4 = 7x−2 + 7x−4 + . . . + 1,
odakle sledi da mora biti y > 4. Odmah uočavamo da je x = 2, y = 4 drugo
rešenje. Zato pretpostavimo da je y > 5. Tada je zbir x2 neparnih brojeva jednak
parnom broju, pa je x2 paran broj, tj. x je deljivo sa 4. Sada možemo pisati:

2y−4 = (72 + 1)(7x−4 + 7x−8 + . . . + 1),

što je zbog 72 + 1 = 50 nemoguće. Dakle, {(1, 1), (2, 4)} predstavlja traženi skup
rešenja.

105
69. Predlog za MMO, 1991. (Hong Kong)

Naći sve prirodne brojeve x, y, z za koje važi:

3x + 4y = 5z .

Rešenje. Posmatrajmo najpre datu jednačinu po modulu 3. Tada sledi

2z ≡ 5z ≡ 1(mod 3),

pa z mora biti paran broj, z = 2t. Stoga, važi

3x = 52t − 4y = (5t − 2y )(5t + 2y ),

odakle dobijamo da je 5t + 2y stepen trojke, dok je 5t − 2y ili jednak 1, ili stepen


-
trojke. Medutim, drugi od ovih slucajeva je očito nemoguć, pošto bi tada broj

(5t + 2y ) + (5t − 2y ) = 2 · 5t

bio deljiv sa 3. Prema tome, 5t −2y = 1 i 5t +2y = 3x . Posmatrajući ove jednačine


po modulu 3, imamo (−1)t − (−1)y ≡ 1(mod 3) i (−1)t + (−1)y ≡ 0(mod 3),
što znači da je t neparan, dok je y paran. Pišimo y = 2v, v ∈ N.
Dakle, imamo jednačine 5t − 4v = 1, 5t + 4v = 3x . Iz prve jednačine sledi
5 = 4v + 1, što odmah povlači da je v = 1, jer bi u suprotnom (v > 2) bilo
t

5t ≡ 1(mod 8), što je nemoguće, budući da je t neparno (naime, 52s+1 = 5 · 25s ≡


5(mod 8)). Pošto je v = 1, to je i t = 1, pa iz druge od dve navedene jednačine
imamo x = 2. Znači, x = y = z = 2 je jedino rešenje zadatka.

106
70. Predlog za MMO, 1978. ((a): Velika Britanija; (b): Holandija)

(a) Prirodni brojevi x, y su takvi da je broj

x2 + y 2
x+y
ceo i deli 1978. Dokazati da je x = y.
(b) Dokazati da na kružnici opisanoj oko kvadrata sa temenima (0, 0), (1978, 0),
(1978, 1978), (0, 1978), nema celobrojnih tačaka, sem navedenih.

Rešenje. (a) Očigledno, zadatak se sastoji u diskutovanju jednačine

x2 + y 2 = m(x + y), (30)

gde m | 1978. Naš cilj je da pokažemo da za posmatrane vrednosti m, ona ima


jedinstveno rešenje x = y = m. Primetimo da se nakon množenja sa 4, prebaci-
vanja članova na levu stranu i dodavanja 2m2 , (30) može pisati u obliku

(2x − m)2 + (2y − m)2 = 2m2 .

Pošto su transformacije bile ekvivalentne, zadatak se sastoji u tome da se pokaže


da se broj oblika 2m2 (za posmatrane vrednosti m) može prikazati kao zbir dva
kvadrata na samo jedan način (naime, kao m2 + m2 ).
Za m = 1, ovo tvrdenje - je očigledno. U suprotnom, ako je m > 1, pret-
postavimo da se 2m2 može predstaviti kao zbir dva kvadrata na neki drugi način
sem m2 + m2 . Na primer, neka je 2m2 = a2 + b2 , pri čemu je a < b. Tada je
a < m, pa s obzirom na to da m | 1978 = 2 · 23 · 43 (zbog čega je m proizvod
različitih prostih brojeva), postoji prost faktor p od m koji ne deli a.
-
Medutim, p | (a2 + b2 ). Po izboru p, važi (a, p) = 1, pa postoji prirodan broj
u tako da je au ≡ 1(mod p). S druge strane, p | ((au)2 + (bu)2 ), odakle je

(bu)2 ≡ −(au)2 ≡ −1(mod p).

Sada rezonujemo analogno kao i u zadacima br. 56 (b) i 66: stepenujući gornju
kongruenciju sa p−1
2 (pod pretpostavkom da je p > 2) i koristeći malu Fermaovu
teoremu, sledi
p−1 p−1
1 ≡ (bu)p−1 = ((bu)2 ) 2 ≡ (−1) 2 (mod p),

odakle je p ≡ 1(mod 4). Ali, 1978 = 2 · 23 · 43 nema takvih prostih faktora, pa


ih samim tim ne može imati ni m. Prema tome, preostaje jedino mogućnost p = 2,

107
dok za svaki drugi prost faktor q | m važi q | a i q | b. Skraćivanjem jednačine
a2 + b2 = 2m2 sa svim takvim prostim faktorima q, dobijamo jednačinu oblika
c2 + d2 = 8. Jedino njeno rešenje je c = d = 2, tako da i u ovom slučaju sledi
željeni zaključak a = b = m, koji okončava rešenje zadatka pod (a).
(b) Jednačina date kružnice je

(x − 989)2 + (y − 989)2 = 2 · 9892 .

Očito, ako je (x, y) celobrojna tačka na ovoj kružnici, onda je to i tačka

(1978 − x, 1978 − y),


-
koja je, primetimo, dijametralno suprotna tački (x, y). Medutim, kako polukružni
luk zahvaćen tačkama (0, 1978) i (1978, 0) koji sadrži tačku (1978, 1978) ceo leži
u prvom kvadrantu, gornja jednačina ima rešenje u skupu celih brojeva ako i samo
ako ima rešenje u skupu celih brojeva > 0.
-
Medutim, 989 = 1978/2 = 23·43, što znači da razmatranja iz tačke (a) povlače
da je x − 989 = y − 989 = ±989, što daje upravo četiri tačke navedene u zadatku.
Ove tačke čine dva dijametralno suprotna para, pa sledi da su to i jedine celobrojne
tačke na posmatranoj kružnici, kao što se i tražilo.

108
71. Predlog za MMO, 1978. (Francuska)

(a) Neka za prirodne brojeve x, y, z važi xy − z 2 = 1. Dokazati da postoje nene-


gativni celi brojevi a, b, c, d tako da je

x = a2 + b2 , y = c2 + d2 , z = ac + bd.

(b) Dokazati: ako je p prost broj i p ≡ 1(mod 4), tada se p može predstaviti kao
zbir dva kvadrata prirodnih brojeva.
-
Rešenje. (a) Za uredenu trojku prirodnih brojeva (x, y, z) za koju je xy − z 2 = 1
kažemo da je loša ako je x 6 y i ne postoje prirodni brojevi a, b, c, d tako da je
x = a2 + b2 , y = c2 + d2 i z = ac + bd. U ovoj terminologiji, zadatak traži da se
pokaže da loše trojke ne postoje.
Pretpostavimo suprotno: neka je (x, y, z) neka loša trojka. Kako je xy = z 2 +1
i x 6 y, zaključujemo da važi

x2 6 xy = z 2 + 1,

odakle je x 6 z (jer je z 2 + 1 < (z + 1)2 ). Ako bi bilo x = z, imali bismo


-
1 = xy − x2 = x(y − x), što je moguće samo ukoliko je x = 1, y = 2. Medutim,
2 2 2 2
tada imamo x = 0 + 1 , y = 1 + 1 i z = 0 · 1 + 1 · 1, što je kontradikcija sa
pretpostavkom da je posmatrana trojka loša. Prema tome, mora biti x < z.
Neka je sada t = z − x, tj. z = x + t. Uvrštavajući ovo u polaznu jednačinu,
imamo:

1 = xy − (x + t)2 = xy − x2 − 2xt − t2 = x(y − x − 2t) − t2 .

Dakle, trojka celih brojeva (x, y − x − 2t, t) = (x, x + y − 2z, z − x) je takode-


2
rešenje jednačine xy − z = 1. Pošto je ova jednačina simetrična po x, y, prve
dve komponente svakog rešenja se mogu slobodno permutovati. Zbog toga, bez
umanjenja opštosti možemo pretpostaviti da je x 6 x + y − 2z (u suprotnom
izvršimo transpoziciju prva dva broja u trojci). Osim toga, posmatrana trojka se
sastoji od prirodnih brojeva. Naime, već smo videli da mora biti z − x > 0. Dalje,
važi µ ¶
2 x+y 2
z = xy − 1 < xy 6 ,
2
pa je 2z < x + y, tj. x + y − 2z > 0.
Sada pokazujemo da je ovako dobijena trojka takode - loša. Pretpostavimo sup-
rotno. Tada postoje prirodni brojevi m, n, p, q tako da važi:

x = m2 + n 2 , x + y − 2z = p2 + q 2 , z − x = mp + nq.

109
Iz ovog sistema dobijamo

z = m(m + p) + n(n + q),

i zatim i
y = (m + p)2 + (n + q)2 ,
kontradikcija sa pretpostavkom da je (x, y, z) bila loša trojka. Prema tome, i

(x, x + y − 2z, z − x)

je loša trojka.
-
Medutim, važi z − x < z. Znači, ukoliko bismo pretpostavili da skup svih loših
trojki nije prazan, sledio bi zaključak da medu - svim takvim trojkama (x, y, z) ne
postoji trojka sa minimalnim z (jer se za svaku lošu trojku može konstruisati loša
trojka sa manjom trećom komponentom). To je, jasno, nemoguće, pa loših trojki
-
zapravo i nema, tj. tvrdenje pod (a) je dokazano.
(b) Neka je sada p = 4k + 1 prost broj. Po Vilsonovoj teoremi, tada važi

(4k)! ≡ −1(mod p).

Ali, važi i p−i ≡ −i(mod p) za 1 6 i 6 2k, pa množenjem svih ovih kongruencija


sledi
(4k)!
≡ (−1)2k (2k)! (mod p),
(2k)!
odakle je
[(2k)!]2 ≡ (4k)! ≡ −1(mod p).
Ako označimo u = (2k)!, dobijamo da p | (u2 + 1), odnosno da važi

pv − u2 = 1
-
za neki prirodan broj v. Primenom tvrdenja (a) na ovaj specijalan slučaj, dobijamo
-
tvrdenje (b).

110
72. MMO, 1977. (SR Nemačka)

Neka su a, b prirodni brojevi, i neka se pri deljenju a2 + b2 sa a + b dobija količnik


q i ostatak r. Naći sve parove (a, b) za koje je q 2 + r = 1977.

Rešenje. Brojevi a, b, q, r treba da zadovolje sledeće uslove:

a2 + b2 = (a + b)q + r, 0 6 r < a + b, q 2 + r = 1977.

Očito, q 2 6 1977, pa je q 6 44. Otuda je

a2 + b2 < 44(a + b) + a + b = 45(a + b),

što zajedno sa nejednakošću (a + b)2 6 2(a2 + b2 ) daje

a + b < 90,
-
pa je r < 90. Odatle je, medutim, q 2 = 1977−r > 1887, tj. q > 43. Zaključujemo
-
da je q = 44 i r = 41, pa preostaje da nademo sva celobrojna rešenja jednačine

a2 + b2 = 44(a + b) + 41.

Nju možemo transformisati u oblik

(a − 22)2 + (b − 22)2 = 1009.

Odredimo sada celobrojna rešenja jedačine x2 + y 2 = 1009, tako da je 0 6 x 6 y.


Kako mora biti y 2 6 1009 6 2y 2 , sledi 23 6 y 6 31. Otuda direktnom proverom
-
dobijamo kao jedino rešenje y = 28, x = 15. Brojeve a, b odredujemo iz uslova

{|a − 22|, |b − 22|} = {15, 28},

pa je traženi skup rešenja

{(7, 50), (37, 50), (50, 7), (50, 37)}.

111
73. Velika Britanija, 1991.

Dokazati: ako su a i b prirodni brojevi i a2 + b2 − a je deljivo sa 2ab, tada je a


potpun kvadrat.

Rešenje. Uslov zadatka možemo zapisati u vidu jednačine

a2 + b2 − a = 2qab,

sa parametrom q ∈ N. Neka je d = (a, b), a = rd, b = sd. Nakon uvrštavanja u


gornju jednačinu i skraćivanja sa d, dobijamo

r2 d + s2 d − r = 2qrsd,

što znači da d | r. Ali, s druge strane r | s2 d, pa zbog (r, s) = 1 sledi r | d. Dakle,


r = d i a = r2 .

Komentar. Kao uopštenje ovog zadatka, možemo postaviti problem nalaženja svih
parova prirodnih brojeva (a, b) tako da za dato q ∈ N važi

a2 + b2 − a
= q.
2ab
Kao što smo videli u gornjem rešenju, tada je a = r2 , b = rs, gde je (r, s) rešenje
jednačine
x2 − 2qxy + y 2 = 1. (31)
Pošto će se jednačine veoma sličnog tipa pojaviti i u sledećim zadacima, u nared-
nom ćemo izložiti jedno opšte teorijsko razmatranje na osnovu kojeg ćemo rešiti
gornju jednačinu, a koje ćemo i kasnije koristiti.

∗ ∗ ∗

Razmatraćemo kvadratne diofantske jednačine oblika

x2 − mxy + y 2 = n, (32)

gde su m, n celi brojevi, m > 0, pri čemu jednačinu rešavamo u N. Jedna od


temeljnih ideja koja se pojavljuje u analizi diofantskih jednačina jeste invarijantnost
njihovih skupova rešenja u odnosu na odredene - transformacije u R2 (tj. u Z2 ). Ne
ulazeći dublje u teoriju kvadratnih formi, nije redak slučaj da su skupovi rešenja
-
diofantskih jednačina invarijantni na odredene linearne transformacije, tako da iz

112
pretpostavke da je (x, y) rešenje sledi da je rešenje i (ax + by, cx + dy) za pogodno
odabrane koeficijente a, b, c, d.
Pošto želimo da primenimo (Fermaov) metod beskonačnog silaska, posmat-
raćemo preslikavanja kod kojih je a = 0 i b = 1, tj. kada je u pitanju transformacija
(x, y) 7→ (y, cx + dy). Naime, ako je x2 − mxy + y 2 = n, tada je
y 2 − my(cx + dy) + (cx + dy)2 =
= n + (c2 − 1)x2 + (m − cm + 2cd)xy + (d2 − dm)y 2 .
Očigledno, uslovi c2 − 1 = m − cm + 2cd = d2 − dm = 0 daju koeficijente c, d
sa željenim osobinama. Dakle, c = ±1; pri tome, c = 1 daje d = 0, i taj slučaj
nam nije interesantan, pošto daje transpoziciju para (x, y). Stoga biramo c = −1,
odakle lako dobijamo d = m. Tako je skup celobrojnih rešenja jednačine (32)
invarijantan na transformaciju

g : (x, y) 7→ (y, my − x).

Imajući u vidu simetričnost jednačine (32), možemo se ograničiti samo na


rešenja kod kojih je x > y > 0 (moguća su i rešenja kod kojih je x = y, ali
n
samo ukoliko je m 6= 2 i 2−m je potpun kvadrat, ili je m = 2 i n = 0). Zbog
toga, od značaja je da ustanovimo kada će rešenje oblika g(x, y) imati prvu kom-
ponentu veću od druge. Očito, ovaj uslov je izražen nejednakošću y > my − x, tj.
(m − 1)y < x. S druge strane, komponente rešenja g(x, y) treba da budu prirodni
brojevi, pa zato imamo nejednakost my−x > 0, odnosno x < my. Dakle, možemo
rezimirati na sledeći način: ako je (x, y) prirodno rešenje jednačine (32) kod kojeg
je x > y i ako pri tome važe nejednakosti

(m − 1)y < x < my, (33)

tada je i g(x, y) = (x0 , y 0 ) prirodno rešenje od (32) kod kojeg je x0 > y 0 . Pri
tome, imamo x0 < x. Na ovaj način, definisan je jedan iterativni postupak za
dobijanje novih rešenja jednačine (32). On će se zaustaviti nakon konačno mnogo
koraka, kada dobijemo ”minimalno” rešenje (x0 , y0 ) kod kojeg ne važi jedna od
nejednakosti (33), tj. kod kojeg je ili x0 6 (m − 1)y0 , ili x0 > my0 .
Sada razlikujemo dva slučaja. Ako je n > 0, tada za proizvoljno rešenje (x, y)
od (32) imamo

x(my − x) = mxy − x2 = y 2 − n 6 y 2 < xy,

tj. (m − 1)y < x. Tako je minimalnost rešenja u ovom slučaju ekvivalentna sa


x0 > my0 . Pri tome, važi

0 > x0 (my0 − x0 ) = y02 − n,

113

odakle je y0 6 n. To zapravo znači da minimalnih rešenja ima konačno mnogo,
jer za fiksirano y, (32) postaje kvadratna jednačina po x, zbog čega za sve y0 ∈

{1, . . . , b nc} imamo najviše dva ”kandidata” za x0 tako da je (x0 , y0 ) minimalno
rešenje. Stoga se minimalna rešenja mogu odrediti (u odsustvu boljeg metoda)
direktnom proverom.
S druge strane, ako je n < 0, tada (32) povlači

x(my − x) = y 2 − n > 0.

Dakle, svako rešenje (32) zadovoljava x < my, pa je minimalnost rešenja ekviva-
lentna sa x0 6 (m − 1)y0 (pri čemu odmah možemo pretpostaviti da je m > 2,
¡ ¢2
jer za m = 1 imamo x2 − xy + y 2 = x + y2 + 43 y 2 > 0, dok za m = 2 važi
x2 − 2xy + y 2 = (x − y)2 > 0). Nejednakost x0 6 (m − 1)y0 je dalje ekvivalentna
sa
x0 (my0 − x0 ) > x0 y0 ,
pa iz x0 (my0 − x0 ) = y02 + |n| dobijamo y0 (x0 − y0 ) 6 |n|. Odavde je ili x0 = y0
n
(u slučaju da je 2−m potpun kvadrat), ili y0 6 |n|. Poslednja nejednakost, analogno
kao i u slučaju n > 0, garantuje da imamo konačno mnogo minimalnih rešenja i
pruža mogućnost da se ona odrede neposrednom proverom.
Prema tome, ukoliko su nam poznata sva minimalna rešenja (x0 , y0 ), tada su
(uz pretpostavku x > y) sva druga rešenja jednačine (32) oblika f k (x0 , y0 ), k ∈ N,
gde je f inverzno preslikavanje od g,

f : (x, y) 7→ (mx − y, x),

budući da za svako rešenje (x, y), x > y, g k (x, y) mora biti minimalno rešenje za
dovoljno veliko k.

∗ ∗ ∗

U jednačini (31) koju razmatramo, parametri su m = 2q i n = 1. Na os-


-
novu malopredašnjih opštih zaključaka, sledi da su sva rešenja te jednačine ob-
k
lika f (x0 , y0 ), k ∈ N0 , gde je (x0 , y0 ) neko minimalno rešenje, a f (x, y) =
(2qx − y, x). Budući da je n > 0, minimalna rešenja su odredena - uslovom
x0 > 2qy0 , odakle je y0 6 1. Znači, y0 = 1 i x20 − 2qx0 = 0 ⇒ x0 = 2q
predstavlja jedino minimalno rešenje, pa su stoga sva rešenja jednačine (31) kod
kojih je x > y oblika f k (2q, 1), k > 0. Drugim rečima, sva tražena rešenja su
iscrpljena parovima uzastopnih članova niza ak , k ∈ N, gde je

ak+2 = 2qak+1 − ak

za k > 1 i a1 = 1, a2 = 2q.

114
74. Vijetnam, 1992.

Naći sva rešenja jednačine

x2 − 5xy + y 2 + 5 = 0

u skupu prirodnih brojeva.

Rešenje. U ovom zadatku je zapravo data jednačina (32) iz komentara prethodnog


zadatka u specijalnom slučaju m = 5, n = −5. To odmah znači da ona nema
rešenje za koje je x = y, pa se možemo ograničiti na rešenja kod kojih je x > y.
Kako je n < 0, na osnovu razmatranja iz komentara prethodnog zadatka ima-
mo da je minimalnost rešenja (x0 , y0 ) ekvivalentna sa x0 6 4y0 , što implicira
y0 (x0 −y0 ) 6 5 i y0 6 5. Slučaj y0 = 1 daje kvadratnu jednačinu x20 −5x0 +6 = 0,
odnosno minimalna rešenja (2, 1) i (3, 1). S druge strane, ako je y0 > 2, tada je
x0 > 3, pa dobijamo

(x0 − y0 )2 + 5 = 3x0 y0 > 18,

odakle sledi x0 − y0 > 4 i y0 (x0 − y0 ) > 8, što je nemoguće. Znači, prethodna dva
minimalna rešenja su i jedina, zbog čega je skup svih rešenja razmatrane jednačine
(uz ograničenje x > y):

{f k (2, 1), f k (3, 1) : k ∈ N0 },

gde je f (x, y) = (5x − y, x).

115
75. MMO, 1988. (SR Nemačka)

Dokazati: ako je za neke prirodne brojeve a, b broj


a2 + b2
ab + 1
ceo, tada je on potpun kvadrat.

Rešenje. Pretpostavimo da je
a2 + b2
= k.
ab + 1
Tada slučaj a = b povlači
2a2
k= < 2,
a2 + 1
tj. k = 1, što jeste potpun kvadrat. Pretpostavimo zato, bez umanjenja opštosti, da
je a > b. Tada je par (a, b) jedno rešenje jednačine

x2 − kxy + y 2 = k

u skupu prirodnih brojeva. Medutim- (prema razmatranjima iz komentara zadatka


br. 73), tada su rešenja ove jednačine i g r (a, b) za r = 1, 2, . . . , r0 , gde je

g(x, y) = (y, ky − x),

a g r0 (a, b) = (a0 , b0 ) je neko minimalno rešenje. Pri tome, imamo a0 > kb0 (tj.
a0 −kb0 > 0), jer je k > 0. Naš cilj je da pokažemo da u ovoj nejednakosti mora da
važi jednakost. Zbog toga, transformišemo datu jednačinu tako da se u njoj pojavi
izraz a0 − kb0 :

k = a20 − ka0 b0 + b20 = (a0 − kb0 )2 + ka0 b0 − k 2 b20 + b20 >

> ka0 b0 − k 2 b20 = kb0 (a0 − kb0 ).


Odavde je b0 (a0 − kb0 ) < 1, odnosno a0 − kb0 6 0. Stoga važi a0 = kb0 , pa
uvrštavanjem u razmatranu jednačinu sledi k = b20 . Dakle, k je ponovo potpun
kvadrat, što je i trebalo dokazati.

Komentar. Za k = 1, jedino rešenje je a = b = 1. Ako je k = `2 za neko


prirodno√` > 1, tada se iz gornjeg rešenja vidi da je jedino minimalno rešenje dato
sa b0 = k = ` i a0 = kb0 = `3 . Prema tome, za posmatranu vrednost k, parovi
prirodnih brojeva (a, b), a > b, koji zadovoljavaju uslov zadatka jesu elementi
skupa
{f r (`3 , `) : r ∈ N0 },

116
gde je f (x, y) = (`2 x − y, x), tj. uzastopni članovi niza as , s ∈ N, datog sa a1 = `,
a2 = `3 i
as+2 = `2 as+1 − as
za s > 1.

117
76. MMO, 1982. (Velika Britanija)

(a) Neka je n ∈ N. Ako jednačina

x3 − 3xy 2 + y 3 = n

ima rešenje u skupu prirodnih brojeva, tada ona ima bar tri različita rešenja u
skupu prirodnih brojeva. Dokazati.
(b) Dokazati da gornja jednačina za n = 2891 nema rešenja u skupu prirodnih
brojeva.

Rešenje. (a) Pošto se drugi i treći član leve strane date jednačine pojavljuju u
razvoju kuba razlike (y − x)3 = y 3 − 3xy 2 + 3x2 y − x3 , možemo transformisati

x3 − 3xy 2 + y 3 = (y − x)3 − 3x2 y + 2x3 .

Desna strana gornjeg identiteta po formi prilično podseća na levu, što motiviše dalje
transformacije:

(y − x)3 − 3x2 y + 2x3 = (y − x)3 − 3x2 y + 3x3 − x3 =

= (y − x)3 − 3(y − x)x2 − x3 = (y − x)3 − 3(y − x)(−x)2 + (−x)3 .


Dakle, ako levu stranu polazne jednačine označimo sa f (x, y), dobili smo da važi
f (x, y) = f (y − x, −x), što znači da je za svako rešenje (x, y) date jednačine par
(y − x, −x) takode- rešenje. Ponovo primenjujući ovo razmatranje, dobijamo da je
i (−x − (y − x), −(y − x)) = (−y, x − y) rešenje (ako bismo još jednom primenili
posmatranu transformaciju, vratili bismo se na (x, y)). Prva dva rešenja su jednaka
ako i samo ako je x = y = 0, što je nemoguće zbog n > 1. Slične kontradikcije
dobijamo i iz pretpostavki o jednakosti bilo koja dva od tri navedena rešenja.
(b) Pretpostavimo da za neke cele brojeve x, y važi

x3 − 3xy 2 + y 3 = 2891 = 9 · 321 + 2.

Tada je x3 + y 3 ≡ −1(mod 3), pa imamo tri mogućnosti (kongruencije su po


modulu 3): x ≡ 0, y ≡ −1; x ≡ y ≡ 1; x ≡ −1, y ≡ 0. U prvom slučaju je
x = 3t, y = 3u − 1 i
2891 = x3 − 3xy 2 + y 3 =
= 27t3 − 9t(3u − 1)2 + 27u3 − 27u2 + 9u − 1 ≡ −1(mod 9),
kontradikcija. Analogno postupamo i u trećem slučaju, dok u drugom slučaju
umesto rešenja (x, y) posmatramo pridruženo rešenje (y − x, −x), pa imamo prvi
od navedena tri slučaja, za koji smo već pokazali da je nemoguć.

118
77. Predlog za MMO, 1988. (S.S.S.R.)

Neka su a, b, c celi brojevi različiti od 0. Poznato je da jednačina

ax2 + by 2 + cz 2 = 0

ima celobrojno rešenje različito od x = y = z = 0. Dokazati da jednačina

ax2 + by 2 + cz 2 = 1

ima racionalno rešenje.

Rešenje. Neka su x0 , y0 , z0 celi brojevi takvi da je

ax20 + by02 + cz02 = 0, (34)

pri čemu je bar jedan od njih 6= 0. Primetimo da tada najviše jedan od ovih brojeva
može biti jednak 0. Bez umanjenja opštosti, neka je x0 , z0 6= 0.
Naš cilj je da odredimo racionalne brojeve x, y, z tako da važi

ax2 + by 2 + cz 2 = 1, (35)

Najpre ćemo pomnožiti (34) sa ( zz0 )2 kako bismo eliminisali bar jedan od nepoz-
natih parametara a, b, c (u ovom slučaju c). Ako radi kraćeg zapisa označimo t =
z
z0 , što znači da imamo smenu z = z0 t, dobijamo

ax20 t2 + by02 t2 + cz 2 = 0.

Oduzimajući ovo od (35), imamo

a(x2 − x20 t2 ) + b(y 2 − y02 t2 ) = 1.

Ako sada uspemo da pronademo - bar jedno racionalno rešenje (x, y, t) ove jednači-
ne, i polazni problem će biti rešen. Možemo pokušati da potražimo rešenje za koje
je a(x2 − x20 t2 ) = 1 i b(y 2 − y02 t2 ) = 0. Drugi uslov će biti ispunjen ako je npr.
y = y0 t. S druge strane, prva jednačina se može pisati kao

a(x − x0 t)(x + x0 t) = 1. (36)


-
Medutim, ako su sada r, s proizvoljni racionalni brojevi takvi da je rs = a1 , tada će
rešenje sistema linearnih jednačina

x − x0 t = r,
x + x0 t = s,

119
biti istovremeno i rešenje jednačine (36). Ovaj sistem ima determinantu 2x0 6= 0 i
racionalne koeficijente, pa za njegovo rešenje (x, t) mora važiti x, t ∈ Q. Tada su i
- racionalni brojevi. Iz prethodnih razmatranja se lako vidi
y = y0 t i z = z0 t takode
-
da ovako odredeni brojevi x, y, z zaista predstavljaju rešenje od (35), pa je zadatak
rešen.

Komentar. Na primer, ako u gornjem rešenju uzmemo r = 1, s = a1 , dobićemo

1+a
x=
2a
i
1−a
t= ,
2ax0
odakle je
(1 − a)y0
y=
2ax0
i
(1 − a)z0
z= .
2ax0

120
78. Predlog za MMO, 1989. (Južna Koreja)

Neka su a, b celi brojevi koji nisu potpuni kvadrati. Dokazati: ako jednačina

x2 − ay 2 − bz 2 + abw2 = 0

ima netrivijalno celobrojno rešenje, tada to važi i za jednačinu

x2 − ay 2 − bz 2 = 0.

- a, b ne mogu
Rešenje. Najpre, a, b 6= 0, jer a, b nisu potpuni kvadrati. Takode,
biti istovremeno negativni, pa pretpostavimo, bez umanjenja opštosti, da je a > 0.
Neka je (X, Y, Z, W ) netrivijalno rešenje jednačine

x2 − ay 2 − bz 2 + abw2 = 0.

Ideja se sastoji u tome da, slično kao i u nizu prethodnih zadataka, od posmatranog
rešenja dobijemo (pogodnom transformacijom) celobrojno rešenje za x2 − ay 2 −
bz 2 = 0.
Naravno, dve uočene jednačine se razlikuju u broju članova sa leve strane: prva
ima četiri, a druga tri člana. To sugeriše da izvršimo grupisanje:

X 2 − aY 2 − b(Z 2 − aW 2 ) = 0.

”Nedostatak” je sada u tome da izraz u zagradi nije, u opštem slučaju, potpun


-
kvadrat, što se, medutim, može ”popraviti” množenjem gornje jednakosti sa Z 2 −
aW 2 . Tako imamo

(X 2 − aY 2 )(Z 2 − aW 2 ) − b(Z 2 − aW 2 )2 = 0.

S druge strane, važi:

(X 2 − aY 2 )(Z 2 − aW 2 ) = X 2 Z 2 − aY 2 Z 2 − aX 2 W 2 + a2 Y 2 W 2 =
= (X 2 Z 2 + 2aXY ZW + a2 Y 2 W 2 ) − a(Y 2 Z 2 + 2XY ZW + X 2 W 2 ) =
= (XZ + aY W )2 − a(Y Z + XW )2 .
Zbog toga, ako označimo

x0 = XZ + aY W,
y0 = Y Z + XW,
z0 = Z 2 − aW 2 ,

121
dobijamo
x20 − ay02 − bz02 = 0.
Očigledno, x0 , y0 , z0 su celi brojevi. Preostaje da pokažemo da je bar jedan od
njih 6= 0. Tačnije, tvrdimo da je z0 6= 0. U suprotnom, sledilo bi Z 2 = aW 2 .
Pošto a nije potpun kvadrat, zaključili bismo da je Z = W = 0. Ali, tada bismo iz
polazne jednačine dobili X 2 −aY 2 = 0, tj. X 2 = aY 2 , što povlači X = Y = 0, što
je kontradikcija sa pretpostavljenom netrivijalnošću rešenja (X, Y, Z, W ). Dakle,
(x0 , y0 , z0 ) je netrivijalno celobrojno rešenje jednačine x2 − ay 2 − bz 2 = 0.

122
79. Predlog za MMO, 1988. (Grčka)

Naći celobrojno rešenje jednačine

x21 + x22 + . . . + x229 = 29x1 x2 . . . x29

tako da za bar jedno 1 6 k 6 29 važi xk > 19882 .

Rešenje. Kao i u nekoliko prethodnih zadataka, ideja rešenja se sastoji u tome da


se polazeći od nekog rešenja date jednačine konstruiše novo. Uz pogodnu trans-
formaciju, komponente rešenja će biti sve veće, pa ćemo, polazeći od očiglednog
rešenja x1 = . . . = x29 = 1, nakon odgovarajućeg broja iteracija dobiti ”dovoljno
veliko” rešenje.
Naravno, voleli bismo da ta transformacija bude što je moguće jednostavnija.
Naime, pokušaćemo da izmenimo samo jednu komponentu rešenja, odnosno da
-
od (x1 , x2 , . . . , x29 ) predemo na (y, x2 , . . . , x29 ). Kako bi ova druga 29-orka bila
rešenje, potrebno je i dovoljno da bude

y 2 + x22 + . . . + x229 = 29yx2 . . . x29 .

Oduzimajući od ovoga polaznu jednačinu, dobijamo

y 2 − x21 = 29yx2 . . . x29 − 29x1 x2 . . . x29 = 29(y − x1 )x2 . . . x29 ,

Nakon skraćivanja sa y − x1 (što ima smisla, pošto očigledno želimo da bude y 6=


x1 ), imamo y + x1 = 29x2 . . . x29 . Lako se proverava da

y = 29x2 . . . x29 − x1

zaista daje novo rešenje razmatrane jednačine.


Sada treba da obezbedimo da uočena transformacija zaista povećava kompo-
nente rešenja. Ako pretpostavimo da smo na početku imali

x1 6 x2 6 . . . 6 x29 ,

tada je
29x2 . . . x29 − x1 > 29x29 − x29 = 28x29 > x29 , (37)
pa je, imajući u vidu da je razmatrana jednačina simetrična po svim promenljivama,
svrsishodno da posmatramo našu transformaciju rešenja u obliku

(x1 , x2 , . . . , x29 ) → (x2 , . . . , x29 , 29x2 . . . x29 − x1 ),

123
kako bismo očuvali neopadajući poredak i time obezbedili da nejednakost (37) važi
nakon svake primene gornje transformacije.
-
Prema tome, ako podemo od već uočenog trivijalnog rešenja (1, 1, . . . , 1), tada
redom dobijamo sledeća rešenja:

(1, 1, . . . , 1, 28),
(1, 1, . . . , 28, 29 · 28 − 1),
(1, 1, . . . , 28, 29 · 28 − 1, 29 · 28(29 · 28 − 1) − 1),
(1, 1, . . . , 29 · 28(29 · 28 − 1)(29 · 28(29 · 28 − 1) − 1) − 1).
-
Medutim, poslednja komponenta poslednjeg rešenja je očito veća od 19882 .

124
80. Kina, 1991.

Rešiti jednačinu u skupu prirodnih brojeva:

x2n+1 − y 2n+1 = xyz + 22n+1 ,

pri čemu važe ograničenja n > 2 i z 6 5 · 22n .

Rešenje. Iz date jednačine je očigledno da važi x > y > 1. Takode,- primetimo


da x i y moraju biti iste parnosti. Zaista, ako je y parno, tada su svi članovi date
jednačine osim x2n+1 parni, odakle x mora biti parno. Slično, parnost x povlači
parnost y. Zbog toga je x − y > 2 i x > 3.
S obzirom da imamo rastavljanje

x2n+1 − y 2n+1 = (x − y)(x2n + x2n−1 y + . . . + y 2n ),

važe nejednakosti

x2n+1 − y 2n+1 > (x − y)(x2n + x2n−1 y) > (x − y) · 2x2n−1 y > 4x2n−1 y.

S druge strane, iz z 6 5 · 22n sledi

x2n+1 − y 2n+1 6 5 · 22n xy + 22n+1 = 22n (5xy + 2).

Kombinujući ove dve nejednakosti, dobijamo

4x2n−1 y < 22n (5xy + 2),

odnosno, nakon deljenja sa 22n xy,


³ x ´2n−2 2
<5+ < 6.
2 xy
(Druga√nejednakost sledi iz xy > 2.) Kako je 2n − 2 > 2, zaključujemo da je
x < 2 6 < 5, pa su jedine mogućnosti

(x, y) ∈ {(3, 1), (4, 2)}.

Prva od njih povlači nejednakost


µ ¶2n−2
3
< 6,
2
odakle je n 6 3, dok druga daje

22n−2 < 6,

125
što je moguće samo ako je n = 2.
Za x = 3, y = 1, n = 2 dobijamo z = 70, za iste vrednosti x, y i n = 3
imamo z = 686, dok za x = 4, y = 2 sledi z = 120. Pošto za n = 3 ograničenje
dato u zadatku glasi z 6 320, odbacujemo drugo rešenje, a kako za n = 2 mora
biti z 6 80, odbacujemo i treće rešenje. Prema tome, jedino rešenje sa traženim
svojstvima je
(x, y, z) = (3, 1, 70)
i n = 2.

126
81. Predlog za MMO, 1995. (Bugarska)

Naći sve prirodne brojeve x, y za koje važi

x + y 2 + z 3 = xyz,

gde je z = (x, y).

Rešenje. Neka je x = az i y = bz, pri čemu je (a, b) = 1. Data jednačina sada


poprima oblik:
a + b2 z + z 2 = abz 2 .
Otuda je a = cz za neki prirodan broj c, pa imamo jednačinu

c + b2 + z = cbz 2 ,

odakle neposredno nalazimo (pošto je bz 2 6= 1, u suprotnom bi bilo y = b = z = 1,


tj. x + 2 = x)
b2 + z
c= 2 . (38)
bz − 1
Množenjem sa z 2 (koje je očigledno pogodno iz ”tehničkih” razloga) dobijamo
b2 z 2 + z 3 b + z3
z2c = = b + . (39)
bz 2 − 1 bz 2 − 1
Drugi sabirak u gornjoj jednačini mora biti prirodan broj, pa je on > 1. To
je ekvivalentno nejednakosti b + z 3 > bz 2 − 1, tj. b(z 2 − 1) 6 z 3 + 1. Nakon
skraćivanja sa z + 1, sledi

b(z − 1) 6 z 2 − z + 1.

Ako je z = 1, tada iz (39) imamo


b2 + 1 2
c= =b+1+ ,
b−1 b−1
pa je b = 2 ili b = 3, a odgovarajuća rešenja su (x, y) ∈ {(5, 2), (5, 3)}. U
suprotnom, dobijamo
z2 − z + 1 1
b6 =z+ .
z−1 z−1
Za z = 2, jednačina (39) postaje
16b2 + 32 33
16c = = 4b + 1 + ,
4b − 1 4b − 1

127
što daje rešenja b = 1 i b = 3, tj. (x, y) ∈ {(4, 2), (4, 6)}. S druge strane, za z > 3
1
važi z−1 < 1, pa zaključujemo b < z + 1, tj. b 6 z. Uvrštavajući ovu nejednakost
u (38) i koristeći b > 1, sledi

z2 + z z 1
c< 2
= =1+ < 2.
z −1 z−1 z−1
Znači, c = 1. Prema tome, b je rešenje kvadratne jednačine

b2 − z 2 b + (z + 1) = 0,

što povlači da je diskriminanta te jednačine z 4 − 4z − 4 potpun kvadrat celog broja.


Ali, s obzirom na z > 3, važe stroge nejednakosti

(z 2 − 1)2 < z 4 − 4z − 4 < (z 2 )2 ,

i time smo dobili kontradikciju. Dakle, skup rešenja je

{(4, 2), (4, 6), (5, 2), (5, 3)}.

128
82. Predlog za MMO, 1988. (Vijetnam); Jugoslavija, 1990.

Neka je n prirodan broj, A skup koji se sastoji od tačno n + 1 prirodnih brojeva,


a P skup svih prostih faktora elemenata skupa A. Ako je |P | 6 n, dokazati da
postoji B ⊆ A, B 6= ∅, tako da je proizvod elemenata skupa B potpun kvadrat.

Rešenje. Neka je A = {a1 , a2 , . . . , an+1 }. Po uslovima zadatka, postoji n različitih


prostih brojeva p1 , . . . , pn tako da za sve 1 6 i 6 n + 1 važi

ai = pk1i1 . . . pknin

za neke nenegativne cele brojeve kij , 1 6 j 6 n. Prema tome, svaki proizvod


π(B) elemenata nepraznog podskupa B ⊆ A se takode - može napisati u obliku
β1 βn
π(B) = p1 . . . pn za odgovarajuće β1 , . . . , βn > 0.
Jasno, π(B) će biti potpun kvadrat ako i samo ako su svi brojevi βj , 1 6 j 6 n,
parni. Zbog toga, svakom broju oblika ` = pα1 1 . . . pαnn pridružujemo niz elemenata
skupa {0, 1},
e(`) = (e1 , . . . , en ),
tako da je ei = 0 ako je αi parno, a u suprotnom je ei = 1. Sada je ` potpun
kvadrat ako i samo ako se e(`) sastoji od samih nula. Osim toga, ako je e(`1 ) =
(e1 , . . . , en ) i e(`2 ) = (e01 , . . . , e0n ), tada je

e(`1 `2 ) = (e001 , . . . , e00n ),

gde za sve 1 6 j 6 n važi

e00j ≡ ei + e0i (mod 2). (40)

Nepraznih podskupova od A ima 2n+1 − 1. S druge strane, nizova dužine n


koji se sastoje od nula i jedinica ima 2n . Kako je 2n+1 − 1 > 2n za sve n > 1,
po Dirihleovom principu postoje dva različita neprazna podskupa B, C ⊆ A tako
da je e(π(B)) = e(π(C)). Zbog toga, iz (40) sledi da je e(π(B)π(C)) nula-niz,
-
tj. da je π(B)π(C) potpun kvadrat. Medutim, pošto za svaka dva disjunktna skupa
X, Y ⊆ A važi
π(X)π(Y ) = π(X ∪ Y )
(po samoj definiciji oznake π), imamo da je

π(B) = π(B \ C)π(B ∩ C) i π(C) = π(B ∩ C)π(C \ B)

(pri čemu smo po potrebi dodefinisali da je π(∅) = 1). Otuda je

π(B)π(C) = π(B \ C) · [π(B ∩ C)]2 · π(C \ B),

129
zbog čega je
π(B \ C)π(C \ B) = π((B \ C) ∪ (C \ B))
potpun kvadrat. Budući da je B 6= C, skup (B \ C) ∪ (C \ B) je neprazan, čime
smo dobili podskup od A sa traženim osobinama.

130
83. MMO, 1990. (Turska)

Konstruisati funkciju f : Q+ → Q+ (gde Q+ označava skup pozitivnih racionalnih


brojeva) tako da važi
f (x)
f (xf (y)) =
y
za sve x, y ∈ Q+ .

Rešenje. Najpre, funkcija f je injektivna, jer iz pretpostavke f (y1 ) = f (y2 ) i date


funkcionalne jednačine očito sledi y1 = y2 (pošto je f (x) > 0). Ako sada stavimo
y = 1, imamo f (xf (1)) = f (x) za sve x ∈ Q+ , odakle je xf (1) = x i f (1) = 1.
S druge strane, x = 1 daje
1
f (f (y)) = za sve y ∈ Q+ . (41)
y

Primenjujući f na gornju jednakost još jednom, dobijamo f ( y1 ) = 1


f (y) . Najzad,
ako sada u polaznu jednačinu uvrstimo y = f ( z1 ), sledi

f (xz) = f (x)f (z) za sve x, z ∈ Q+ . (42)

Obratno, veoma lako se proverava da svaka funkcija f : Q+ → Q+ koja zado-


voljava uslove (41) i (42) zadovoljava i funkcionalnu jednačinu datu u zadatku.
Imajući u vidu osnovnu teoremu aritmetike, svaki pozitivan racionalan broj
može se zapisati u obliku pα1 1 . . . pαnn , gde su p1 , . . . , pn prosti brojevi i α1 , . . . ,
αn ∈ Z. Zbog (42), mora da važi

f (pα1 1 . . . pαnn ) = f (p1 )α1 . . . f (pn )αn ,


-
pa je tražena funkcija u potpunosti odredena svojim vrednostima na skupu prostih
brojeva. Drugim rečima, ako odredimo f (p) za sve proste brojeve p tako da (nakon
proširivanja f na Q+ putem gornje jednakosti) važi f (f (p)) = p1 , tada se lako
pokazuje da dobijena funkcija zadovoljava uslove (41) i (42). Preostaje da se
primeti da je željeni cilj postignut ako definišemo
(
pi+1 ako je i neparno,
f (pi ) = 1
pi−1 ako je i parno,

gde je pi , i ∈ N, niz prostih brojeva, pošto tada za neparno i važi f (f (pi )) =


f (pi+1 ) = p1i , dok za parno i imamo f (f (pi )) = f (p−1
i−1 ) = f (pi−1 )
−1 = 1 .
pi

131
Tabelarni pregled porekla zadataka

Zemlja porekla MMO pMMO MMT(p) BMO(p) NO


Australija 21,45
Belgija 22 15,39,64 34
Bugarska 35,44,50,81 53,60 46,66
Čehoslovačka 32 12,17,48
Francuska 71
Grčka 79
Holandija 70b 65
Hong Kong 69
Irska 8,37
Jugoslavija 38 47 82
Južna Koreja 9,78
Kanada 56
Kina 80
Kipar 55
Kolumbija 49
Kuba 52
-
Madarska 7,19 67 2a,18,24,30,
31,42,62
Novi Zeland 4
Poljska 20 23
Rumunija 29,59 2b,25,40,41, 46 10,27,28,
57c 57ab
S.A.D. 6,16,26,58,
63
SR Nemačka 5,72,75 3
S.S.S.R. 36 13,33,51,77 1,61,68
Švedska 43
Turska 83
V.Britanija 76 11,14,70a 73
Vijetnam 54,82 74

-
Legenda: MMO – zadatak na Medunarodnoj -
olimpijadi, pMMO – predlog za Medunarodnu olimpi-
-
jadu, MMT(p) – zadaci na medunarodnim takmičenjima 1980. i predlozi, BMO(p) – zadaci na Bal-
kanskim olimpijadama i predlozi, NO – nacionalne olimpijade

132

You might also like